Você está na página 1de 196

M. L. Krasnov G.I.

MaKarenKo
A.I. Kiseliov
;

CALCULO
VARIACIONAL
(ejemplos y problemas)

~ Editorial MIR

Loe autores de este libro &un


Mijafl i<tasnov 1 Grigori Makn
~,ko, candidatos a Doctores en
Ciencias fisicomateuyticas y docentes cl~l Instituto En~tfoo
de Mosc, y Alexandr Kiseliov ,
colaborador cienHlicu superior
del Instituto Unif~ado de In
vestigaefones Nucleares de Ja
ciudad de Duhna.
Este compendio contiene pro
blemas y ejercicios dedicadM "
ilustrar los diferentes principi<lS
de la teor(a y los mtodos de
resolucin de las ecuaciones por
el clculo de variaciones.

M. L. Krasnov, G. l. Makarenko,
A. l. Kiseliov

CLCULO
VARIACIONAL
(ejemplos y problemas)

Traducido del ruso por Carlos Vega j


candidato a doctor en ciencias fsico-matemticas

Editorial MIR

l.S.B.N.: 84-(>()4..1605-4
D.L: M-5299-1992
@F.ditorial MIR- 1992
Imprcsi6n: Grficas Zenit. Madrid

Prefacio a l a edicin espaola . . . . . . . . . .

Observaciones pre limi nares . . . . . . . . .

Capitulo l. Extre mo de funciones de varias variables


l. E xtre mo inco ndic ionado
2. Extremo c.ondicionado . . . . . . . . .
Captulo l J. Extremo de funcionales
3. Funcion al. V ari ac in de una [unc io nal 'i s us p ropie
dad es
. . . . . . . .. . .. .. . . . . . .
4. Pro blema ele ment al dt>I Clcu lo va riac ro na l. E<::uacin de Eulcr . . . . . . . . . . . . . . . . .
5. Generalizaciones del prob lema ele mental del Clculo
vari acional . . . . . . . . . . . . . . . . . .
6. l nvariancia de la ecuacin de Eu ler .
7. Campo de ex t remales . . . . . . . . . . . . . .
~ 8. Condiciones suficientes de extremo de una funcional
9. Ex t remo co nd iciona do . . . . . . . . . . . . .
10. Problem as varacionales con fron terns mviles . . .
l l. Prob lem as discontinuos. Var iaciones unilaterales
12. Teor a de H a m ilton - J acobi. Principios variacion ales
de la Mecnica . . . . . . . . . . . . . . . . .
Captulo 1 Jl. M todos directos en el Clculo var iacion al
13. Mto do de difere ncias fini tas de Euter . .
14. Mtodo de R. ilz, Mtodo de Kantorv1ch .
15. Mtodos v ariacion ales para la determinacin de los
va lore~ y de las funciones propios . . . . . .
Res pues tas e indicaciones . . . . . . . . . . . . . . .

11

HI

26

50
68
77

80

106
121

134

143

15B
160
167

180

BIBLIOGRAFJA
1. Couran t .R . y D. Hilbert , .Methoden der Mathematischen Physi k
(Mtodos de la Fsica Matemtica), vol. 11, Spr inger, Berlin, 1937 .
2. L. Elsgoltz , Ecuaciones difer enciales y clc:ulo variacional, Editorial MIR, Mosc, 1969.
3. 11 . M . I'e111J</JaHO u C. B. c/JOA!Uk, BapHal(KOHHoe JtC'lHCJieHHe ,
Cl>l:laMaTrH3 , 1961 (/. M. Ouelfand y S. V . Fomn. Clculo variacional).
4. E . Goursat, Cours d 'an al yse mathma tique <Curso de Anl isis
Matemtico), vol. IJI, 5 ed .. GauthierVillars. Pars. 1942.
5. M. A . Jlaep eHm t.ee u .ll. A. JliocmepHUK, Kypc eapHaUHOHHoro
HC'UICJl('fflUI, rocTeXH:JAaT, 1950 (M. A . Laurientiev y L . A . lustern ik, Curso de clculo v ariac ion al ).
6. J . Rey Pastor, P. Pi Calleja, y A. C . Tre;o, An lisis matemt ico,
vol. 111, 211 ed. , K.apelusz, Buenos Aires , 1961.
7. L. C. Y oung, Lectures on the calculus of variat1011s an d op t i ma!
control theory (Lecciones sobre el clculo variaclonal y la teori a
de co ntrol ptimo) , Phil adelp h ia, London, Toronto, 1909 .

PREFACIO A LA EDIC1N ESPANOLA


Hoy da todo ingeniero tropiez a frecuentement~ con
problemas que requieren buenos conocimien tos matemticos
y perici a en la aplicacin de distintos mtodos matemticos.
Se puede afirmar que la elevacin de la cultura matemtica
de los ingenieros contribuye a nuevos logros en la Tcnica.
El Clculo variacional es uno de los captulos del Anl s ic;
Matemtico clsico ms importan te para las aplicaciones.
Actualmente, en varios Institutos politcnicos e1 Clculo
variacional se incluye en el programa obligatorio del curso
de Matemticas su periores. Existen valiosos libros sobre el
Clculo variacional como, por ejemplo, los libros de L. Elsgoltz [2}, de E. Goursat [4), de L. C. Young [7] , etc. En cuan to
a los problemas, muchos de ellos aparecen disemin ados en
numerosos te xtos o artculos cientficos especi ales dedicad os
a este tema. Pero, por lo que conocemos, no exist.e en la
literatura correspondiente ningn libro de probl emas ded icado especialmente al Clculo variacional. Los autores se
han planteado la tarea de preparar un cierto rnnim0 de
problemas referentes a los captulos pri ncipales del Clculo
variacional cl sico (sin tocar las cuestiones relacionadas. con
la Teora de direccin ptima).
El libro est escrito de forma que al principio de cada
pargrafo se dan los elementos tericos ind ispensables {defi niciones, teoremas y frmul as) y se anal izan deta lladamente
ejemplos t picos.
El libro contiene ms de 100 ejemplos r esue lto~ y 230
problemas para eJ trabaj o individual. Al final se dan las
respuestas a todos problemas y, en algunos casos, las indicaciones correspond ientes. Por eso, el libro pued e servir tan to
para estudiar asignatura individualmente como para proftmd izar en el material que se expone en las lecciones.

PREFACIO

Consideramos deber nuestro agradecer al traductor Carlos


Vega, candidato a doctor en ciencias fsico-matemticas, por
el gran trabajo que se ha realizado al revisar los problemas
y por las tiles observaciones que ha hecho contribuyendo
al mejoramiento del libro.
M. L. Krasnov
O. /. Makarenko
A. 1. J( iseliov
.~'\osc,

30 de Octubre de 1974.

OBSERVACIONES PRELIMINARES
l. Si A es un conjunto cual qu iera de elementos, la proposicin
cel elemento a pertenece al conjunto A se representa simblicamente as:
a E A.
Si se escribe a E A (o bien a E A). ello significa que el elemento a
n o pertenece al conjunto A .
Siendo A y 8 dos conjuntos, la proposicin A es un su bconjunto
del conjunto 8'1 (A e B, s imblicamen te) significa que todo elemento x
del conjunto A tambin pertenece al conjunto B.
2. La unin y 1a interseccin de dos conjuntos A y B se defi nen
del modo siguiente:
la unin A U B = {x 1 x E A o x E 8} es la totalidad de los
elementos x que pertenecen por lo menos a uno de los conjuntos A o B:
la interseccin A n B = {x 1 X E A. X E B} es la totali dad de los
elementos x que pertenecen tanto a A como a 8 .
3. Si A es un conj unto formado por nmeros reales, se deno mi na
cota superior (cota superior exacta) de A el menor nmero real M tal
que a~ M p ara todo a E A. En otras palabras, M es la cota superior
de A si para lodo a E A es a ~ M y si para cualquier s > O, por peque
o que sea, existe como minimo un eleme nto b E A tal que M - e < b.
SI no existe tal nmero, convendremos en decir que la cola s uperior
de A es
En ambos casos designaremos la cot a superior del conjunto A
por s up A.
Anloga mente se define la cota inferior de l conjunto A que se represen ta por inf A.
4. Se denomina espacio lineal todo conjunto R de elementos
x, y, z, ... de naturaleza arbitrarla para los cuales estn definidas dos
operaciones, de adicin y de multiplicacin por nmeros, que cumplen
los axiomas siguientes:
!~ X
!1 = y
x;
2 (x
y) + z = x
(y + z);
3 existe un elemento O (elemento nulo) tal que x
O= x para
todo x E R:
4) para todo x E R existe un elemento - x (elemento opuesto)
tal que x
(-x) = O;

+ oo.

++

5) l

X =

x;

a (Jlx) = (a~) x;
7) (a
~) x = ax
8) a. {X+ V) = l;tX
6)

+ ~x;

+ ag.

JO

OBSEHVAC JONES P RELlM I NARF.S

5. Vil espacio lineal R se llama normado si a todo elemento x E R


U x \l. llamado norma de
este <.>lemento, con la particulari dad de que
1) x U = O slo s i x = O;
2) Ct X IJ = 1 et. l 11 X 11 ;
3) x + y 11 ~ 11x11
11 y11 (axioma triangular p ara las nor
ma s):
6. Un conjunto M de elementos x, y , z, . . . de naturaleza arbitra
ria se denom ina espacio mtrico si a todo par de elementos x, y de M
le corresponde un nmero r eal no negativo p (x, y) de modo que
1) p (x, y) = O s i, y slo si, x = y (axioma de identidad):
2) p (x, y) = p (J, x) (axioma de simetra);
3) (l (x , y)
p (y, z) ;.. p (x , z) (axioma triangular).
El n mero p (x, y) lleva el nombre de distancia entre los elemL11los
X e y.
Toao espado lineal normado es mtrico : basta tomar p (x, y) =
= 11 X - IJ 11
7. El espacio e [a, b] es<.>! espacio formado por todas las fun ciones
,e (x) con tinuas en {a., b] donde
le corresponde un nmero real no negativo

11 !I lle =

mx

o ~x~ b

ly

(x [.

El espacio C 1 [a , b] es el espacio formado por todas l as funciones

y (x) que, a parte de ser continuas, tienen derivada primera cont inua

en [a, bl donde
11 Y llc 1 =

mx

a ~:x: < b

l Y (x) 1 +

mfix

a:% x <b

1 y' (x) 1.

El espa.cio Cn ta, b] es el espacio formado por todas las runclones


y (x) que, a parte de ser continuas. tienen en (a, b] derivadas continuas
has ta de orden n-simo incl usive (fl es u n n mero natura l fij o) donde
1\

11 Y /len= ~ mx 1ch.) (x) I


k = O Q~X~b

A veces la norma del ele.mento y (x) en Cn [a, b) se define as:

11 Y llcn =

mx { 1Y (x) J,

a::!'oc~b

1y' (x) , . . . , 1y <n> (x) 1}.

Captulo I

EXTREMO DE FUNCIONES DE VAR.IAS VARIABLES

l. Extremo inccndicionado
Sea f (x1 , x 2, , Xn ). o brevemente f (x), una funcin definida
en un recint o D del espacio eucldeo En de n dimensiones.
Diremos que la funcin f (x) alcanza su valor mximo (mnimo)
en el punto x0 E D si

(x) ~

(f (x)

(xo)

(xo))

cualquiera que sea el punto x E D.


TEOREMA DE BOLZANO-WEIERSTRASS. Toda funcin continua en
un recinto acotado cerrado alcanza en l sus valores mximo y mnimo.
DEF'INICION 1. Sea f(.x) una funcin de[inida en un recinto De E <m.
Diremos que el punto .x< 0 > = (x? .... ~) E D es un punto de mximo
estricto (un punto de mnimo estricto, respectivamente) de la funcin
f (x) si existe una vecindad U (x<>) del punto x01J tal que la desigualdad
f (x) < f (x<0 >) (la desigualdad f {x) > f (xt >), respectiv a~nte) se
cumple para todos los puntos X E g (x<>) n D. X =t= x<>. Es decir, lo
que caracteriza el punto de mximo estricto (el punto de mnimo estricto, respecti vamen te) es que
f = f {x) - f (x<0 >) < O (t./ > O, respectivamente)
para todo X E Q (x~l) n D, X#= x<0 1.
En cambio, si para el punto x<> existe una vecindad Q (x<0 >)
tal que para todos los puntos X E Q (x<l)
D se cumple la desigual dad
f (x) ~ f (x<>) (la desigualdad f (x) ;;;.: f (x<0 >), respectivamente). se
dice simplemente que el punto x<> es un punto de mximo (un punto
de mnimo, respectivamente).
DEFINIClN 2. Los pun tos de mximo y de mnimo da la funcin
f (x) se denominan punt<Js de extremo de la misma.

J. Basndose en la definicin, hallar Jos puntos de extremo de las funciones


a)

f (x,,

b) f(x ..
e)

f (x,,

x 2 ) = x~ + x~;

x2)=
x2 )

xr + Xi
1

Si
si

xi+x2=0
l

= x- x~

en el recinto D {x:

Xt + Xi=#= 0,

+ x~ ~

1}.

'

CAP. l. EXTRf>MO DE FUNCIONES

12

TEO~EMA 1 (condicin necesaria de extremo). S ea


{XI> Xi, . . . , Xn). un.a fun cin definida en una vecnda

x'> =

(x~.

(x), x

del punto

xg, . . . , xg). Si este punto es un punto de extremo de la

funcin f (x)

y.si en l existen las derivadas o'

Xj

ellas son iguales a cero:

of "(xiO>) =

VXj

(i =

1. '?, .. ., n),

todas

(. 1 2
)
1 = ' ' , n.

Sl la funcin f (x) es diferenciable en el punto de extremo x< 0>, su


diferencia l en. este punto es igual a cero: df (xi>) = O.
EJEMPLO 1. Hallar los puntos de exti:emo de la runcin z = x2
y'!..
SOLUCION . Los puntos de extremo estn entre los puntos para los
cuales dz = O. En nuestro caso, dz = 2x dx
2y dy. La condicin
dz = O se cumple en el punto x = O, y = O solamente. En erecto, si
x = y = O, tenemos dt = O. Recprocamente, sea dz = O; basndonos
en que dx y dy son arbitrarios, tomemos dy = O de modo que O = dz =
= 2x dx de donde, puesto que dx es ar bitrar io, resulta que x = O;
anlogamente encontramos que tambin y = O. En el punto (0, 0)
tenemos z = O; en todos los dems puntos tenemos z = :<'2
yZ > O.
Por eso, el punto (0, O) es un punto de minimo estricto de l a fu ncin

= xz + y2.

Si se ampla la clase de funciones en la que se busca el extremo


incluyendo en ella las funciones no dilerenciables en alg unos puntos,
se llega a la sigu iente condicin necesaria de extremo.
Si x< 0 1 es un punto de extremo de la funcin f (x1 x2 Xn)
cada una de las derivadas parciales

:1

vX

(i

1, 2 , .. ., n} es igual a cero

o no existe en dcho punto.


EJEMPLO 2. Consideremos la parte superior z ;:;;.- O del cono z2 =
= xll .yi. Es obvio que la funcin z tiene. mnimo en el punto (O, O).

Pero las derivadas :; y :: no existen en est.e punto.


DEP!NIClN 3. Los puntos en los que se cumple la condicin necesa
ria de extremo de la funcin f (x) se denominan puntos crticos de la

misma.

Los puntos x< 0 1 en los que df (x>) = O se denominan puntos esta


cionarios de la funci n f (x).
La condicin df (x<>) = O es equivalente a la condicin

of (x<Ol)
ax

(i=I, 2, ... , n).

La existencia de punto crtco no garantiza an la existencia del


extremo de una funcin. P or ejemplo, el punto (O, O) es un punto es tacionarlo de la uncin z = x - y'J. y, sin embargo, la fun cin z no
tiene extremo en l: en cualquier vecindad del punto (O. O), por pequea
que sea, la funcin toma valores tanto positivos como negativos.

13

EXTREMO I NCONDICION 1\DO

1. Condiciones suficientes de extremo estricto


DEFIN JCION 4 . Se dice que la forma cuadr lca
n

A (x) = A (xi.

X2, . . ,

Xn) =

_;

i, ;-

a ""' a;

i,

OXX ;

= 1, 2, . . . , n:

es definida positiva (definida negativa, r~specliv amenl e) si A (x) >O


(A (x) < o respectivamente) para todo punto X E
X -:::fo o, y se
an ula slo para x = O, o sea, para x1 = x9 = . . . = x11 = O.
La for ma cuadrt ica se denomina no negativa si jams toma valores
negativos. Por ejemplo, las formas

en,

son ambas no negat iv as. La primera es definida positiva ya que se


anula slo para x 1 = x 2 = ... = X n = O; en camhio la segunda no lo
es ya que se anula, por ejemplo, para x1 = l. x2 = - 1, x 1 = x 4 =
= ... = Xn = 0.
Una forma cuadrtica definida positiva o definida negativa se
denomina forma cuadrtica defi nida .
Una forma cuadrtica que toma v alores tan to positivos como
nega tivos se denom ina indefinida.
TEOREMA 2. (condiciones . s uficien tes de; extremo estricto). Sea

(x) un.a funci11 de{in ida en una uecindad del punto x'' =
xa ... , xft) en la que son continuas sus segundas deriuadas y sea
x <> un p unto estacionario de la funcin f (x) . Si la forma cuadrtir.a

=0 (xy,

( 1)

f en el punto xi 0 >, es def inida


positiva (definida 11egativa, respeclivamenle). el punto x t0 > es punto de
mnimo estrcto (punto Je mdximo tstriclo, respectivamente) : sl la forma
cuadrtica (1) es indef inida, no hay extremo en. el punto x0 >.
o sea , la segunda diferencial de la funcin

CRITERIO DE S'\'L V ESTER DE FORMAS CUADRAT ICAS DEF INIDAS


POSITIVAS. Condi cin necesaria y suficie11te para que la. forma cuadrtica
n

A (x) =

11 (Xi> X2, , x 11 )

~ a;XXj
' 1-= l

(2)

14

C.Ar

bX l IH'.MO DE FUNCION!.':-

l.

cott au = J;; i, ; =- 1, 2, . . . , u; sea definida pusiliva es que se cumpla


11 12 a13

21 a22 a23

>

O,

a31 a32 :l3

au

a12 1n

>0-

.... '
~

Condici6n necesaria y suficiente para que la forma cuadrtica (2)


sea de{ inida nega/illa es que se cumpla
11 12 a13
a21 022 a23

<O,

a31 a32 a33

au

12 - tn

a21 a2z .

zn

a,.1

a,.u

an2

CASO n2- Sea f (x, y) una funci n definida en una vecindad del
punto (x0 , y 0 ) en la que son continuas sus derivadas parciales de segundo
orden y sea (x0 , y 0 ) un punto estacionar io, es decir, sea
f~ (xo. Yo) =
Entonces, si en el punto (x 0 , y 0 )

l'!cxf'11

(xo. Yo) = O.

{/';11)

> O,

hay extremo en este punto; a saber, mximo si en l


y mnimo si en l

/';ex

<O

({~11

< O)

f~x

>

(f'jy

> 0).

Si en el punto (x 0 , Yo)

r;x1'11 - <f'~:v>" < o,


no hay extremo en el punto (x0 , y 0). P or ltimo, si en el punto (x 0 , Yo)
f';.:itf;y - (/';.,11) 2 =

15

EXTREMO 1 NCONDICIONADO

i 1

en dicho punto puede haber extremo y puede no babe.rlo; este caso


requiere un estudio complementario.
. EJEMPLO 3 . Consideremos 1as funciones z = x'
y4, z = -x' - y4
y z = x4 - y4. El punto (O, O) es un punto estacionario de las tres y
11
en l se tiene z'.~:xZv - (z;11) = O para cada una de las funciones.
Es fcil ver que el punto (0, O) es un punto de mnimo de la pri
mera fun cin. un punto de. mximo de la segunda y no es punto de
extremo de la tercera. Efectivamente, en Jos tres casos tenemos
2 (O, O)= O; sin embargo, en cualquier vecindad del punto (O, O),
a excepcin del propio punto, )os valores de la funcin son positivos
en el primer caso y negativos en el segundo mientras que en el tercer
caso la fu ncin z = x' - {/' toma, en cualquier vecindad del origen
di :::oorde11adas, valores tanto positivos (por ejemplo, si x =FO e y = O)
como negativos (por ejemplo, si x = O e y =FO).
t::JEMPLO 4, Hall ar el ext remo de la funcin de tres variables

f = x + y 2 + :z -

xy

+x-

2z.

SOLUC IN. Determinamos los puntos estacionarios de la funcin

f.

Consideremos con este fin e\ sistema de ecuaciones

:~
of

<)y

of

oz

resolvindolo,

cocont ramo~

= 2x-y+ 1 = 0,

)i

= 2y-x= O,
= 2Z-2= 0;
2
x0 = ,

J1
1

Yo=-3 y

Zo =:

l.

Consideremos lii forma cuadrtica (1) en el punto P0

--} , 1).

Tenemos

r;.x=

2,
1,
f ;x= O,
encontra mos
au = 2,
au = -1 ,
f~x= -

En el punto P 0

a1

de motlo que
11

>o,

O,

f;u =
fJ11 =

-1 , f;z= O,
2, fj'l. = 0,

!;11=

O. / ;z=2.
- 1,

a 13 = O,

2,

023

'=o,

~3

111.

a,,=

1 2 -11-1o
au Gz
21 22

- 1

1-

= o,
= 2,

2 - 1
3 --... 0
l
2 - ~

= 6 > 0.

( -

E.- ,
.)

16

G,\P . l. EXTR EMO DE FUNCIONES

Basndose en el criterio d~ Sylvester, llegamos a la conclus in de que


la (or ma cuadrtica es definida positiva; por lo t anto, en v irtud del
teore ma 2, el punto P 0 es un punto de mnimo estricto s iendo f (P 11 ) ==
4

= - 3

EJEMPLO s. Ha llar el ex t remo de la funcin de dos variabl es

z=x 3 y2 (6-x - y).I


Determinamos los puntos estacionarios;

SOl.UCl N.

3x2y 3 = O, }
12x'.ly - 2x'y - 3x3y2 = 0,

z~ ,_, 18x2y 2

Zy =

4xy2

de donde x 1 = O, y1 -: O y x2 = 3, yz = 2. H emos obtenido dos puntos


estacionarlos P 1 (O, O) y P2 (3, 2).
Ca lculc:>mos l as segundas d!!ri Vadas de la fun cin:

z;;.x = 36xy2
ZJy

i;y =

l 2x2y:i - 6xy3,
1 2.~~ - 2xt - 6x3 y,
3Gx2y - 8x3 y - 9x:ly2
-

En el pun to P 1 tenemos z;..x = z"yu = z;. 11 = O de modo que


(z;11 )2 =O y queda pendiente e problema sobre la existencia
de ex tremo en es le punto; para resolverlo hab r que rec urrir a las
derivadas superiores.
En el punto P 2 tenemosz;x = -144, zyy = -1 62 y z;y = - 108.
Queda daro que ~xZVv . - (z;;11 )~ >O y como z"xx < O, en el pun to
P 2 l3, 2) hay mximo siendo zmx = 108.

z~xzZ11 -

Hall ar los mximos y los mnimos de las funciones:


2. f = (x - l )2 - 2y 2
3. f = x 4 + y 4 - 2x2 + 4xy - 2y2
4.

+ y2) e-<x2+112>.

(X2

1+x -y

s. f =V l +x2+y2
z2

y2

6 . f=x+4X+y-+-z(x>O , y>O. z>O}.


7.
8.
9.

f
f
f

= x2
=

-=

xy

+y

2x

+ y.

sen x sen y ~~n (x + y) (O ~ x ~ n, O ~ y


x1x; ... x~ (l - X1 - 2x2 - . . . - nxn )
(X1

:>O,

X2

>O, . . ,

X 11

>O).

~ :i)

EXTREMO INCONDICIONADO

l.

17

10. Demostrar que la funcin z = (1


e11) cos x - ye"'
tiene una cantd atl infinita de mximos y no tiene mnimos.
11. Ser condicin suficien te para que la funcin f (x, y)
tenga mnimo en el punto M 0 (x0 , 9 0 ) el que esta funcin
tenga mnimo a lo largo de cualqu ier recta que pase por el
punto M 0? Considerar la funcin f (x, y) = (x - y 2 ) x
X (2x - y2) .
12. Demostrar que (a d iferencia de las funciones de una
variable) incluso para l as funci ones de dos variables la existencia de un extremo nico - mximo o mnimo - en un
recin to D no significa aun que este extremo represente el
valor mximo o minimo de la funcin respecto a todo el
recinto. Considerar los ejemplos:
a) z = x'.! - y'.!
2e-.x2,

-oo

b) z =

x3 -

4xll

<x<+

+ 2xy -

oo, -oo

<Y<+

oo;

y",

D {-5 ~ x ~ 5; - 1~y~1}.
13. Sea f (x) una funcin peridica con perodo 2n. Entre
todos Jos polinomios trigonomtricos de grado n
n

; + ~ (a.h cos kx + ~h sen kx)


h=I

determinar, escogiendo convenientemente los coeficientes a.1;


y ~'" aquel que ofrece el valor mn imo para el error cuadrtico definido por la igualdad

6!= 2~

-n

[t (x) - i - ~ (akcoskx+~1tsenkx) J2 dx.


k~I

2. Mtodo del gradiente. Suponga mos que es preciso hall ar e l


mfnimo de la fun cin f (x), donde x = (x 1, x 2 . , xm) Tomemos
un punto xO = (xf, xU ... , xi.> y calculemos el gradiente de la uncin
f (x) en este punto
m

grad f (x) =

~ ~~X:}
i- 1

ei.

donde ei. e2 . , en es una base ortonormal del es pac o R. 11


Si se tiene gr ad f (xi>) :: O, ponemos
xi = x - h1 (grad f (xO), e1i) (k = 1, 2, ... , rn) ,
2-01387

GAP. l. EX TREMO DE FUNCION ES

18
donde h1
ponemos

>

O es suficientemente pequeo. S i se tiene grad

en general, s i se llene grad


x~

(xl)

=f=. O,

f (x11 - 1 ) =f=. O, ponemos


hn {gr ad f (xtt-1), e,.)

x~- 1

(k = 1, 2 , . . , m ; hn >O).
As obtenemos. si se cumplen determinadas condiciones, una su

cesin montona decreciente {/ (x")}. SI x" y X' es un punto de


ml nl mo de la funci n f (x), se tiene g_rad f (x11) - O cuando n-+- oo.
EJEMPLO 6 . Halla r el punto de mnimo de la funcin f (x) = x.
SOLUCIN. Tomemos, por ejemplo, el punto x:O = l. Tenemos
grad f (x"} = 2x"I = 21 =F O.
Por eso, ponemos
x1 = x;O -h2=1 -211 . donde h >O.
Tenemos ahora
grad f (x1) = 2 (1 - 211) l .

Si h =F

!,

es grad f (x1)

=I= O y

ponemos

x = x - 2h (l - 2h) = ( 1 Con ti nuando este proceso, encontramos


xn = (1 - 2h)11
2

Es claro que, s iendo O <

2h)2.

Ti< \,

se tiene xn - O para 11- oo. El


f (x) = xi. Si /1 = ~,
O y obtenemos la sucesin estacionaria {O}

punto x = O es el punto de mnimo de la funcin

es x1 = O y grad f (x') =
cuyo li mite es el cero.
EJ.EMPLO 7. Hallar el punto de mlni mo de la funcin f (x, y) =
yz.
=
SOLUC ION. Tomemos, por ejemplo, el punto (1, 1), o sea, tomemos
x0 = 1 e IJ = l. Tenemos
grad f (1, 1) = 2
2j.

r+

Pues to que grad f (!, 1) =/=O, ponemos


x1
y1

Tenemos

= x0 = y -

2xll

2yh

=
=

1 - 2h,
1 - 2h.

(h >O)

grad f(xt, y 1) = 2(1-2h)i+2(1- 2h) j 4: O

y, por eso, lomamos


xi= xl- 2xlh = (1--2h)2,
yi= y1-2y l / = (l -2h)Z.

19

EXTREMO CONDICION ADO

2.

Continuando este proceso, encontramos


xn = (l - 2h)n,
yn = (1 - 2h)n,
de modo que para O < h < 1 obtenemos una sucesin de pun tos
Mn (x"', y11) convergente al punto M (O, O) de mlni mo de la funci n
considerada. Es obvio que
gr ad f (xn, yn) = 2 (l - . 2h)n l
2 (l - 2h)"' j-+ O cuando n-.. oo.
Es decir, el punto (O, O) es el punto de mnimo de la funcin f (x, y) =

=xi+ y'l..

Empleando el mtodo de gradiente, hallar el punto de mni mo


de la funcin
z = x2
y'J. - 2x
4y
5.

+ +

2. Extremo condicionado
Sea z = f (x1 , x2, . , Xn) una (uncin de n variables definida d~
un recinto D del espacio En.
Supongamos, adems, que las variables x1 , x2> .. .-e71 estn
ligadas por (m
n) condiciones complementarias

<

:1?~' .X~> ~ : . ~~):=~}

(I)

q>m (x,, X2, , Xn) = O,

que se denominan ecuaciones de enlace.


Sea x<0 l = (xy, x~ ..., x~) un punto interior del recinto D.
Se dice que f (x1 , x9 , , xn) tiene mdximo condicionado (mnimo
condicionado, respectivamente) en el punto (x~. tj, .... xYi) si la de
sigualdad
f (xi. X2, , xn) ~ f M, xK .... x~)

>

desigualdad f (x1 , x 2. , xn)


f .(x~. xi, . ... x~). respectivamen
se cumple en una vecindad del punto (x~, x~, ... , x~) siempre que
puntos (x1 , x 2 , , Xn) y (x~, x~, .... xi) verifiquen las ecuaciones
enlace (1 ).
EJEMPLO 1. La funci n z = x3
y"' tiene mfnimo incondidonado,
igual a cero, en el punto (O, O). Agreguemos la ecuacin de enlace
x y - 1 = O; se trata entonces de deter minar el mlnimo de las
Z-coordenadas de los puntos de Ja s uperficie z = x'
y'l. considera ndo
slo aquellos valores de x y de y que satisfacen la ecuacin x
y- 1=
= O. Este mnimo condicionado no se puede alcanzar en el punto
(O, O) pues este ltimo no satisface la ecuacin de enlace. Resolviendo
la ecuacin de enlace x
y - 1 = O respecto a y e introduciendo la
expresin obtenida y = \ - x en la ecuacin de la superficie, encontramos z = xt.
(1 - x)t., o sea, obtenemos una funcin de una

{la
te)
los
de

variable. Extremndola, encontramos

Xcr

~ y zm 1a = ~- A partir
~

20

CAP . l. EXT REMO DF. FUNCI ONES

de la ecuacin de enlace, de te rminamos Ycr

=;.

El punto (

~,

; , ; )

es el vrtice de la par bola que corresponde a Ja interseccin del paraboloide z = x~


y'I. con el plano x
y - 1 = O.

Anlogamen te se puede proceder en situaciones ms gc.>nerales.


Supongamos que se busca el extremo condicionado de la funcin
z = f (x, y) s iendo qi (x, y) = O ta ecuacin de enlace. Supongamos que
para los valores considerados de x y de y la ecuacin q> (x. y) = O determi na y como una fun cin unvoca diferenciable y = 'I' (x) . Sustituyen
do y por 'i> lx) en la funcin f lx, y), obtenemos una funci n de una
variable x; z = f (x, '!> (x)) = F (x). El extremo (incondicionado) de
la funcin F (x) sera el extremo condicionado buscado de la funcin
f (x, y) co n Ja condicin de e nla ce qi (x, y) = O. En la prctica este
mtodo resu lta poco cmodo ya que pa ra a plicarlo es preciso resolver
la ecuacin <p (x, y) = O res pecto a una de las vari ables.
Para hallar los extremos de la funcin z = f (x1t 2 , . . . , Xn) con
las condiciones tle. enlace (i) se em plea el mtodo de los multiplicadores
de Lagrange.
MeTODO DE LOS M.ULl'IPL ICADORES DE LAGR.l\NO.E. Supongamos que
1) las der ivadas pardales de primer orden de l as runciones
f (Xt. x2, . ... Xn) y cp, (x1. x 2, .. . , Xn) (i = ! , 2, . . . , m) son contin uas
en el recinto D;
2) m

< n , :mmdo el

rango de Ja matr iz {

i:; )

(i

t , 2, ... , m;

= I, 2, . . . , n) igual a m e n todo punto del recinto D .


Consideramos una funcin nueva (fuucin de Lagra nge)
m

. '-.:'
<l> -=- 1+
L..J
'- '

h lp,

donde /..1 son factores constan tes in de termin ados.


Despus analizamos d extremo incondicio nado de la funcin
<P (XJ, x2 , . . . , x,.), o sea, formamos el sistema de ecuaciones

o<I>
OX1

o'11

=O, ax; = O, ' .,

c<I>
OXn

=O

(2)

y, a partir de este sistema y de 1as m ecuaciones de enlace


tp = O,
<jl 2 = O, . , 'Pm = O,
determinamos los valores de los parme tros /.1 A.2 , , A.m y las coordenadas (x1 , x 2 , ., xn) de los posibles puntos de extremo.
Las condiciones (2) son condiciones necesarias de extremo tanto
para la funcin de Lagrange como par a la funci n inicial z =

= f (X,

X21

Xn}.

Si el pu nto (x?, x~, .. . , x~) es u n punto de extremo condicionado


de la luncin l (xI> x 2, . , Xn ) , ser a la vez un pun lo estacionario de

1a runcin de Lsgr auge, o :,(!a, !!11 este punto s~r ~:. = O (i = 1, 2, . ..


. . , ri) . Para atw ltzar el punto estacionari o (xY. x~ ... x?1) en tanto

EXTREMO CONDICIO~A.00

2.

21

que ex tremo condiciona do de la funcin de Lagrange <ll(x1, x 2 ,


habr que considerar la for ma cuadrtica

x,,)

n m

B (d;ci, dx2 . . , d,'TI-m} =


f,

~ /J11 dx; dxj.


;-1

o sea, la segunda diferenci:il de la funcin de Lagrang<', teniendo en


cue nb las con diciones

a,.,

oq

v .\ t

vX2

;-dx1 + T dx2 ~

OfP
. .. \ -i)dx" =- O
Xn

(i = 1, 2, ... , m).

Si la forma cuadr lic11 (3) es definida, en el punto (x~. xg, . . . , x?i)


tendremos extremo condicionado es tricto; a saber, mximo condicionado estricto si la forma cuadrtica (3) es definida negativa, y mlnmo
condicionado estricto s i la forma cuadrtica l3) es defi nida posi tiva.
En cambio si la forma cuadrtica (3) es indefinida, en el punto
(x~. x~, ... , x~) no habr extremo condicionado.
Por consigu iente, la existencia en el punto (x'l, x~ . .. , ~) de
mxi mo (mnimo} incondicionado de la funcin de l.agrange (tomada
con los valores encontrados para ;>...11 1.3 , . . , ,m) implica la cxis~~ncia
de este punto mximo (mnimo) condicionado de 111 funcin z =
f (x 1, x2 , , Xn) con las condicione~ de enlace

Ql

(x 1 , x2 .... x,,)

= O

(l - 1, 2, .. ., m) .

La ausencia de extremo incondicionado de la funcin de

La~range

<t> (x1 , x 2 . . , .xn) no significa an la ausencia de ex tremo condicionado

de la funcin f (xlP X2, . . . , Xn):


EJEMPLO .2. Hall ar el extremo de la funcin z = xy con la condicin !J - X = 0.
SOLUC IN. Formamos la (uncin de Lagrange
<D (.x, y) = xy
(y - x}
y el sistema corres pondien te para determinar ). 'J lss coordenadas de
los posibles puntos de extremo

+ '-

~~ = y -

).= 0, }

a<D

Jy = x+ "' = O,

y- x= O.
La primera ecuacin da A= y. Tenindolo en cuenta, encontramos la
segunda ecuacin x + y = O. Es decir,

x+y = O, }
y - x= O.
de donde .x = y = O y, adems, A. = O. Por lo tan to, la funcin correspondiente de Lagrange es <D (x, y) = xy. La funcin <D (x, y) no tiene

extremo Incondicionado en el punto (O, O).

22

CAP. J.

EXT'REMO DE PUNCIONES

Sin embargo, exste el extremo condicionado de la funcin z =


con la condicin y = x: en efecto, tenemos en este caso z = x',
donde resulta que hay mfnlmo condicionado en el punto (O, O).
EJEMPLO 3. Hall ar el extremo condicionado de la funcin
f (x, y, z) = xyz
con las condiciones
<p 1 (x, y, z)=x+y-z-3=0, }
<f'2 (x, y, z)=x-y- z- 8 = 0.
SOLUCIN. Formamos la fu ncin de Lagrange
t!> (x, y, z) = xyz
J..1 (x
y - z - 3) + /.. 9 (x - y - z - 8)
y el s istema de ecuaciones para determinar los parmetros Ai y
y 1as coordenadas de los posibles puntos de extremo

xy

de

4
( )

=yz+t..1+~=0,

E~,,+

i..1

- ... -. 1
ai=xu-i.1-~ = 1

(5)

x+u-1-3 = 0,
x-y-z-8=0. .
Resolviendo el sistema de ecuaciones (5), obtenemos
11

A.1 = 32,

23 1

11

~=-321

x=4'

Y=- 2

11

Z= - 4

La segunda dierencial de la fllncin <I> (x, y, z) es ig..ial a


aiai
a2m
aZ<D

d2<I> =- dxz+ -ayz- dyz+-iJz2- azz +


axz
82(,I>

~$

(}Z$

, 2 ax oy dx dy -f-2 iJy iJz dx dz+ 2 ~ dy dz.

F.n nuestro caso,


d9<1> = 2z dx dy
2y dx dz
2x dy dz.
De las ecuaciones de enlace (4) encontramos

(6)

dx+ dy-dz=O, }
dx-dy - dz = O,

de donde dx = dz, dy
ob tenemos

11
(T

hay mximo, siendo /mu= 32 .

O. Introduciendo estas expresiones en (6) 1

8 (dx) = 2y dil.
En el punto estacionario se tiene B = -5 dx2 <O, o sea, en el punto

, - 25 , - 411

605

EXTREMO CONDICIONADO

2.

EJEMPLO 4. Hall ar el extremo de 13 funcin z


con la condici n
n

c0s2

x + cos2

y-x=T

SOLt:ClN.

La~range

Formamos l a [uncin de

<D(x, y)=cos2.x + cos2y+J.,

(u-x- ~ )

y el sistema de ecuaciones para deter minar el piira melro /, y


dl!nadas de los posibles puntos de ex tremo

~~ ~~ - 2cos xsen x-A.-0,

~~

la~

1.:oor

- 2 cos y sen y+ J...= O,

n 0,
.
y- x --;-=

es decir,
l7)

sen 2x = -'A.,
sen 2y= i.,
n

(8)

y-x=4,

(9)

De las ecuacionc!l (7) y (8) tenemos sen 2x

2sen (x
Debido a (9).

resulta sen (x

+ y) cos (y -

tenemos cos (y - x) =

+ y) = O, es
X + y ~ kn ,

+ S<'ll 'ly = o,

x)

o sea,

= O.

}~~-:::fo O y,

(1 0)

or eso, <le (10)

decir,
k

0, J, 2,

Resolviendo las ecuaciones (9) y (11 ), tendremos

Determinamos las sei,:u11 das derivadas de la fundn tJJ (x, y ):


cJ2<D
i)xZ

CJ2<D

= -2 cos 2x,

i}y2.

=-

kn

)2(D
i)x ()y

=0,

2 cos 2y.

En los puntos P-ii, ( 2 -- 8 ,

n )
zkn +s

se tiene

(l !)

24

CAP. J. EXTPEMO DE FUNCIONES

<D;~>~ V -

(tl>;,i)2 = 4 COS (

k!r. -

COS (

kn

)=
2cos 2kn

= 2> O.

Por consiguiente, hay exlr~mo condicionado en los puntos P11 - Adems,


para k = 2n es

a2a> 1

iJxZ Pzn

V 2 <o

= -

y, por l!so, en los puntos Pin se tiene mxi mo condicionado siendo

.."max -1
-V2 .
- + -2para

X=

2n+ 1 es

o sea, en los puntos Psn+ 1 tenemos mlnlmo condlclonado siendo


Zmtn=

-V2

1- - 2- .

En los prob lemas que siguen haHar el extremo condicionado.


14.
15.
16.
17.
18.
19.
20.

= xy s iendo xi+ yt. = l.

f = x2 + !f siendo ~ + ~ =l.
f = xyz siendo x +y + z = 5 y xy + yz + zx = 8.
f = ex11 s iendo x +y= a.
f = 6 - 4x - 3y siendo x 2 + y2 = l.
f = x - 2y + 2z siendo x2 + y 2 + z1 = 9.
f = sen x sen y sen z siendo x + y + z = ; , x > O,

y> O y z> O.
21. Demostrar la desigualdad
n~

l,

x~O

g~O .

2.

EXT R EMO CONDICIO NADO

25

22. Hallar el valor mximo del producto xyzt de cuatro


nmeros no negativos x, y, z y t si la suma de los mismos
permanece constante: x + y + z + t = 4c.
23. Hallar la distancia mfnima del punto M (1 , O) a la
elipse 4x2 + 9y 2 = 36.
24. Hallar la distancia de la parbola y = xz a la recta
x - y = 5.
25. Hallar los lados del rectngulo de rea mxima
inscrito en Ja circunferencia x2 +y"' = R'J.
26. Inscribir en la esfera de radio R el cilindro de mxima
superficie total.

Captulo 11

EXTREMO DE FUNCIONALES

3. Funcional. Variacin de una funciona l y sus


propiedades
1. Definicin de funcional. Proximidad de curvas. Sea M una
clase de funciones y (x). Si a toda funcin y (x) E M le corresponde,
segn una regll, un nmt-ro determinado J se dice que en la clase M
est definida la funcional J y se escribe J = J {y (x)].
La clase M de funciones y (x) en la que es t defini da la funcional
J fy (x)] se denomi n3 e.ampo de definicin de la funcional.
EJEMPLO l. St>a M = e
IJ el conjunto dt' todas las [unciones
continuas y (x) definidas en el segmento [O, I] y sea

ro.

J (y (x}l

= Jy (x} dx I>.

( 1)

o
Entonces J (y (x)} es una funcional de y (x): a toda funcin y (x) E
E C [O, 1J le corresponde un valor determinado J (y (x)]. Tomando en
(1) funciones concretas en lugar de y (x), obtendremos los valores corres
pon dientes de J 1y}. Por ejemplo, si y (x) = 1, tenemos
1

J[l)=

l dx=I ;

si y (x) =ex, tenemos


1

J{ex=

Je"'d~=e-1;
o

s y (x) = cos nx, tenemos


1

J(cosnx)=

Jo

cosnxdx = O.

1
) En adelante, al considerar funcionales in t~.~rales, escribiremos
en el ntegrando y en lu~ar de y (x), y' en l ugar de y' (x). y etc.

3.

FUNCIONAL.

VA~IACION

DB

UNA

FUNCIONAL

27

EJEMPLO 2. Sea M = C1 [a, b] la clase de funciones y (x) que


tie nen derivada continua en el segmento [a, bl y sea
J [y (x)l = y' {x0 ),
donde x0 E [a, bJ.
Queda cl aro que J [y (x)l es una fu ncional definida en la clase de
fu nciones sealada: a toda funcin de esta clase le corresponde un
nmero determinado, el valor de la derivada de esta funcin en el
punto rij o x0
Siendo. por ejemplo, a = 1, b = 3 y x 0 = 2, tenemos para
y (x) = x"'
J \x2 ] = 2x lx=:i = 4;

= 4 y par a y (.~) =
1
= In (1 x) tendremos J {In (l + x)] = l + x 1 x - Z= 31
EJEMPLO 3. Sea M =
l-1, 1j la clase de funciones y (x) con
tinuas en el segmento [-1 , l] y sea q> (x, y) una funcin definida y
continua para todos los -1 ::;;; x::;;; l y para todos los val ores reales
de y. En tonces,

para y (x} = x2

l encontr amos J {xi+ l I

j q> (.~. y) dx

J [y (x)] =

- 1

ser una funcional def inida de la clase de funciones ind icada. Por
ejemplo, si q> (x, y} =

~ yZ' para la funcin /1 (x) = x tendremos

J txJ =

X dx

i+x2 =0 y para y(x)=l + x

-1

tendremos
1

J [l -1-x)=

xdx

l +{l +x) 1

ln

Y5- arctg 2.

-1

e,

EJEMPLO . Sea M =
[a, b) la clase de funciones (x) que tienen
derivadas continua y' (x) en el segmento {a , bJ. Entonces

J fy(x})=

Vl +y'Zdx

(2)

a
ser una funcional defi nida en esta clase de funciones. Desde el punto
de vista geomtrico, la funcional (2) representa la longitud del arco
de la curva y = y (x) cuyos extremos son los puntos A (a, y (a)) y
B (b, y (b)).
~:l Se denomina variacin o incremento oy (x) del argumento y (x)
de la fun cional J [y (x)l la diferencia entre dos funciones y (x) e Yo {x)

28

CAP. 11. EXTREMO DE FUNC IONALES

pertenecientes a

!~

clase cons iderada M de funciones:

6y (x) = y (x) - Yo (x)

).

Para la clase de fun ciones k veces difercn riab les tenemos


(6y) <ll> = 6y<li> (x).
Diremos q ue las curvas y = y (x) e y= y 1 (x) definidas en el
segmento la, bl san cercanas en el ~ntido de proximidad de orden nulo
si es pequea rn la, b} la magni tud 1 y (x) - y 1 (x) 1- Desde el punto
de v isla geom(~triw , esto significa qu(' son prximas las ordenadas
de d ichas curvas en !a, bj,
Diremos qu~' la:> curvas y = y (x) e y= y 1 (x) defi nidas en el
seg men t o Ja, bl son cercanas en el sentido de proximidad de primer orden
s i son peq ueas en la. bl las magnitudes I y (x) - y1 (x) 1 y
1 y' (x) - Y (x) 1 Desde el pun to de vista geo mt rico, esto significa
que en la, bl son prximas t an t o las ordenad as de dichas curvas como
las direccione:> de sus tangentes en los puntos correspondientes .
Las curvas y = 11 (x) e y = y1 (x) son cercanas en el sentido de
proximidad de k -smo orden si son pequeas en (a, b] las miignitudes

1 y (x) -

!/1 (x)

1 y; (x) - Yi (x) l. . . .. 1 y<"-> (x) - y p11 (x) 1.

l.

Si las cu rvas son cercanas en el sentido de proximidad de k-simo


orden, con mayor razn lo sern en el sentido de proximidad de cual
quier inferior.
. temen l e gran de
EJEMPLO s L a curva y ( x) = sen nn'x con n su r1c1en
y la cur\'a y 1 (X) r.. O son cercanas en (O, n ) en el sentido de prox imidad
de orden nulo ya que

lu (x)-yi(x) I =

senn2x 1

1
~ n'

o sea, el valor al>soluto de esta diferen cia es pequeo en todo e l segmen


to [O, n i si n es su[ iden lemente gra nde.
No hay proximidad de primer orden ya que

1 y' (x) - Y (x) 1 = n 1 cos n'x 1


y, por ejemplo,

l'll

Jos r u nlos x

= !~

tendremos 1 y' (x) -

Y (x) 1 ""''

11, o sea, 1 y' (x) - y; {x) 1 puede resultar tan granlle como se quiera
si n es suidentement<.> grande.

EJEMPLO 6 . La curva y (x)

se~2nx con n s uficientemente gra nde

y la curva y 1 (x) -:= O son cercanas en (O,


de primer orden ya que. tanto

I !I

(x)-Yt (x) 1

nl en el s entido de proximidad

sen nx
112

<"

l
/j2

1
) Para abreviar, en lo que sigue escribiremos s implemente 611
en lugar de By (x).

'a.

FUNCIONAL.

VARIACIN

DE

UNA

FUNCIONA L

29

como

. 1= ICOStlX
1y, (.c} -yi(x)
- n- I

< n1

son pequeos.

Determinar el orden de prox imadad de las curvas en los


problemas que siguen.
cos nx

27. y (x) =

n~+i

sen X

28. y(x) =

n
X

29. y(x)=sen11

Yt (x) iii! O en (0, 2nl.

y 1 (x) =O

[O, n] .

en

e Yt (x) =O en (0, 1).

Se denomina distancia entre las curvo,s y::: y (x) e y= y 1 (.x)


y (x) e !Ji (x) son funciones continuas en [a, bJ,
el nmero no negativo p igual
y
al mximo del mdulo 1 Y1 (x)M(l,I)
- 11 (x) 1 en el
segmento
Q ~ X ~ b:
P= P IYl (x), Y (x)] =
= mx lyi(x)-u(x)j.
(a ~ x ~ b). donde

o~~b

EJEMPLO 1 .

Hallar la ds-

tancia p entre las curvas


y (x) = x e !/l (x) = x2 en el

segmen to [O, 11 (fig. 1).


SOLUCIN. Segn la definicin p = mh t
1. osea,

p=

O~x~I

mx
o~x~ l

cln y=

(.x -

r-x

x'). La fun-

)t

Fig. 1

x - x' se anula en los extremos del segmento {O,

I].

Determinemos el mximo de la funcin y= x - x2 en el segmento


[O, 1}.
Tenemos
1

y' = l-2x e Y'=O para x = 2


de modo que

En los problemas que siguen hallar fa distancia entre


las curvas en los segmentos indicados.

30

CAP. 11. EX TRF.MO DE PUNCIONAl, F.$

30. !J (x)

= xrx

3 L y (x) = sen 2x

32. y (x)

Yt (x)

!Ji (x) =sen x

== O en

!11

(x)

[0, 21.

en

~J.

[o.

In x en fe-

eJ.

Suponga mQS que las curvas y= y (x) e y = y1 (x) tienen en el


segmento la, bl derivadas continuas de orden n.
Se llama distancia de n-si mo orden entre las curvas y = y (x)
e y = YI (x) el mayor de los mximos de las expresiones
1 IJ1 (X) - y (x) , . 1 Yi (x} - y' (x) 1, .. ., l y\nl (x) - 11"" (x) 1
en el segmento [a, b). Rep resentemos esta distancia as

Pn

Pn \y t (x), IJ (x) )

mx

O~A~n

mx

a~x~b

1 y11 ) (x) -

y01) (x) 1.

Desde este pun to de vista se puede interpretar la distancia definida en


la pg. 29 como distancia de orden nulo.
EJF.MPLO 8. Hall ar la distancia de primer orden entre las curvas
y (x) = x' e y1 (x) = x3 en el seg mento O ~ x < 1.
SOl.UC.l\)N . Ca!cu!C?mos las derivad as de las funciones dadas:
y' (x) = 2x e Y. (x) = 3x2 y consideremos las funciones y, (x) =
= x1 - x3 e IJJ (x) = 2x - 3x11 Determinamos sus mximos valores
en el segmento [O, lj. Tenemos y = 2x - 3x 2
y

Fig. 2

lgua lan<lo e~t a deriva da a cero, encontramos los puntos estacio


narios de la funcin y 2 (x): x1
y1 I

2 =

~-3

..!.
27

O y Xz =

. !\ hora bien, Ys l.x,,.o =O,

y el valor dl' y 1 (x) en el extremo de la derecha es

3.

Ya (1)

F UNCIONAi..

VARIACIN

OE

UNA

fllNCIONAI.,

31

O. Por eso,

Determinamos ahora la distancia


y' (x) = 2x e Yi (.x:) = 3.x:':

Po= mx

0~%~ 1

Po de orden n ulo entre las derivadas

1 Y3(x)1 = mx l 2x-3x2
O ~x~1

Consideremos el grfico de la funcin y,= 1 2x - 3x1 1 {fig. 2). Puede


verse de l que
= 1. Por cons iguiente, la distancia p1 de primer
orden entre las cur vas y (x) = .x:1 e y 1 (x) = x:' se.r igul a

Po

Pl

33.
y (x) =
34.
y (x) =
35.
curvas

mx

<Pe. Po)=

l.

Hallar la distanc.ia de primer orden entre las curvas


1
, el.
In X e Yi (x) = X en el segmento
Hallar la distancia de segundo orden entre las curvas
x e y1 (x) = -cos x en el segmento [O, ~
Hallar la distancia de 1001-simo orden entre las
y (x) = ex e y 1 (x) = x en el segmento [O, l J.

re-

Se llama e-vecindad de n-simo orden de la curva IJ = y (x)


x ~ b) el conjunto de las curvas y = y1 (x) cuyas distancias de
ns imo orden a la cur va y = y (x) son menores que e:
Pn = Pn [y (x), Y1 (x)) < 6.
La 6-vecindad de o rden nulo se denomina e-vecindad fuerte de la
funcin y = y (x}.
La e-\ecindad fuerte de la curva y = y (x) est formada por todas
las curvas comprendidas en la franja de 2e de anchura construida a
partir de la curva y= y {x).
La &-vecindad de primer orden se denomina e-vecindad dbil de
la funcin. !I = y (x).
(a~

2. Continuidad de una funciona l. Una funciona l J (y (.x)) definida


en la clase M de funciones y (x) se llama conJnua en y = Ye (x) en
el sentido de proximidad de n-simo orden si cualquiera que sea el
nmero e > O existe un nmero TJ > O t-al que la desigualdad
1 J [y {.x)J - J (y0 (x)J 1 < & se cumple para todas las funciones admisibles y = y (X), o s(!a, para todas 1as funciones que satisfacen las
condiciones

1 Y (x) - !lo (x) 1 < 11

1 y' (x) - Y {x) 1


..

En otras parabras, si se tiene 1 J [y (x)l -

rr1

(y (x), !lo (x)l

< TJ,

1 y<ni {x) -

ut"> (x) 1 < 'l

J l!le (x)} 1

< e siempre que

< TJ

32

oe

CAP. TI . EXTREMO

FUNCIONALES

Toda runcional que no sea continua en el sentido de proximidad


de nsimo orden se denominar disconlintla e.n este seJ1tldo de proxi
mldad. Poniendo
y<I>.> (x)

y~"'> {x)

+ ct(l)<k> (x)

(k

O, 1, 2, . . . , n),

donde a es un parmetro y w (x) es una funcn cualquiera de la clase


M, podemos persuadirnos de que
lm y ' h} (x) = y~k) (x)

(k = O, 1, 2, .. , n)

a. ... o.,

y, por eso, podemos definir la continuidad lle Ja uncional J fy (x)]


en !J = Yo (x) de la forma siguiente
lm J (Yo (x} CXCI) (x}) = J {Yo (x)).
ci ...

Demostrar que la funci onal

EJEMPLO 9.

J {y (x)l =

I'

~ (!I +2y') dx

considerada en el espacio C 1 {O, 1(es continua en la funcian !lo (x)


x
en el sentido de proximidad de primer orden.
sotucrON Tomemos un nmero cualquiera e >O y demostremos
que existe un nmero TI > O tal que 1 J ly (x}l - J (x) < e siempre
que 1 /1 (x) - x 1 < 'l y 1 y' (x) - l 1 < 1}. Tenemos
1

IJ [y(x)J- J [xJl =I

J(g + 2y'-x-2)dxl ~
o

Jo

Jg-xJdx+2

Jly'-1 ldx.
o

Tomemos Tl=T Entonces, para todas las funciones y (x)E CdO, ll


tales que
e

)y(x)-x)<T

tendremos

l!l'(x)-11< 3

1 J [y (x)) - J [x) <s.

Es decir, para todo s

>O existe un

nmero 11 > O (por ejemplo,

TI = ; ) tal que l J {y (x)J - J [x] l < e s iempre que Pt fy (x), xj <


<TI Pero, segn la definicin, esto significa precisamente que nuestra
funcional es continua en la funcin !lo {x) = x en el sentido de proxl

1 8.

FUNCIONAL.

VARIACIN

DE

UNA

FUNCI ONAL

mldad de primer orden. Es fcil ver que esta funcional es continua e


el sentido de proximidad de primer orden en cual quier curva y (x)
E e, (O, l).
EJEMPLO 10. Consideremos la funcional

I [y (x)I

y' (x0 ),

donde las funciones !I (x) E e, [a, b] y Xo E [a, b).


Esta funcional es discontinua en el sentido de proximidad de
orden nula en cualquier funcin y (x). Efectivamente, escogemos cp (x)
de modo que q>' (x0 ) == 1 y que 1 <p (x) 1 < 11 en el segmento (a, b }.
Consideremos la funcin y (x) = Yo (x)
<p (x), donde !Jo (x) E
E la, b). Entonces tendremos (xo) = !I (Xo)
1. Es obvio que
p (y (x), Yo (x)} < t, o sea, que las curvas y (x) e !lo (x) son cercanas en
el sentido de proximidad de orden nulo. Al mismo tiempo J (y (x)l - J [y 0 (x)) = 1, es decir, Jos valores de la funcional no son prximos
por cercanos que sean, en el se.nlido de proximidad de orden nula, los
argumentos y (x) e Yo (x).
Hablando con ms precisin, existe un e
O (por ejemplo, cua l
quier a < 1) tal que para cualquier 'l >O existirn funciones y (x)
para las cuales
Po [y (x), Yo (x)J < 11 y, sin embargo, 1 J [y (x)} - J IYo (x}) 1 ~ s.
Esto significa precisamente que la funcional J (y (x)) es discontinua
en el sentido de proximidad de orden nulo.
Demostremos que esta funcional es continua en el sentido de
proximidad de primer orde t.
Tomemos un e > O cualquiera. Tendremos

e,

y'

>

1J

(y (x)] -

J IYo (x)) = l 11' (xo) -y~ (Xo) J.


= e tendremos

Queda <:laro que lomando 11

1 J [u (x)J -

J IYo (x)J 1

<e

siempre que Ps [y (x), y0 (x) J < 'l que es lo que se querla demostrar.
Este ejemplo permite ver que de la continuidad de la funcional en el
sentido de proximidad de n-simo orden no implica, hablando en tr
minos generales, su continuidad en el sentido de proximidad de orden
Inferior.
EJEMPLO 11. Consideremos 1a funcional
R

J (g(x)] =

''J.dx

o
definida en el espacio C 1 {0, n). Demostremos que es discontinua en la
funcin Yo (x) == O en el sentido de proximidad de orden nulo.
.
sennx
En efecto, sea !lo (z);;: O en [O, n] y sea !Ir> (xJ
Enton-

= -n-.

l
ces, Po (Yo (.x), !In (x}) =- de modo que Po ~ O cuando n ~ oo.

3-()1387

34

C AP. 11. E X TREMO DE FU NCIO NALES

Por ofro lado, la dHerencia


11

J lYn (,")\ -J (Yo (x)]=

J\

cos2nx d

x=2

o
no depende de n. Es decir, J Yn (x)) no tiende hacia J [g 0 (x)) = O
cuando n ..... oo y, por consigui ente, nuestra luncional es discontinua
en l a funcin Yo (x) = O en el senti do de proximidad de orden nulo.
Proponemos al lector demostrar que esta runcional es continua
en la funci n y 0 (x)
O en el sentido efe proximidad de primer orden..

Analizar la continuidad de las funcionales sguientes


36. J ly (x)J =y (xo). donde y (x) E e [a, b] y Xo E [a, b],
en el sentido de proximidad de orden nulo
37. J ly (.x)f = mx 1y (x) I, donde y (x) son funciones
continuas en el segmento la, b], en e l sentido de proximidad
de orden nulo.
38.
tl si y (x) toma al menos un valor negatvo,

si y (x) ==O,

J (y (x)] =

1 si y (x)~O e y(x) ~O,

en el sentido de proximidad de orden nulo


1

39. J (y (x)] = ) 1y'1 dx, donde las funciones y (x)


o
tienen primera derivada continua en el segmento [O, 11:
a) en el sentido de proximidad de orden nulo;
b) en el sentido de proximidad de primer orden.
11

40. J lY (x)J = ) V l + y' 2 dx en la funcin Yo (x)


o
donde y (x) E C1 [O, 11]:

== O,
O

a) en el sentido de proximidad de orden nulo;


b) en el sentido de proximidad de primer orden
re

4 t. J fy (x)l =

J(1 + 2y'
o

2)

dx en la funcin Yo (x) e= O,

FUNCIONAL.

VARIACIN

DE

UNA

FUNCIONAL

35

donde y (x) E Ct. (0, n), en el sentido de proximidad de


primer orden.
EJEMPLO

12. Demostrar que la funcional


l

J[g(x}]=

JxVl+y2dx
o

definida en el conjunto de las funciones y (x) E C [O, l] es continua


en la funcin y 0 (x) = x2 en el sentido de proximidad de orden nulo.
SOLUCJON. Pongamos y (x} = x2
CU] (x), donde T) (x) E e (O, l]
y a. es tan pequeo como se quiera, tenemos

J [y (x)]

=1 [x2 + art (x)J =

Jx3 Vl + (x2+ar)2dx =
o
1

Jo

x3

V 1+x'+ 2ax2r +a.2f12 dx.

Pasando al limite cuando o:-+ O, obtenemos de esta igualdad


t

lfm Jig(x)J=
a-o.O

xV l+x'dx=l[x2J

lo que equivale a la continuidad de la funcional en la funcin

Yo (X)=

x.

Sea M un espacio lineal normado formado por las


funciones y (x).
La funcional L [y (x)) definida en el espacio M se denomina
lineal si satisface las condiciones
DE.PlNICiN.

1)

l {cy (x)}

el (y (x)J,

donde e es una constante cualquiera y


2)

L IY1 (x)J

+ y" (x)J =

L l.!lt (x)J

+ L (y

donde Y1 (x) E M e !Jz (x) E M.


Por ejemplo, la funcional
b

L [y(x))=

(U' + y)dx

definida en el espacio C1 fa, bJ es, obviamente, lineal.

(x)J,

AP. lf . EXTR EMO DE FUNCIONALES

Existe otra definicin de funcional lineal:


La funcional L [y (x) 1se denomina lineal si 1) es continua y 2) satis
lace la condicin

L YJ (x)

+ Yz {.%)) =

L (Y1 (x)]

cualquiera que sean 1 (x) E M e !/t (x)

+ L [y, (x)]

E M.

42. Demostrar la equivalencia de las dos definiciones de


funciona l lineal.
43. Demostrar que la funcional L [y (x) J = y (x0 ) es lineal.
44. Sea L [(y (x)J una funcion al lineal. Demostrar que si
, l (y(x)J
.
r 1la razon
llY(x)JI-+ O cuando 11 y (x) 11 ~O, es L y (x) =O.

3. Variacin de una funcional . Sea J [y (x)J una funcional defin


da en el conjunto M de funciones y (x). La magnitud

ll J

(6y

61 (y (x)) = J [y (x)
By) - J fy (x)]
y(x) - y (x), donde 1J (x) E M e (x) E M)

se denomina incremento de la funcional J {y (x)) correspondiente 111

incremento 6y del argumento.


EJEMPLO u . Hallar el incremento de la funcional
t

J [y {x)) =:

Jyy' dx
o

definida en el espacio C1 [a, bj si y (x)'


SOLUCIN. Tenemos
M=J(x2)-J[x]=

x e !h (x) = x'.

Jx2'2xdx- Jxl dx=) (2xS -x) dx=O.

45. H allar el incremento de la funcional del ejemplo 13


siendo y (x) ~ e Y1 (x) = l.
DEFJNIClN. Si el incremento de l a funcional J [y (x)]

l:!.J = J [y (x)

+ 6yJ -

J [y (x}]

se puede representar en la forma


M

L [y (x), c5y)

+ ~ (y {x), 6y) 11 By 11.

donde L {y (x), 6yl es una funcional lineal respecto a 6y y~ (y (x), 6y)- O cuando U6y U - O, entonces la parte del incremento lineal res
pecto a 6y, o sea, L [y (x), 6yJ, se llama fJariacin de la funcional y se

FUNCIONA L.

VAR IACJON

DE

UNA

FUNCIONAL

37

representa por 6J. Se dice en este caso que la funcional J [y (x)) es


di ferenciabfe en el punto y (x).

46. Demustrese que la variacin &J de la funcional

J [y (x)l se determina unvocamente (si es que existe).


EJEMPLO 14. Demustrese que la funcional
b

= )

J [y (x)]

!I d:z

definida en el espacio C la. b] es difercnciable en todo punto !I (x)


de este espacio.
SOLUCION. Tenemos

6.J=J

lv <x>+oyJ - /

fy(.rlJ=
b

J<v+~.v)

d:r -

yd.x=

Jav

ci.r.

Es decir, !11

Jf>y

dx. Pero sta es una funcional lineal res pecto


a
a 6y. Todo el incremento se ha reducido en nuestro caso a una funcional
lineal respecto a 6y. La funcional consderada es diferenciable en todo
I>

punto y (x) y su variacin es 6J

6y dx.

47. Demostrar que toda funcional J ly (x)l lineal continua


es siempre d iferenciable.
EJEMPI.O 1s. Demostrar

que la funcional
b

J tu {x)l

=)

y'l.dx

definida en el espacio C [a, bJ es dilerenciable en todo punto y (x).


SOLUCIN. Tenemos

61=

J(y+au)dx-)

udx=

J2y0ydx+ J{Oy)idx.

(3)

En el (lltlmo miembro de (3) la primera integral representa la funclo


nal lineal respecto a 6y cual quiera que sea la funcin fija y (x). Estl

38

CAP. 11. EX TR EMO DE FUNCIONALES

memos la segunda integral de este miembro. Tenemos

I>

(y)'l. dx =

J g 2
1

~(

mx

Q~XEib

6y

dx ~

Jaul)z dx=(b-a)(l611 IF=((b-a)ll6yllJlf6y1J.

J
a

O, la magnitud
(b - a) 11 6g 11 -i- O.
Es decir, hemos logrado representar el incremento !lJ de la funcional
como la suma de L !y {x), oyl y de una magnitud infinitslma de
segundo orden con respecto a 11 6y U. Segun nuestra definicin, la
funcional considera da es dferenciable en el punto g (x) y su variacin es

Si

11

11 -

61=2) y(>ydx.
a
t

48. En la funcional J (y (x)J

y 8y

Jy'l.dx

o
axz.; comparar 8J y l:!i.J para a

tomar y = 2x

1; - 0, l y 0,01.
.

x!/dx tomar y = tr y
o
y = ax; comparar 6/ y 6J para a = 1; 0, 1 y 0,0 l.
50. Analizar si son o no diferenciables las funcionales
siguientes:
1) J [y (x)l = y (a) en el espacio e la, bl.
2) J ly (x)J =y (a) en el espac io c. ra, b}.
3) J [y (x)l = V 1 + y''I. (a) en el espacio C1 [a, bl.
4) J {y (x}J = 1 y (a) 1 en el espacio e [a, bJ.
51. Demostrar que la funcional J [y (x)J es diferenciable
si to es J [y (x)]. Hallar la variacin de / 2 [(y (x)J.
52. Sea F (x, y) una funcin continua de sus argumentos
con derivadas parciales continuas hasta de segundo orden
inclusive en el recinto a~ x ~ b, - oo <Y< + oo. Demos
trar que la [uncional
49. En la funcional J [y (x)J

J y(x)l=

JF(x, y) dx
o

3.

FU NCIO NAL

V ARI ACIN

DE

UNA

F UNCIO N AL

39

definida en el espacio C (a, lr] es d iferenci able y que su variacin es


b

<'J =

r iJF ~,y

y) o ydx.

16 . Consideramos la funciona!

EJEMPLO

J (y (x)J =

F (x, y, y') dx

definida en el espacio C1 [a, b) de fu nciones y (x) que son continuas


en el segmento [a, b) y (Ue tienen en l derivada continua de primer
orden . La fu ncin F (.~. y, y') es conti nua respecto a todos sus argu
mentos y tiene derivadas parciales continuas hasta de segundo orden
inclusive en el recinto
a~

-oo

b,

< y< +oo, -oo

< y'

< +oo.

Determinamos el incremento AJ de la funcion al correspondiente al


Incremento y del argumento siendo y E C1 ta, bl. Tenemos
b

AJ (y (x))=

5(F(x, y+6y, y'+y'}-F(x, y, y'))dx.

(4)

e
Segn la frmula de Taylor

f(x, g+y, t/+/)- F(x, y, y')='


=

~=

oy +

~:, 6 y' -1- R (x, y, y', y, tly'),

(5)

donde R (x, y, y'. 6y , y') es el trmino complementario de la frmul a


de Taylor. Introduciendo (5) en (4), ob tenemos
b

liJ {!! (x)]

= J( ~~ hu+;:, o/) dx+

JR

(x,

y, y', ~!l. hy') dx .

(6)

El primer sumando en el segundo miembro de {6) es lineal respeclo a


6yly y'. Suponft.'amos que las segundas deri vadas parciales de la
fundn F1.(x, y, y ) respecto a y e y' no pasan, en valor absoluto, de una
constante M >O en un recinto acotado respecto a y e y'. Tendremos
entonces
b

J R (x, y, y', 6y , 6y') / dx < 2M J/J y 1/ dx = ZM (b- a) 11 y /1


2

40

CAP. 11 . EXT'Rl:MO DE FUNCJONALES

donde 1\ 8y 1\

= i;i;t

a.~x.~b

(1 oy 1. 1 6y' 1). Por consiguiente, el segundo

su mando del segundo miembro de (6) es una infinitslma de segundo


orden respecto a 11 Oy 11- Es decir, en v irtud de la definicin, \a funcional J [y (x)J es dlferenciable en el espacio C1 (a, b) y su variacin es
b

OJ =

J(~~ Oy+ ~:, Oy' )

dx

(7)

o
EJE'-'PLO

17 . H ali ar la variacin de la funcional


1

J (y (x)] =

J (y'ev+xy

2)

dx.

-t

SOLUCION La funcin F {x, y, y')= y'e11


xy2 es, evidentemen te,
continua respecto a todas las derivables x, y e y' en conjunto y sus
derivadas parciales de cualquier orden respecto a y e y' son acotados
en cual quier recinto acotado de variacin de y e y' . Por esto, la funcional considerada es diferenciable en C1 (-1 , 1) y, segn la frmula (7),
su v ariacin es

IH = ) ((y' ell + 2xy) Oy + e116y' 1dx.


-t

53. En la funcional
J [y (x}J =

j' (y' y + xy'

2)

dx

k(x-'l)

tomar y = In x y f>y = --;=:--- ; comparar /J.J y BJ para


k= 1; 0,1 y 0,01.
54. En la funcional
1

J [y (x)] =
tomar y = x?. y 6y = kx 8 ;
0, 1 y 0,01.

J(x g'
2

!f) dx

o
comparar J y fJJ para k = 1;

55. En la funcional J [y (.t) I =

5yi sen x dx tomr y =

=sen x y fJy = k cos x; comparar J y 6J para k = -1;


0,3' y 0,03.

FUNCIONAL.

'3

VAR IACION

DE

UNA

PUNCIONAL

41

parciale~ de segundo orden de la fun


zm+1 ) respecto a todos los argumentos
recinto a =::;;;; x =::;;;; b y - oo
Z1t
oo

56. Las derivadas

cin F (x, z1 , z2 ,

... ,

son continuas en el
< <
(k = 1, 2, .. ., m + 1). Demostrar que la funcional

J l!I (x)] =

JF (x, y, t', .. , !/"'') dx


Q

es diferenciable en el espacio Cm la, b] y que su variacin es


b

6J =

J[:: 6y + ::. 6y' + ... + ;,:>oym> Jdx.


0

<J

4". Segunda definicin de la variacin de: una funciona!. Se llama

variacin de la funclonal J [y (x)] en el punto y = y (x) el valor que


toma en a= O la derivada de la funcional J (y (x)
ciayJ (considera
da en tanto que funcin de ci) respecto al parmetro :

(J

6J = <Jtz J lY (x)+a6yJ 111<-o.


Si existe la variacin de la funcional en tanto que parte principal
lineal de su incremento (o sea, si existe la variacin en el sentido de
la primera definicin), tambin existe In variacin en tanto que valor
en a = O de. la derivada respecto al parmetro ci y ambas variaciones
coinciden.
EJEMPLO 18. Empleando Ja segunda definicin, ballar la variacin
de la funciona\
b

J [g (x)J =

Ji'- dx.

ll

SOLUCICN. La variacin de esta lunconal en el sentido de la pri


mera definicin es
b

fJJ = 2

J11 6y

dx

(vase el ejemplo 15). Determinemos la variacin de la funcional


J [y (x)) basndonos en la segunda dellnicin . Tenemos
b

J(v(x)+a.6y)=

J
o

(y+a.By)2dx.

42

CAP. JI. EXTR EMO DE FUNCIONALES

Por eso,

a~

J fy(x)-t-croy)=2) (y+ay)gdx
o

de modo que

J =

0~

J {y (x)

+ a~y) la- o= 2

yy dx,

Las variaciones de la funcional en el sen tido de la prime.ra y de


la segunda definiciones coinciden.

Para las funcionales que siguen hallar, en los espacios


correspondientes, sus variaciones en el sentido de la segunda
definicin .
b

57. J [y (x)l =

J (x+ y) dx.

58. Jty(x)]=

J(y -y' )dx.

59. J (y (x)] = y2 (O)+

j (xy+ y'

2)

dx.

o
lt

60. J [y (x)l = ) y' sen ydx.


o
b

6t. J[!/11 !Jz, ., Ynl= \ F(x, Y1 Yz, !1111


"a

y;, !J~,
.. , y~) dx,

donde F es una funcin continua de sus argumentos y sus


derivadas parciales respecto a todos los argumentos son
cont inuas en un recinto acotado O de variacin de los mismos.
OBSERVAC!N. La segunda definicin de la variacin de una fun
cional es en cierto sentido ms amplia que la primera pues existen
lunclonales que tienen variacin en el sentido de la segunda definicin
aun cuando no se pueda despejar la parte prindpal lineal en el ncre
mento de las mismas. Para explicarlQ rec1mlremos a las fu nciones;

3.

FU NCIONA L.

VA RIACIN

DE

UNA

FUNCIONAL

43

en este caso nuestra afirmacin equivale a que Ja existencia de las


derivadas en cualquier direccin no basta para la existencia de la dife
renclal de la funcin.
Sea
xy
f (x, Y)

donde p y cp son las coordenadas polares del punto (x, y). Las derivadas
parciales

:~ y !; ex isten en todo punto

son iguales a cero en el origen

de coordenadas; s in embargo, no existe la diferencial df en el origen


de coordenadas. Efectivamente, supongamos que df existe. En este
caso el gradiente de la funcin f seria igual a cero en el origen de coor

denadas y, por eso, Ja derivada

df ~ O) en cualquier direccin tambin

seria igual a cero. Pero es fcil persuadirse de que


df (O, O)
1
dl
= 2 sen 2 cp
lo que, en general, es diferente de cero. Agul cp es el ngulo entre el
vector l y el eje Ox.

5. Segunda variacin de una funcional. Una funcional J [x, y)


dependiente de los elementos x e y (q ue pertenecen ambos a un espacio
Ji rieaJ) se denomina bilineal S es Una funci onal lineal en
para X
fij o y una func ional lineal en x para y fijo. O sea, la funciona J [x, y)
es bilineal si

J (a1xt
Gt2JCs, y)
l (x, ~1Y1
P2flal

a 1 J [X, y]
j}/ [x, y]

Poniendo en la funcional bilineal y

+ a,) [x2. y],


+ ~2J [x, Y2l

x obtene mos la expres in

J {x, x] llamada funcional cua dr tica.

Toda funcional bllineal definida en un espacio de dimensin


finita se denomina forma bilineal.
Una Suncional cuadrtica J (x, x) se denomina defini da positiva
si J [x, x] > O cualquiera que sea el elemen to no nulo x.
Por ejemplo ,
1) la expresin

,,

Jfx, y]=

A{l)x (l )g(i)dt,

(1

donde A (/) es una funcin continua fija, representa una funcional


b

bilineal en el espacio C la, b] mientras que la expresin

A (t)

x (t)

dt

(l

representa, en este mismo espacio, una funcional cuadrtica que resu lta
definida positiva si A (t)
O para todo t E(a, b);

>

CAP. ll, EXTREMO DE FU NCfONALES

2) l ' expresin
b

J (x, x) =

(A (/) x2 (/} +B (t) x (f) x' (/)+e (t) x'2 (t)] dt

es un ejemplo de una funcional cuadrtica definida para todas l as


funciones pertenecientes al espacio C1 [a, bJ;
3) la expresin
,,

J [x,

Yl = } ) K (s,

t) x (s) y (t) ds dt,

a a

donde K (s,t) es una funci n fija de dos variables, representa una tuncional bilineal en e la. b).
DEFINICION. Sea J fy (x)) una funcional definida en un espado
lineal normado. Diremos que la funcional J [y (x)l tiene segunda
variacin si su incremento 6J = J (y (x) + 5y] - J (y (x)) puede
ser representado en la forma
1
t..J =La [~Yl+2L,.i~YI+~11~y11 2

donde L 1 (6yJ es una funcional lineal, L 2 [6y] es una funcional cuadr


tica y ~ -+ O cuando \1 6y \1 -+ O.
La funcional cuadrtica L 2 [8y) se denomina segunda variacwn
{o segunda diferencial) de la uncional J [y (x)] y se designa por 6'-J.
La segun da va riacin de una funcional se determina unlvocamente
(si es que existe).
EJEMPLO 19. Hallar Ja segunda variacin de la funciona l
1

J IY (x))

definida en el espacio C1 [O,


SOLUCIN Tenemos

6J = J (y (x)

=~

11 de

(xy2+y') dx
las funciones y (x).

+ 6yJ- J [y (x}] -=-

[x (11+6g)2+(.y' +6g')'-xy2-y'3J dx=

o
1

=) [2xy6y+x (6y)2+3g'20/ +3y' (6g')2+(<~g')3Jd.t=


o

J (2xy6y+3y'26y'}dx+ J(.x{6y)2+3y'(6y')2Jdx+) (y')Sd~.


t

'

(8)

FUNCIONAL-

VARIACIN

DE

UNA

FUNCIONAL

45

Fijemos y (x). El primer sumando del ltimo miembro de (8) ser


entonces una funcional lineal respecto a 6y; el segundo sumando de
este miembro ser una funcional cuadrtica; finalmente, el ltimo.
tercer, sumando de este miembro puede ser estimado asi
1

Jo

(6y')Sdx 1~(mh l 6y' 1)2

Jo

J 6y' 1dx

~(

Jo l

6y' 1 dx) ll 6y 112

(la norma se toma en el sent ido del espacio C 1 (0, 1)), o sea, podr ser
representado en la forma ~ 11 (jy 1111 donde ~ - o cuando 11 &y n - o.
Por definicin, para nuestra funcional existe la segunda variacin
62 J igual a
1

6?.J=2

(x(liy)2 +Sy'(6y')2)dx.

62.
tica.
63.
siendo
64.

Ha!Jar la segunda variacin de una funcional cuadrHallar la segunda variacin de la funcional

eF(11i

F (y) una funcin dos veces diferenciable.

Demostrar que las funcionales de t ipo


b

J [y(x)] =

JF (x, y, y') dx
o

definidas en el espacio C1 [a, b) son dos veces diferenciables


si la funcin integrando F tiene derivadas continuas hasta
de tercer orden inclusive. Hallar la frmula para la segunda
variacin.
Consideremos la funcin CD (a.) = J [y (x) + aOy]. La segunda
variacin 6't.J de la funcional J (y (x)] se define tambin mediante el
valor de la segunda derivada de la funcin <D (a.) en el punto a.= O:
a2<D (a.) 1
da.2
ti-O.

Para las funcion ales de tipo integral (que predominarn en nuestras


consideraciones) ambas deriniciones coinciden.

Hallar la segunda variacin.


b

65. J [y (x)J = ~ F (x, y, y', ... , y1m>) dx.


el

46

CAP. 11. l!XTRP.M.0 DE FUNCIONALES

66. J (z (x, y) 1=

Jf P (x. y, z,

Zx,

zu) dx dy.

G
I>

67. J

lY Y21

!/nl

=) F(x, Yt y~. . . .' !lr11 y;, y~


a

... , y~) dx.


6. Extrtmo de una lunc:lonal. Condicin necesar ia de extremo.
Diremos que la funci onal J (y (x)] alcanza su mzlmo en la curva
y = y 0 (x) s i los valores que toma la funci o nal J [y (x)l en cualesquiera
curvas prximas a y = Yo (x) no son mayores que J [y 0 (x)). o sea , si

J [y (x)} -

J (Yo (x)l ~ O.

Si 6.J ~ O y .J = O slo para y (x) = y 0 (x), di remos que se


alcanza mximo estricto en la curva y = y 0 (x).
Anlogamente se define la curva y = y 0 (x) en l a que se alcanza
un mltilmo. En este caso se tiene /;.J ;;;i, O para todas las c urvas prxi
mas a l a curva y = Yo (x).
EJEMPLO 20. Demostrar que la funcional
1

J (y (x)I =

J(xz+

y2) dx

o
alcanza mini mo estricto en la curva y (x) e O.
SOLUCION. Cualquiera que sea la funcin y (x}. continua en [O, I],
tenemos
1

= J (y (x)] -

J [O]=

(x2 + y2) dx -

xz dx =

y2 dx :;i.. O;

adems, el s igno de Igualdad se da slo para y {x) = O.


EX~EMOS FUERTE y D2BIL. Diremos que la fun cional J (y (x)J
alcanza su mdxlmo relativo fuerte en l a c urva 11 = y, (x) si
J ly (x)] ~ J i!le (x)}
para todas las c urvas admis ibles y = y (x) pertenecientes a una e-vecindad de orden nulo de la curva y = y 0 (x). Anlogamen te se define el
minimo relativo fuerte de una funcion al.
Diremos que la funcional J [y (.x)} alcanza su mdx imo relativo dbil
en la curva y = Yo (x) s i
J [y (x)) ~ J [Yo (x)l

para todas l as curvas admisibles y = y (x) pertenecientes a una e-ve


cindad de primer orden de la curva y = y 0 (x). Anlogamente se define
el nnimo relativo dib!l ck una funcional.

F UNCIONAL.

3.

VARIACIN

DE

l iNA

47

FUNCIONAL

Los mximos y mnimos (fuer tes y dl>iles) de la funcional


J (y (x)l se denominan extremos relativos.
Todo extremo fuerte es al mismo tiempo ext remo dbi l pero no
viceversa.
El extremo de la funcional J (y (x)l referente a la totalidad de las
funciones en las que est deinida la funcional se denomina extremo
absoluto.

Todo extremo absoluto es al mismo tiempo ex tremo re lativo fuerte


y dbil pero no todo extremo relativo ser extremo absoluto.
EJF.MPLO 21. Cons i cJeremos 1a funciona 1
fl

J (y (x)I

=)

yi (1 - y'2) dx

o
en el espacio de funciones y {x) E C 1 [O, n) que satisfacen la condicin
y (O) = y (n) = O. En el segmento (O, n} del eje Ox hay minimo dbil
de J. En electo, tenemos J = O si y = O; por otra parte, para las curvas
perte.necientes a una e-vetindad d e primer orden .de este segmento,
donde e es cual qu ier nmero positivo me nor que l , se tiene 1 y' 1
J
de modo que el integrando es positivo para y =FO y, por consiguiente,
la funcional se anula s lo si y = O. Es decir, la lundonal alcanza
mni mo dbil en la curva y= O.
Mnimo fuerte no hay. Basta tomar

<

y (x)

-V sen

nr

En este caso
J ly(x)) =

Jse112 11x(l-ncosZ nx) d.x=


o

= _!_
n.

sen2nxdx - .!..

rsen22f1Xdx ,.,.~-~

2n

8 '

o sea, J <O en estas curvas s i n es suficlcnlemente grande. Por otra


arte, siendo 11 suficientemente grande, todas estas cur vas se encuenran en una vecindad tan pequea como se quiera de orden nulo de la
curva y = O. P or consiguiente, no se alcanza mlnimo fuerte en y = O.
EJEMPLO 22. (Weierstrass). Consideremos la funcion al

J[y{x))=

x '-y'2dx;

y (-1 ) = - I ,

y ! l) =I.

- 1

En el segmento [ - 1, 1) tenemos J [y (x)] ~ O y, adems, J [y (x)} = O


slo si y ' (x) = O, o sea, si y (x) = C = const. La fu nci n y (x) = e
pertenece a \a clase C 1 l - 1, 1] de las funciones que tienen primera derivada continua en t>I segm('nto (-1 , I], pero 110 satisface l as condiciones
de frontera da das. Por consiguien te, J [y (x)] > O parH todas l as
(unciones y (x) E C1 [-l, 1l que satisfacen las condiciones y ( -1)
-1

48

CAP.

J) .

oe

EXT~EMO

PUNCIONAt. BS

e !I (J) = J. En o tras palabras, la funcional tlene cota inferior pero


sta no se alcanza en las curvas y (x) E C1 (-1, 11. Efectivamente,
consideremos la fa milia monoparamtrica de curvas
X

Ya. (x) =

arctga.
l

'

a. > O.

arctgn

Todas ellas satisfacen las condiciones de frontera: Ya (-1) = -1 e


Ya. (1) = 1. Pasando al lmite para a. - O, obtenemos la Funcin
-1, s i -1 ~x <O,
y(x) =
O, si
x = O,
{
+ l , si 0 < X 4' l.
es decir, (x) = sg x (flg. 3). Esta funcin pertenece a la clase de
funciones diferenciables a trozos en el segmento (-1, 1).

Y""YoJx)

-t

Flg. 3
Tenemos
t

J (!la. (X)}=

J
-t

cr.x2dx

(a.2

+.x2) arc tg2 -a1=


1

2a.

i
arctg2 -

a:

J0

1 ( 1-a. arctg
arctg2 a.

~) .

3.

PUNClO NAL.

VAR IACIN

DE

UNA

49

F UN CION AL

Queda cl aro que J {y~ (x)J -+ O cuando a. -+ O. En la funcin l mite


y(x), que satisface tas condiciones de frontera!/'(-!)= -1 e y (1 ) :c..,,
= 1, el valor de l a fun cional J [y (x)/ es igual a cero: J (x)}
O.
Por consiguiente, la funcional J IY (x)) alcanza su valor mni mo
en la cur \'a (x) = sg x que pertenece. a la clase de funciones dieren
d ables a trozos en el segmento (-1, IJ pero no pertenece a la cl ase
C1 (-1, l].
TEO~EMA (condicin necesar ia de extremo de la funcional). Si la
funcional diferenciable J [y (x)l alcanza su valor extremo en la curva
y= y0 (x), donde y 0 (x) es un punto interior del campo de defnic in
de la funcional, entonces en y
Yo (x) se tiene

f>J [Yo (x)] = O.

Las funciones para las cu a les 6J


estacionarias.

(9)

O se deno minarn f unciones

Hallar las ecuaciones funcionales para las funciones estacionarias de las funcionales que siguen, empleando la condicin necesaria de extremo (9) y los lemas fun damentales del
Clculo variacional.
, b

68. J [qi (s)J =

JJK (s, t)

q> (s) cp (t) dsdt

a a
b

+ Jcp (s)ds-2 J 'fl(s)/ (s)ds,


2

donde K (s, t) es una funcin continua s imtrica de s y t en


a< s <
el recinto D {a .::;;. t -~ b , f (s) es una fun cin continua
en fa, b] y cp (s) es el argumento funcional continuo incgnito.

b}

+oo

69.

J [q> (x) J=

J[p

(x) q>' 2 (x)

+ 2qi (x + l) X

-oo

X q> (x- 1) - cp2 (x)-2cp (x)

f (x)J dx,

donde el argumento funcional q> (x); es contin uo y tiene derivadas continuas a trozos en todo el intervalo - oo < x <
< + oo, p (.x) tiene derivada continua y f (x) es contin ua.
70. J [ cp (x)J =

4-01387

[p ( x) q>' 2 (x)

+ q (x) q2 (x) - 2cp (x) f (x) 1dx,

50

CAP, 11.

EXT~fMO

DEiPUNCIONALES

donde p (x) tiene derivada continua, q (x) y f (x) son continuas y el argumento funcional q> (x) tiene dos derivadas
continuas.
4 Problema elemental del Clculo variacional.
Ecuacin de Euler
Supongamos que la funcin F (x, y, y') tiene derivadas parciales
continuas hasta de segundo orden inclusive respecto a todos sus argu
mentos.
El problema elemental del Clculo variacional es el siguiente:
entre todas las fu nciones y (x) que tienen derivada continua y que
satisfacen las condiciones de frontera
1J (a)

11 (b)

(1)

hallar la funcin que ofrece extremo dbil a la runcional


b

J 111 (x)) =

F (x, y, 11') dx.

(2)

En otras palabras, el problema elemental del Clculo variacional


consiste en hallar el extremo dbil de la funcional de tipo (2) en el
conjunto de todas las curvas suaves que unen dos pun tos fijos P1 (a, A)
y P,. (b, B).
TEOREMA 1. Condicin necesaria 1 ) para que Ja fu ncional (2), defi
nida en el conurito de todas lat. funcioties y = (x) que tienen derivada
continua y que satisfacen las condiciones de frontera (1), alcance su valor
extremo en la f unctn y (x) es que e.sfa funcin verifique la ecuacin
de Euler
(3)

Las curvas integrales de la ecuacin de Euler se denominan extre-

males (o curvas de Lagrnnge).

En forma desarrollada la ecuacin de Euler da


v" (x} F11 'v' -1- y' (x) Fyy
F:r.11 , - Fy = O (F11 , 11 , .::6:0)
y representa una ecuac;n <lHerencial de segundo orden de mo do que
su solucin ' general comprender dos constan tes arbi tra rias cuyos

valores se de term inan. hablando e n trminos generales, de las condi


ciones de frontera (1) .
La funcional (2) puede a lcanzar extremo slo en las cxtrcmales que
satisfacen las condic iones (l ).
1) Esta cond icin es necesaria p ara el extremo dbil. Como quiera
que todo extremo fuerte P.S al mismo tiempo un ex tremo db i l, cualqu ier condicin necesaria para el extremo dbil tarnbin ser nt:ccsarla
p~ra el extremo fue r te.

s 4.

PROBLEMA ELEMENTAL

51

El problema de contorno
Fv-d F11 . = 0 }
dx
y(a) =A , y(bl= B

no siempre tiene solucin y si la solucin existe, puede no ser nica.


EJEMPLO 1. 1.En qu curvas puede alcanzar su ex tremo la funcional
2

J[y{x))=

J(y'2 -2xy)dx;

y(l}=O,

y{2)

~ - 1?

Aqu tenemos F (x, y, y')

SOLUCION.

= y"J. -

2xy de modo que

la ecuacin de Euler da y+ x = O. Su solucin general es


x3
JJ = - 6 +C1x+C2.

Utilizando las condiciones de frontera, obtenemos para C1 y C'l el


seguiente sistema de ecuaciones lineales:

C1+C2=

~, }

2c,+c2 = 6 .

De aqu resulta C1 =
y C2 =0. Por consiguienie, el extremo puede
alcanzarse slo en la curva
X

Y=5 {I -x2).
EJEMPLO 2.

Ha ll ar las extremales de la funcional


3

J [y (x)) =

(3x - y) .1/ cix

que satisfagan las condiciones de fron tera y (1 )


SOLUCJN.

1 e y (3)

La ecuacin de Euler es 3x - 2y= O, de donde y

~.

= ; x.

La extrema! IJ = ~ x no satisface la condkin y (I} = 1 y, por eso,


nuestro problema variacional no tiene solucin.
EJEMPLO 3. Hallar las extremales de la funcional
2it

Jly(x)]=

(Y''-y2)dx

que satisfagan las condiciones de front era y (O)

l e IJ (2n}

l.

52

CAP. 11 . EXTREMO DE FUNCIONALES

sOLUClON. La ecuacin de Euler tiene la forma


solucin general es
y = C1 cos x
C9 sen x.
Utilizando las condiciones de frontera, encontramos

y+

g = O; su

COS X

+ C sen X,

donde Ces una constante arbitraria. Es decir, el problema variacional


considerado tiene un conjunto tnlinito de soluciones.
Hallar las extrernales de las funcionales siguien-tes.

71. J [g(x)] = I (12xy-y'2)dx;

y(-1) = 1, y(O) = O.

-1
2

72. J{y(x)l = ) (y' 2 + 2yy'+y2)dx; y(l)= 1, y(2)=0.


1
1

73. J[y(x)l= jYY(l + y'z)dx; y(O) = y(l)=y


2
o
1

74. J[y(x)l=) yy' 2 dx; y(O)= l, y( l)=:-14.

o
n

75. J(y(x)l=) (4ycosx +y' 2 -y2)dx;


o
y (O)= O,

y (n) =O.

76. J{y(x)=

J(y' -yi-y)e'l/rdx; y(O)=O,


2

y(l)=e-1

77 . J[y(x)] =

J (y' 2

2x.y)dx; y(- 1)= -1, y(l)=l.

-1

78. J (y (x)] =

J (y' -2xy) dx;


2

y ( -1) =O, y (0) = 2.

- 1

79. J [y (x)} =

e,.
j (xy' 2 + yy') dx;
1

y(I) =O, y (e)= J.

Pl OBL EMA ELEMENThL

53

La ecuacin de Euler (3) para la funcional (2) es una ecuacin


diferencial de segundo orden y, por eso. la solucin y = y (x) de 13
ecuacin de Euler debe tener segunda derivada y" (x). Sin embargo,
se dan c.asos en que 1a (uncin que o[rec.e el extremo a la funcional
b

J {y (x))

F (x, y, y') dx no tiene segunda derivada.

a
EJEMPLO -1. La fun cional
1

J ly\:c))=

J y2 (\ - y')2 dx
- 1

con las condiciones de frontera


y (- 1) = O e
y( \)= 1
alcanza s u valor m n imo, igual a cero, en la funcin

_ { O si x ~ O,

V (X) -

,
X SI

x> O.

Aun cuando la runcin u (x) no tiene segunda derivada, satis[ac.e la


ecuacin de Eulcr correspond iente.
Efectivamente, tenernos F (x, y, y') = y 2 (1 - y' )2 y, poniendo
y = v (x), obtenemos la ecuacin de Euler
2u(l - v' )2 +

!, (2u2 (t -

v')] = 0.

(4)

Pero, seg n la definicin de la fun cin u (x), en (-1, 1) tenemos

,, =

"
,, = O;
o sea, a pesar de que la ecuacin de Euler (4) es ormalme.ntc de segundo
P ,

-2u11 (1 -

u') = O y, por consiguiente, tambin dd F ,

orden y a pesar de que u (x) no ex is le, la ecuacin de Euler se convierte


en identidad al sustituir en ella v (x).
TEOREMA 2 . Sea y= y (x) solucin de la ecuacin c Euler
d

Fy-dXF 11. = 0.

Si la funcin F (x, y, y') tiene derivadas parciales continuas hasta de


segundo orden inc/usiue, entonces la funcin y = y (x) tene segunda
derivada continua en todos los puntes (x, y) para los cuales
Fyy [x, y (x), y' (x)] :6=0.
COROLARIO. La extremal y = y (x) puede tener puntos angulares
slo en aquellos puntos en los que Fyu = O.

Asi, en el ejemplo 4 tenemos que FY'Y' = 2y2 se anu la en los


puntos del eje Ox; la extrema} t iene punto. angular en x = O.
TEOREMA 3. (Berns ten). Supongamos que en la ecuacin

u = f (x, y, y')

54

C,\P . 11. E.'n.REMO DE FU NCIONALF. S

las funcones f, f11 y f , son continuas en lodo punto finito (x, y) para
cualquier valor fi nito de y' y supongamos que exisle una constante k > O
y unas fu nciones
a = a (x, y) > O y
~ = ~ (x , y) > O.
acotadas en cualquier porcin finita del plano, tales que
2
f y (x, y, y') > k
y
1 f (x, y, y') 1 ~ a..y'
~
(6)
Entonces, por dos cualesquiera puntos del plano (a , A) y (b, 8) de abscisas
distintas (a ;:f= b) pasa una curva integral y = rp (x) de la ecuacin (6),
y slo una.
EJEMPLO 5. Dcmostrnr que por dos cualesquiera puntos del plano
de a bscisas distintas pasa una extrema! nica de la funcional

J{ y(x\j "'""
sOLUCION.

Jc- 2llt( y'2 -

\)d,'(.

La ecuacin de Eulcr para l a fun cional considerada es


y# = 2y (1

+ y' 2)

de mo do que se puede a plicar el teorema 3. En efecto, tenemos en


este caso

(x, y, y') = 2y 11
Adems,

+ y' 2)

1 f (x, y, y') 1 = 1 2y (1
o sea, a = ~ = 2 1 y 1 -;;;, O.
EJEMPLO

fy = 2 (1

y' )

>2=

k.

+ y' ~ 1 ~ 2 1 y l y' 2 + 2 1 y 1,

s. Demostrar que no hay ex trema! de la fu ncional


J (y(x)J=

J (yi+ Vi -l y'2) dx

que pase por dos cualesquiera puntos del plano de abscisas distintas.
sOLUCION. La ecuacin de Euler tiene la forma
3

y" =2g{l + y'2)2


(7)
y el teore.ma 3 no se puede aplicar ya que no se cumple l a segunda de
las condiciones (6) (debido a que f (x, y, y') crece, res pecto a y' , ms
r pido que la segunda potencia de y') . Las condiciones del teorema 3
son de carcter suficiente. O sea, si estas condiciones no se cumplen, de
ello no se puede deducir que no hay extrema! que pase por dos puntos
cualesquiera de abscisas diferentes. Demostremos que por los puntos

A (O, O) y B { ; , 2) no pasa ninguna ex trema) de la funcional con


siderada-

Tomando en la ecuacin (7)


y' = P

dp

y = P dy ,

4.

PROBLEMA

ELEMENTAL

55

obtenemos
dp

p dy

.~

= 2y ( ( + p)Z-2 ,

o sea,

pdp

3
(1 +Pz)2

~2ydg.

I ntegrando, encontramos

----=y2-C , o sea,

0+P2

de modo que

(C - y 2}
dy

-;;=

Vi + y'2= 1

VI-(C-y2)2
C-y2

donde C es una constante real. Separando las variables en l a ltima


ecuacin e integrando del punto A al punto 8 , obtenemos
2

1
C-y 2
- = l
dy
2
. Vl-(C-yZ)2
.

(8}

Cualquiera que sea el n mero real C, el denomina dor del integrando de


(8) ser complejo en cierto inter valo (a, ~) e: (O, 2) de variacin de
la variable y . Por consiguiente, la Igualdad (8) es imposible. Esto quiere
decir que no se puede trazar extrema! alguna por los pun tos A (O, O)

y B(~.

2).

80. Demostrar que por dos cualesquiera pup tos del plano

pasa una y slo una extrema! de la funcional


J [ !I (x)l =
EJEMPLO 7.

J JI 1 + y2 + y'

dx .

Demostrar que toda ecuacin


y" (x} = f (x, y , y')

(9)

es ecuaci n de E ul er para cierta funcional


r
J fy (x) } =
F (x, y , y') d."<.

1) Cmo se determina la fun c in F (x, y, y ') a partir de la funcin

(x, y, y')?

2) Hallar tod<1s l as funcionales cuyas extremales son las rectas

y= C1x

+ C2.

SOL UCIN. Busq uemos la funcion al cu y a ecu acin de Euler

Fy - Eyx-F u'uY' - FvvY" =0

56

CAP. II. EXTR EMO DE FUNCIONALES

coinci da con la ecuacin (9). Es decir, debe cumplirse la identidad en

x, y e y'

F 11 -F11 ,x-F1111 y' -F11 , 11.f (x, y, y');;; O.


Derivando respecto a y', obtenemos
1

F11'11' x+ F111111Y + F111111f.+FY'11'f 11, =0.


Tomando u = F11 , 11 , , obtenemos para la funcin u una ecuacin en
deriva das parciales:

au
+ , <Ju +f ay'
ou + f 11
rJx Y ay

u=

(IO)

Por c om;i~uiente , la bsqueda de la funconal, o sea, de la funcin


F (x, y, y ), se red uce a la integracin de la ecuacin lineal en derivadas parc-iales (10) y a la cuadratura sucesiva.
Consideremos la segunda cuestin. En este caso la ecuacin de
Euler debe ser y" = O y para la funcin u se obtiene, de acuerdo con
(10), la ecuacin

~+ ,2!!;__0

"

( 11)

u au - .

Integremos esta ecuacin.


La ecuacin de las caracterlsticas tiene la forma
dx

dg

dy'

-1 =y;-=o-

Integrando este sistema, encontramos


y'= C1

y= C1x + Ch

de don de C 9 = y - xy'. Por eso, la solucin ge neral de la ecuacin


(11) es

u (x, y. y') = <I> (y' , y -

xy' ).

donde ID es una funcin ar bitraria diferenciable de sus arg umentos.


De aqu l
F(x, y, z) = a(x,

y)+z~(x,

:i

y) t

(z-t) <l> (t, y-tx)dt,

o
donde a (x, y) y ~ (x, y) son funciones arbitrarias de sus argumentos
que cumplen la relacin

= oxa~ .
O!J

Se pue de ver de la s olucin que existe una cantidad infinita de problemas variacionales que tienen l a ecuacin (9} co mo ecuacin de E uler.

PROBLEMA ELEMENTAL
CASOS ELEMENTAi.ES DE INTEGRACIN DE J.A ECUACIN DE EULER.

= F (x, y}.
En este caso la ecuacin de Euler tiene la forma

10. F no depende de y', o sea, F

F11 (x, y)= O.

(12)

La solucin de esta ecuacin finita (no diferencial) no contiene ele


mentos arbitrarios y, por eso, no satisface, hablando en trminos gene
rales, las condiciones de frontera y (a) = A e y (b) = B.
Slo en casos excepcionales, cuando la curva (12) pase por los
puntos de frontera (a, A) y (b, 8), existir una curva en la que podr
alcanzarse el ex tremo.
EJEMPLO s. Hallar las extremales de la funcional

2"
J (y (x)I

Jo

y (2x -y) rlx;

y (O) = O,

y(

~)= ~.

SOLUCJON. La ecuacin de Euler tiene la forma 2x-2y = O, o sea,


y= x. Puesto que las condiciones de frontera se satisfacen, la integral

. :n

y (2x -

y) dx puede alcanzar su extremo en la recta y

otras condiciones de frontera, por ejemplo, y (O) = O e y (


la extrema! y=

x. Para

~ )=

x no pasar por los puntos frontera (0, O) y ( ~ ,

1,

1)

de modo que el problema v ariacional con estas condiciones de frontera

no tendr solucin.
20. F depende de y' en forma lineal, o sea,
F (x, y, y')= M (x, y)+ N (x, y) y'.
En este caso la t>cuacin de Euler tiene la forma

aM

a -

aN

iJx = O.

Igual que en el caso 1, Ja ecuacin obtenida es finib:t y no diferencial.


En general, Ja curva determinada por Ja ecuacin ..,M - JaN = O
uy
X
no satisface las condiciones de frontera y, por consiguiente, el
problema variacional no tiene, como regla, solucin en la clase de
funciones continuas. Por otra parte, si en un recinto D del
.
oM - a;
iJN = o, 1a expres1on
.,
F (:<, y, y ' ) =
p 1ano x Oy se t 1ene
Ty
""' M (x, y) dx

+ N (x,

y) dy es una diferencial total exacta y la

CAP. 11 . EXTREMO DE FUNCIONA LES

58
funcional

JF

J (y (x)J=

(b, B)

(x, y, y') dx=

(M ttx+N dy)

(a, A)

no depende dt'l camino seguido en la integracin: el valor de la fundo


na( J (y (x)l es el mismo en todas las curvas admisibles. El problema
variaconal carece de sen ti do.
EJEMPLO 9. Analiiar el extremo de la funcion al
b

J( y(.x)J=

(y2+2xyy')dx;

g{a) = A,

y(b)=B.

Aqu F depende de y' en forma lineal. Tenemos


aN
oM _ oN =
oy = 2Y,
ax = 211
y
oy
ax - 0

SOLUC IN.

aA1

o sea, el in tegrando (y 2
2xyy') dx es una diferencial exacta. Por
consiguiente. la integral no depende del camino seguido en la integra
cin:
b

(ti, B J

[y (.x)l = J(y2 dx + 2x dy) = J d (xy2) = xy2 1::: = bB2 a

aA2

(11, A )

para cualquier curva de integracin y (x) q ue p ase por los puntos (a, A)
y (b, B). E l problema variacional carece de sentido.
3. F depende slo de y' . o sea, F = F (y').
La ecuacin de Euler ti ene la forma

Fy y!/ = O.
En este caso las ex tremales son t odas las rectas posibles

y= C1 x
C3 ,
don de C1 y C2 son constantes arbitrarias.
EJEMPLO 1o Hallar las ex tremales de la fu ncional
b

Jf y(x)} =

Vl+ y'3dx;
y (a) =A,
y (b)=B .
a
Esta fu ncional determina la longitud de la curva que une los puntos
(a. A) y (b, 8 ). Desde el punto de vista geomtrico, el problema con
siste en hallar la curva de longitud mnima que une dos puntos fij os.
SOLUCJN. La ecuacin de Eul er t iene la formar/'= O. Su solucin
general es

4 .

P ROBLEMA

E L E M ~ NTA L

59

La extrema\ que salis[ace las condiciones de fr~)ntera y (a)= A e


y (b) = B es , ob \'iamente, la recta q ue p<i:>a por los puntos (a, A)
y (b. 8) :

B-11

~b---a-(x- a) +A.

4. F no dE'pe nde dt" y, o sea, F "= F <x, y') .

En esk caso la ecuacin de f. ul er t>s

:X F!I' (x. y') :.... o. de donde

resulta

siendo C1 una const ante arbi traria.


La ecuaci n {13) es una ecuacin di ferr.ncial <le pr i mer orden.
lnh!grn dola, enc:ontra mos las ex trcmales d el prohlema.
EJEMPLO 11. E11trl las curvas que unt:n los p u ntos A (1, 3} y
8 (2, 5) hall ar la cu rv a en la que puede alcama r su c.:dr l'mo la luncional
2

J 111 (x))=~ ) y' (1

+ x ?.y') clx .

SOLU Ci N'. Puesto 4ue F no dl"pcndl' de y, la <cuacin de Eulcr

tiene la fo rma

: x F.y' (x ,

y')~ O,

1~x (l \ 2x2y' ) =0

o sea,

de donde

1 +2x2y' = C1.
,
CQ.xZ
1- t de mo do que
En t onces, y=

q -1, e2 ,
Y=--

do u d e

e1* --

1-C1

. Por consigu iente, las ex trema les representan una fam tlia
2
de hiprbolas. De terminemos la extrcmal que pas :i. por los puntos
fjos. Para ha ll a r los v alores de l as constantes Cf y C2 ormamos e l
sistema

de donde res u Ita

cr =

- 4 y Cz

= 7.

La ex trema 1 buscada es y =

= 7- .!.
X
5". F no depende explcitamente de x, o s ea, F
En este caso la ecuacin de. E ul er tie ne la for ma

Fy - Fyy ' Y' - F11' 11'!1h = 0.

F (y, y') .

60

CAP. 11. EXTREMO DE FUNCIONALES

Multiplicando ambos miembros de esta ecuacin por y', ob tendremos


en el primer miembro la derivada exacta
ecuacin ser

:i> (F- y'F

71

,)

:x

(F - y'Fv>

sea,

la

=O, de donde
(14)

F - y'F11 , =C1

s iendo C1 una constan le arbitraria. Esta ecuacin puede ser integrada


resolvindola respecto a y' y separando las v ariables o introduciendo un
parmetro.
EJEMPLO 12 (cuerpo de resistencia mnima en un fluid o). Deler
minar la for ma del cuerpo slido que, al moverse en un flui do de gas,

del fluido
/(

do gas

Fig. 4
encuentra resistencia mnima. P ara simplificar consideraremos el
cuerpo de revolucin (fig. 4).
sOLUCIN Suponiendo que la densidad del gas es suficientemente
pequea y que las molculas, al chocar con la superficie del cuerpo, se
reflejan de forma especular, obtenemos para la componente normal de
Ja presin
p = 2pvll sen2 e.
donde p es la densidad del gas, v es la velocidad del gas respecto al
cuerpo y fl es el ngulo entre la velocidad y su componente tangencial.
La presin es perpendicular a la superficie de modo que la componente
segn el eje Ox de la fuerza que acta sobre un anillo de anchura
(1
y') l/2 dx y de radio y (x) se puede representar en la forma

df =2pu2 sen2 0 [2ny (1

+y'2)7 I sen Odx.

PROBLEMA ELEMENTAL

61

La fuerza resultante que acta en la direccin positiva del eje Ox


es igual a
l

F=

4npo2 sen3 6y ( 1+y'2)2 dx.

o
Supongamos, para simplificar el problema, que

Entonces, la fuerza de resistencia ser igual a


l

Jo

F= 4n:pu2

(15)

y'3y dx.

El problema consiste en hallar la funcin y (x) en la que F alcanza

su valor menor posible siendo


y (O)

y (l)

R..

(16)

La ecuacin de Euler para la funcional (15) tene la forma

::e (yy'2.)=0.

y'-3

(17)

La solucin particular y= O de esta ecuacin debe ser rechazada en


virtud de las condiciones de frontera (16). La ecuacin (17) puede ser
representada as! :
y' 3
3yy'y = o.
(18)

Multiplicando p,or y' ambos miembros de (18), vemos que el primer


miembro es (y 3 y)'. Integrando, encontramos
y'Sy

= q,

de donde resulta

, _ V!i
c1

y -

-f

y= ( 1x + C2} '.

Utilizando las condiciones de frontera (16), obtenemos


4

R.3

Ci = de modo que

Ca = O

62

CAP. 11. EXTREMO DE FUNCIONALES

o sea, el contorno con ex tremas fijos que corresponde a l a resistencia

!4 .

mnima del cuerpo es una parbola de grado

EJEMPLO 13 . Hallar la extremal de la funcional

y (x)] =--

rbV,l
+y'?.
!I
dx

J
Cl

que p~ sa por dos puntos fijos (a, A) y (b, B) pertenecientes al semiplano


supenor.
SOLUCJON. Puesto que la funcin integrando no contiene explcitamente x, la ecuacin de Euler, segn (14). da

Vr+Y'2
y

Despus de simplificar, encontramos y


1

lf 1 +

y''j

= e\,

= Ct. Integrando la ltima ecuacin, encontramos (x

donde

C1 =

+ CiJi +y'=

= Cf, o sea, una familia de circunferencias con centros en el eje Ox.


La extremal pedida ser la que pase por los puntos fijos. El problema
tiene solucin nica ya que por dos puntos cualesquiera del semi plano
superior pasa una y slo una semicircunferencia con centro en el eje Ox.
OBSERVACJN. Segn el principio de Fermat, el camino que recorre
un rayo de luz al propagarse con la velocidad u (x, y) en un medio bidimensional no homogneo constituye una extrema! de la funcional
xi

J [y (x)) =

lf--::y;2

(J

(x, y) dx.

xo

Como hemos visto en el ejemplo anterior, si la velocidad de la luz t1


es proporcional a y, los rayos de luz representan arcos de circunferencias
con centros en el e.je. Ox.
Sea q una curva. Denominaremos longitud ptica de la curoa q
el tiempo T (q) que se precisa para recorrerla al moverse segn esta
curva con la velocidad de la luz t1 (x, y}.
Supongamos que el semiplano superior y > O es un medio ptico
en el que. la velocidad de la lui en todo punto es igual a la ordenada del
mismo: v = y. Como hemos visto, los rayos de luz en este medio sern
semicircunferencias con centros en el eje Ox. Se puede demostrar que,
si uno de los extremos del arco AD de la semicircunferencia q se halla
'!n el eje Ox, su longitud ptica es infinita (ig. 5). Por eso, diremos
que los puntos del eje Ox estn en el infinito. Consideremos que 1as
semicrcunfer<.>ncias con centros en el eje Ox son rectas, que las longilu
des pticas de los arcos de estas semicircunferencias son las longitudes
de dichas rectas y que los ngulos entre las tangentes a las semicircunferencias en el punto de interseccin de las mismas son los ng ~ los

PROBLEMA ELEMENTAL

4.

63

entre d ichas rectas. Obtenemos una Geometra plana en la que se


conservan muchas proposiciones de la Geometra habitual. Por ejem
plo, por dos puntos fij os se puede trazar una recta y slo un a (y a que

Fig. S
por dos puntos del semiplano se puede trazar slo una semicircunfefenci a con cent ro en el eje Ox). Dos rectas se consideran paralelas si
tienen un punto infinito comn (o sea, si las dos semic ircunferencias
y

111

Fig. 6
son tangentes en un punto B pertenec:ientt' al eje Ox). Entonces, por
todo punto A que no se hal!e en la recta q se pueden trazar dos rectas q,
y q2 paral<'las a q. Liis rectas que pasan por el punto A v que se encuentran en los ngulos vt>rtkales l y l ll, cortan la recta q mientras que las
rectas que se encuentran en los ngu los Il y IV no la cortan.
Hemos obtenido el modelo de Poincar de la Geometra plana de
Lobachevski (lig. 6).

64

CA P. Jl. l!XTREMO DE FUNC IONA LES

Hallar las extremales de las funcionales:


b

81. J[y(x)]=

S[2xy+ (x + e")y']dx;
2

y(a) =A,

y(b)=B.
t

82. J(y (x)l=


83. J [y (x)l =

Jo (e + x/)dx; y(O)=O, y(l)=a.


11

ztJ/4

(
(y'2-y2) dx; y (O)= 1, y

T .) = 112
2

o
1t

84. J [y(x))= } (y' 2 -y2)dx; y(O)= 1, y(n) = -1.


o
1

J(x+ !/ )dx; y(O) = 1, y( l ) = 2.


o
J[y(x)J = J(y + y' )dx; y(O)=O, y(l)= l.

85. J

lY (X)l =

86.

o
i

fo (y' +4y2)dx; y(O)=e y( l )=l.


Jo (2'-y )dx; y(O)=l, y( l) =e.
(x)] = J
+ y'2) d x .
2

87. J[y(x)I=

88. J[y(x)]=

89. J [y

(xy'

(1

90. J (y(x)] =

J" (Y+ Y;) dx.


a

91. Demostrar que no tiene extremos la funciona l lineal


b

J[y(x)) =

J[p(x)y' +q(x}y+ r(x)] dx,


Q

donde p (x) E c 1 la, bl, q (x) E e [a, bl y r (x) E C [a, bl,

PROBLEMA ELEMENTAL

65

92. Consideremos la funcional


J[y(x)]=

J" F(x, y, y')dx


a

con las condiciones de frontera y (a) = A e y (b) = B. Demos


trar que la ecuacin de Euler subsiste al agregar al integrando
F (x, y, y') dx Ja diferencial tot al de cualquier funcin u =
=u (x, y).
b

93. J ry (x)l =

J(y + y' +2yex) dx.


2

94. J [y(x)J

='Y

(!2-y' 2 -8ych x) dx;

1)

y(0 )=2 ,

y (

~) =2ch ~ .

95. Hallar las extremales de la funcional


b

J (y (x}I =

Jxny'

dx

y probar que para n ~ 1 no existen extremales que pasen


por dos puntos situados a distintos lados del eje Oy.
PROBLEMAS VARIACIONALBS EN FORMA PARAMETRJCA. En muchos
problemas es cmodo, y a veces Imprescindible, emplear l a representa
cin paramtrica de las lineas
X=

<p (/), }

Y=\ll (t),

t 0 ~ t-<1 1

donde <p (t) y ip (1) son funciones continuas con derivdas continuas
2
2
a trows siendo, adems, ( ~~ )
4= o.
Consideremos la funcional

+ ( '!':' )

le=

donde

dx

t1

Je

F(f, x,

y,;,

F(t, x,

dy

x =- e y= df

~-0 1 3&7

y)df =

y, -~

y)dt,

(19)

CAP. ii. EX'tR.eMO DE lftJNCIONALES


Para que los valores de la funcional (19) dependan slo de la
lnea, y no de su paramelrizacin que puede efectuarse de distintos
modos, es necesario y suficiente que la funcin integrando no contenga
explcitamente el parmetro t y sea positivamente homognea de grado
uno respecto a Jos argumentos e

x y:

F (x, y,

h,

ky)=kF (x, 11,

x, y),

k> O.

Por ejemplo, en la funcional


Je =

Je

x dy - y dx

la funcin Integrando es positivamente homognea de primer grado.


Efeclivamente, tenemos
F (x, y. ;,

g) = xy-y~

y es obvio que

F (x, y, k;,

Si Ja curva

ttY) =kF (x,

y,

x, y).

ofrece el extremo a la funcional Je en la clase de lineas C que unen los


puntos fijos (x0 , y 0 ) y (x11 y1), las funciones q> (t) y 'i> (t) satisfacen
las ecuaciones de Euler

F,-: (F;)=O,}

(20)

Fv- -;(F. ) =O.


11

Una de las ecuaciones (20) es consecuencia de Ja otra.


Las ecuaciones de Euler se pueden representar en la forma de
Weiersfrass

F.-F.
%1/

-=
r

11%

(21)

3 '

F (;2+y2)2
donde res el radio de curvatura de la extrema! y F 1 es el valor comn
de las razones
F ..
P
F ..
%X

1/1/

%1f

F1=-- = - - = --

yi

;z

_;9

f 4.

PROBLEMA LEMENtAL
EJEMPLO 14. Hallar las extremales de la funcion al
(XI lft)

J y2g'2

Je=

dx.

(0, O)

SOLUCIN. Puesto que puede haber extrema les que se cortan con las
rectas paralelas al eje Oy en ms de un punto, consideraremos el pro-

blema en forma paramtrica .


Poni~ndo x = x (t) e y= y (t), encontramos que la funcin

y2 .

lrifegrando tiene la forma y' -;-- x, o sea, es posit ivamente homognea


.x2

g.

de primer grado respecto a ~ e


La primera de las ecuaciones (20) da

..!.
(g1. yi
) = o'
dt

xz

de donde

2) =Cf.
2

Y' (

Integra n do la ltima ecuacin, encontramos


y1 = 2C1x

C2.

Puesto que la extrema! debe pasar por el origen de coordenadas,

!J;,,
1

tenemos C2 = O. La segunda condicin de rrontera da C1

o sea, en definitiva,

EJEMPl.O 1s.

Hallar las extremales de la funcional


t1

Je=

JlVx2+2+a2<x.V-yx)J dt.
to

SOLUCIN. Poniendo

F(x,

y,;,

u)=V;s+i+a(xy-y;),

vemos que la funcin F es positivamente homognea de primer grado


respecto a ~ e
Empleemos las ecuaciones de Euler en la forma de

y.

68

CAP. 11. EXTREMO DE PUNCIONALES

Wcierstrass. T~memos
F

F.=- -a2

----3-

F1=---:-- =
g?.
(~Z+g2)2
Por eso, la ecuacin (2 1) tiene en nuestro caso la forma
l
-=2a2,
l/X

Es decir, la curvatura .!.. de la extrema! es constante. Por lo tanto, las


r
extremales son arcos de circunferencias; en particular. se tienen circunferencias completas si
x(lo) = x(t1), }
!I (lo)= Y (t l)

Hallar las extremales de las funcionales:


(x1, 1/f)

96. Je= }

gz-yzxz di.

(0, 0)

)C

(l, 2)

97. Je =

!/ -3el//X;c'I.

(0, O}

di.

(1, O)

J (K V~2 +y2 -~y)dt,

98. le=

( - 1, O)

donde K >O es una constante.

S Generalizaciones del problema elemental


del Clculo variacional
J. funcionales que dependen de derlvedes de rdenes superiores,
Supongamos que se tiene la runclonal
XI

J [.Y (x)J =

SF {x,
:o

g (x), y' (x), ... , g<n> (x)J dx,

(1)

donde F es una funcin diferenciable n


2 veces respecto a todos los
argumentos e y (x) E C11 [x0 , x 1], y supongamos que las condiciones de
(rontera tienen la forma

= !IO

Y (xo) = Y, , y< n-1> (xo) = ,;,,n- i),


!/ (xl)=Yit !I' (x1)=111'. ,y<1H> (X1)=y\n-o.

!/:("o)

(2)

GENERALIZ /\CIONES DEL PROBLEM A ELEMENTAi.

69

Las extremales de la funcional (1) con las condiciones (2) son las
curvas integrales de la ecuacin de Euler - Posson

d
Fv - Tx fv, +

d2
dn
dx2 Fii#- . . . + (- t)n dxn f11<n> =0.

EJEMPLO 1. H allar la extrema! de la funcional


1

J {y (x))-

(360x2y - ywZ) dx;

o
y (O) = O,

y' (0)

y (1)

1,

SOLUCJN. La ecuacin de E uler -

360x2

tf'l

+ dx2

= o,

y' (1)

2,5.

Poisson tiene la forma

(-2y")=0, o sea, y1v

180x2,

y s u solucin general es
t
11 =2 x6 -I C1x3 +C2xZ + C3x+ C4
Empleando las condiciones de [rontera, e ncontramos
C1 =

2 .

C2 =-- -3,

C;i=l

C4 - 0.

La extrema! pedida es
1

u=c x6+ 2

x3-3x2 +

x.

Consideremos el caso cuando en l a frontera no se dan todas las


condiciones (2) s ino un nmero de las mis mas de modo que, despus
de emplear l as condiciones de frontera, en la solucin general de la
ecuacin de Euler - Poisson contienen lodavia constantes arbitrarias_
Para resolver este problema es preciso hallar l a va riacin de la funcional (1), transformarla tomando en cons ideracin las condiciones de
fron tera dadas y obtener condiciones complementarias l' n la frontera
igualando Ja variacin a cero.
EJEMPLO 2. Hallar la curva y = y (x) que ofrece valo r ex tre ma! a
la funciona\
b

J (Y (x)J =

TJ

y 'l dx

(3)

con las condiciones

y (a) = O

y (b) '= O.

SOLUCIN. La ecuacin de Euler f/ IV

0.

Poisson tiene la forma.

(4)

70

CAP. 11. EXTREMO DE PUNCIONALES

Su solu cin general

y = C1

+ C,.x +

C3 x2

+ C, x3

(5)

contiene cuatro constan les arbitrarias Ct (i = J , 2, 3 y 4) y las condiciones de fro ntera (4) no bastan para de terminarlas. Por eso, como
hemos explicado, calc ulamos la variacin de la fun cional (3). Tenemos
b

6J =

y(jy dx.

(6)

CI

Integrando (G) por partes dos veces, obtenemos


b

BJ =y" (x) (jy' (x)

1:- J

y"'fJy' d."'=

'1

=y" (x) 6y' (x)

1: - y"' (x} 6y (x) 1: + JyIYfJy dx.

(7)

La expresin (7) debe anularse en la extrema} y (x) de la fun cional


(3). Debido a la arbitrariedad de Ja funcin 6y, resulta que y 1v = O;

sta es la ecuacin de -Euler - Poisson para la funcionar (3). Pero si


l a Integral del timo miembro de (7) se anula, la expresin. de frontera

fy"

(x) fJy ' (x) - y"' (x) 6y (x)J

J:

tambin debe ser igual a cero idnticamente. Puesto que f>y (a) =
y () = O (extremos fijos), resulta que debe ser

y" (b) 6y' (b) - y" (a) fJy' (a) = O.


En virtud de la arb itrariedad de las magnitudes 6y' () y fJy' (a},
obtene mos necesariamente
y" (a)

y" (b)

= O.

(8)

Las condiciones (8) conjuntamente con las condiciones (4) determinan


unvoca mente la extrema! en la familia (5): y
O.

2. Funciona les que dependen de m funciones. En el caso de una


funcional que depende de m funciones y 1 (x), Ya (x), ... , Ym (x)
x1

J (Yt. Y21 . , Yml =

J F (x, Ytr J/2, , J/m. Y,

y;, .. , y;,.) dx

y con las condiciones de frontera de t ipo


Yh (xo) = y~.

IJk (Xi) =

Yl

(k = 1, 2! ... , m)

p.

G ENERALIZACION ES DEL PROBLEMA EL EME NTA L

71

las extremales se determinan del siguiente sistema de ecuaciones diferenciales de segundo orden
F

vk

d
dx

- - F , =0
Vtt

(k= 1, 2, .... m)

f.91

que se denomina sistema de ecuaciones de Euler.


EJEMPLO 3 . Hallar las extremales de la funcional
2

J [y (x), z (x))=) (g'2 + z2 +z'2) dx


t

con las condiciones de frontera


y (1) = 1,
y (2) = 2,

z ( 1) =

o,

z (2) = l.

SOLUCcN. En este caso el sistema de ecuaciones (9) tiene la forma

y =o, }
z- z" = O.
Resolviendo este sistema, encontramos
y = C1x
C2 Y z = C8 ex + C~e-:c.
En virtud de las condiciones de frontera, tenemos
1
e2
C 1 =1, C2=0, C3 = -e -2 -1
y
C1t.= - e2-J
de modo que la extrema! pedida

Y=X,
}
sh (X-1)
Z=
shl

es una curva alabeada que constituye Ja interseccin de dos superfici es


ci1lndricas.
EJEMPLO " HaJlar las extremales de la funcional
J fy (x:}, z {x)J =

J" (2yz-2y2 +y'2-z'2)

dx

o
si

O,
y (n) = I,
z (0) = O
y
z (n) = -1.
sOLUClON. El sistema de ecuaciones (9) tiene Ja forma
y" +2y - Z= , }
y (O)

Z'+y=O,
de donde, eli minando la funcin z, obtenemos
YIV
2y
y= 0.
La solucin general de esta ecuacin tiene la forma
!J = C1 cos ~
C 1 sen x
x (Ca cos x
C4 sen x}.

72

CAP. 11 . EXTREMO OF. FUNCIONA LES

En virtud de las condiciones de frontera y (O) = O e y (n)


1
C1 = O y C3 = - - de modo que

1, tenemos

1t

g=C2 sen x+C,xsen x- - cosx.


n

z se determina de la condicin z = g" + 2y. Tenemos


1
z= C2 ~nx+C.\ (2cos x+x sen x}+- (2 sen x-x cosx).
n

La funcin

Las constantes C'J y C 4 se determinan de las condiciones de frontera

z (0) = O y z (n) = -1 , de donde resulta que C4 = O y que C2 es


arbitrarlo. Entonces.

1
z = C2 sen x+- (2 sen x-x cos x).
n
La familia de extremales es

11=C2senx-_:_cosx,

x~x

!~

, ~~ }dx

z = C2 sen x+
(2 sen
cos x},
donde Cz es una constante arbitrarla.
3. Funcionales que dependen de funciones de varias variables
independientes. Consideremos la funcional
J [z (x, g)J =

j ~ F ( x,

y , z,

ay,

(10)

donde F es una funcin diferenciable tres veces respecto a sus argumentos, y supongamos que se pide hallar la funcin z = z (x, y) que sea
continua conjuntamente con sus derivadas hasta de segundo orden
inclusive en el recinto D, que lome valores fijos en la frontera f del
recinto D y que realice el extremo de la fun cional ( 10).
Si el extremo de la funcional (10) se alcanza en la superficie
z = z (x, y), la funcin z = z (x, y) satisface la ecuacin de Eu ler Ostrogradski

F:i.- ax {Fp}- ay {Fq}=O,

o {Fp}
Tx

(11)

(Fq} son las derivadas parciales completas


011
respecto a x e y, respectivamente:

donde

ax{Fp}

ay {Fq}

az

op

oq

op

oq

Fpx + Fp:r:ax+Fppa-+Fpq ox ,

az

= Fqu +Fqz ?Jg

+ Fqp Ty +Fqq ay ~
oz

aqu se ha lomado, para abreviar, itl' = p

az
iJJI

= q,.

GENERALIZACIONES DEL PROBLEMA ELEMBNTAL

73

La ecuacin (11} representa la condicin necesaria de extremo de


la funcional (10). Es una ecuacin en derivadas parciales de segundo
orden; se busca su solucin z = z (x, y} que toma valores fijos en la
frontera r.
EJEMPLO 5. Escribi( la ecuacin de Euler - Ostrogradski para
la funcional
J !z(x, y)] =

JJ(( ~: )2-( :~ }2]

dxdg.

D
SOLUCJN

tramos

Tenemos F(x, y,z, p,

y, segn (11), encon-

q)-p2-qZ

a (2p)- a (-2q) =O, o sea,


-ax011
o2z

02z

Jx2 -

iJy?. =O.

Pua la funcional

J[:z(xh

xz, ... ,

Xn)J =

5J... J

F(xtt x2, ... ,

Xn,

z, Pt

pz, .. ., Pn) dx1 dx2 .. dxn.,

oz
-

donde p11 =
(k= l, 2, ... , 11), la condicin necesaria de extremo
0 X11
viene dada por la siguiente ecuacin de Euler-Ostrogradski
n

Fz-

~ 0~t

{Fpi}=O,

... 1

o, en forma desarrollada,
n

Fz-

( FXiP+FzPiPt

+ ~ Fppj
J-t

1-1

a:: )=

(12)

La funcin z = z (x1 , x,, ... , xn.), solucin de esta ecuacin, debe


satisfacer en la frontera r del recinto n-dimensional D las condiciones
de frontera dadas.
EJEMPLO 6. Hallar las condiciones que debe cumplir la funcin
z (x1, x2 , , xn) para que la integral de Drlchlet

D[i(x1.~,

... ,xn)l=)J

...
Q

J ~ (::,

)2dx1dx2 ... dxn

i-1

alcance en ella su mnimo si dicha fncin toma valores


la fronter~ r del recinto Q.

en

c:Ie~erminados

74

CAP. 11. EXTREMO DE FUNCIONALES


n

SOLUCJON. En este caso F=

(::t.)2 ,

o sea, F no depende

{ ... j

expHcitamente de

x2 ,

Xt.

, Xn,

z. Por lo tanto,

Fi=Fip =F~ipi = O,

= {

F .

2 si l = j,
0 si i =/= j

PtPJ

y, aplicando la 6rmula (12), obtenemos


n

"\."' iJ?..z =0
4..1 ox~

; ... t

(ecuacin n dimensional de Laplace).


OBSERVACION. Si bajo el signo de la integral figuran las derivadas
de Ja funcin z (x, y) hasta de orden 11, la ecuacin de Euler - Ostro
gradski tiene la forma
a

ifl

az

Pi - ax {Fzx}- iJy {Fzu} + iJxz {Fzxx} + 2 iJxou {Frx11> +

az

iJn

+ iJy2 {Fzw}- ... + (-J)n ayn

--

{Fzv11 ... 11} = 0.

(13)

EJEMPLO 1.

funcional
J (z (x, y)J = )

Escribir la ecuacin de Euler -

Ostrogradski para la

J[ ( :x~ ) + ( :~ r+
2

+2 ( iJxaiziJy )2 SOLUCIN

2zf (x, y) dx dy .

Tenemos
f)'l.z ) 2

F = ( iJxZ

+{

(}2z ) 2

0112

iJ'l.z } a
+2 ( aia
-2zf (x,

y).

Aplicando la frmula (13). encontramos

-2t (x,

.!/)

(]2
+ ox2

i)'l.z )

2 axz

iJZ
+ iJy2

iJZz )
2 iJy2

+
()2

+ 2 axay
9 sea,

2 ()Zz

a.xay

p.

GENERALIZACIONES DEL PROBLEMA ELEMENTAL

75

La ltima ecuacin se representa brevemente asi:


M.'>Z = f (x, y).

Hallar las extremales de las funcionales s iguientes:

Jo (y +2y.'2 +J/2) dx;


J

99. J [y (x}l =

y (O)= O,

y ( l) =O,

y' (O} = 1

e y' ( 1) = - sh 1 .

100. J [y (x)J =

J(240y-y"'2) dx;
-1

y(-1)= l,
y'(O)=O,

y(O)=O,
y"(-1)=16,

y' ( - 1) = - 4 ,5'
y" (O)= O.

101. J [y (x)] - ) (y+ y") dx;


a

y(a)-Yo.
102. J [y (x)J =

y(b) =YJt

y' (a)= y~.

y' (b) =Y;.

y(b) =B.,

y' (b) = 8 2

J" (y' + yy") dx;


2

cJ

y(a) =A 1 ,

y' (a) =A2 ,

f03. J {g (x)J =

J(y' + f/
2

2)

dx;

o
y(O)=O, y(l)=shl, y'(O)=l,
104. Hallar la extremal de la fundonal

y'(l)=chl.

J (y (x)l = -} }

tf2 dx

o
con 1as condiciones
y (O) = O,

y' (O) = O,

y' (1) = l.

11/l

105. J [y (x), z (x)] =

J{2z-4!f + y' -z'


2

dx;

V(O)= O, y ( ~ ) = 1~ z (O)= O, z ( ; ) = J.

76

JI . EXTREMO DE FUNC IONALES

CAP

S(2xy-y' 2 + ~

106. J[y(x), z(x)J=

) dx;

-1

z(l)=I,

y(-1}=2,

y(l)=O,

z(-1)=-l.

107. J[y(x), z(x)J=

(y'2 +z'2 -2yz) dx;

y (0) =O, y ( ~ ) = 1, z (O)= O, z ( ~ ) = l.


t

108. J[y(x), z(x)J =

J(y' +z' +2y)dx


2

y( l )= 2 ,

y(0)=1,

2(0)=0,

z(l)=l.

109. Probar que la ecuacin de Euler de Ja funcional


b

J[y(x), z(x)]=

F(x, y, z, y', z')dx

tiene. las s iguientes primeras integrales:


-JF

1) oy' = C si F no comprende y;
oF
2) F - y , iJy'

e s1. F

no comprende x.
Escribir la ecuacin de Euler-Ostrogradski para las
funcionales:
J

- z

110. J (z(x, y)]=

iJF
oz =

J~ [( :: ) +
4

+( :;
111. J [i(x, y)]=

)4+l2zf(x, y)jdxdy.

JJ(~:~ + ~:~ )2dxdy.


D

t 12. J [z (xi. Xz, ... , Xn )l =

=~
.-c(x,, x 2 ,

r Jr~a
n

j ... 1

(Xi, Xz, ' Xn) (

::J )2-

Xn)z2 +2z/(xtt x 2, ... , Xn)Jdx1 dxa ... dxn

iN VARiANCI A

oe

LA ECUACIN DE EULE R

11

113. Deducir la ecuacin diferencial de las superficies


de rea mnima.
114. Hallar la extrema! de la funcional
1 1

J Jezv sen .z dx dy

J [z (x, y)]=

11

o o

con las condiciones z (x, O) = O y

(x, 1) = J.

6. lnvarianda de la ecuacin de Euler


Si la funcional
I>

J [y (x)] =

F (x, y , y') dx

se transforma efectua ndo una sustitucin de Ja variable independiente


o una sustitucin simultnea de la funcin incgnita y de la variable
independiente, las extremales continan determinndose de la ecuacin de Euler que se obtiene a partir del Integrando transformado. En
esto consiste la invariancia de la ecuacin de Euler.
Sea x = x (u, v) e y = y (u , u) con la particularidad de que

Entonces

F (x, y, y')dx=

Xr,

Xo

1Yu

Yo

1=f- Q.

r[
F

x(u, v), y(u, v),


X (xu

!Ju + !J11V~
Xu

+ Xvtlu,

+ x 0 v~) du =

<Il (u, v,

v~) du

y las extremales de la funcional inicial se determinan de la ecuacin


de Euler para la funcional

<D (u, v,

v~) du:

d
<Dv-- <D0 ,=0.
du
EJEMPLO 1.

Hallar las extremales de la funcional

"'1
J [r (q>)J=

Jvri+r'2 dq.

"'

18

E.XT~MO

cM>. 11.
SOLUCION.

DE

PUNCONLES

La ecuacin de Euler para esta funcional es

V r2+r'2

d
r'
-~ yrz+r2

o.

La sustitucin de variables x=rcosq> e 11 = rsenlp da


y llt!Va a la funcional
b

J [JI (x)] =

J'\l +

y'2 dx

cuya ecuacin de E uler es 11" .... O de modo que

!1 = C1x +Cs.

Por consiguiente, las extremales de la funcional inicial vienen dadas


por la ecuacin
r sen qi = C1 r cos c:p
C1 ,

donde C1 y C1 son constantes arbitrarlas.


EJEMPLO 2. Hallar las extremales de la funcional
lD 2

J (y (x)} =

(r"11'2- exyZ) dx.

o
sOLUCION.

La ecuacin de Eulcrpara la funcional considerada tiene

la forma

y" - y'

+ et=y == o.

Realicemos la sustit ucin de variables

x= lnu, }
11=v.
La funciona l inicial se transforma entonces en
2

t(v (u)] =

(e-In "u2v'2-elll vv2)

~=

(v'Z - v2) du

y su ecuacin de Euler v"


V=

+ v = O se integra fcilmente:
C1 COS

U+

C1 sen

U,

Volviendo a las coordenadas iniciales x e y, obtenemos la ecuacin


de las extremales en la forma
g = C1 cos eo:
Ca sen e".

s.

NVtANCIA DE LA EUACIN DE

eUtER

70

115. Ha llar las extremales de la funcional


Ql(

Jrsen<p Vr +r' dcp.


2

J (r(<p)) =

!fo

116. Probar que las extremales de la funcional


'Pi

J [r (q>)] =

i f (r sen q>) V r+ r' d<p


2

To

se determinan por cuadraturas.


117. Hallar las extremales de Ja funcional
b

J (y(x)) =

JY x2+ y lf l + y' dx.


2

Igual que en el caso de una variable, la ecuacin de Euler Ostrogradskl es invariante respecto a las transformaciones de coorde

nadas.

EJEMPLO

a. Escribir la ecuacin de Laplace


iJZz

<)Zz

axa-+ ay2 =O

(t)

en coordenadas polares.
SOLUCION.

Consideremos la funcional
D (z (x, y)]=

5 {z~+z~)

dx dy.

La ecuacin de Euler - Ostrogradski para esta funcional es precisa


mente la ecuacin (l). Pasemos en la funcional de las coordenadas
cartesianas (x, y) a las coordenadas polares (p, q>): .t = p cos cp, y =
= p sen q>. Tenemos
op
op
oq>
sen q>
aqi cos q>
ax = cos cp,
iJy =sen qi,
--p- '
ay =-p-

ax=

y, por eso,

D (z (p, q>)J =

JJ[(

Zp

:~ +

Zqi

~~ )2 +

+(zp g~

+zq>

~~

) ]pdpdq>=

JJ(pz~+~z~)dpdq>.
G

Formando ta ecuacin de Euler -

Ostrogradski para esta ltima


.Integral, obtendremos la ecuacin de Laplace en coordenadas polares;
p.1 Zcpq> pzpp Zp = 0.

cAP.

i1. ~ xTR EM

oe l'lvNctNAu1s

7. Campo de extremales
La familia de curvas y= y (x, e) forma un campo propio en el
recinto D del plano xOy si por cada punto (x, y) de este recinto pasa
una y slo una curva de la familia y = y (x , e).
El coeficiente angular p (x, y) de la tangente a Ja curva de la
fa milia y = y (x, e) que pasa por el pun to (x, y) se denomina inclinacin
del campo en el punto (x, y).
La familia de curvas y= y (x, e) forma un campo central en el
recinto D del plano xOy si estas curvas cubren sin cruzarse todo el
y

Fig. 7J
reci nto D y arrancan de un mismo punto (x0 , y 0) que no pertenece al
recinto D. El punto (x0 , y 0 ) se llama centro del haz de curvas.
EJEMPLO 1. Dentro del circulo x1 + y2 ~ 1 la familia de curvas
y= Ce.,., donde Ces una constante arbitraria y, en particular, C = O,
fo rma un campo propio y a qu e estas curvas no se cortan en ningn
punto y por todo punto (x, y) del crculo pasa una y slo una curva
de esta familia (lig. 7). La lncllnacl6n del campo en un punto cual
qu iera (x, y) es igual a

= y.
2. La familia de parbolas y = (x + C)'' no fo rma campo
2
p (x. y)

EJEMPLO

ce~

propio den~ro del crculo x + y"~ l porque distintas curvas de la


familia se cortan dentro del circulo y no cubren todo el recinto (fig. 8),
EJEMPLO 3. La fam ilia de curvas y = Cx forma un campo central
en el recinto x > O.

s 7.

CAMPO DE EXTREMALES

81

Fig. 8

fforman campo (propio o central) en los recintos indicados las siguientes familias de curvas?
tJ 8. y=Ctgx;

11 9. y=Ccosx;

a) lx!<~;

b)

120. y=(x- C)3;

Si

2 <x~n ;
x'l.

e)

y2

4 + 9 ~ l.

12 1. y=C(x2-2x);

a)

O~x< l ;

122 .

y= e sen

n:

a)

b)

-L~x ~3 ;

(x - : ) ;
n

4 ~ x~ 2 ; b) 3 ~x~n ;

123. y = ex+c;

e)

8 ~x ~ 2n.

x2+y2~ 1.

S i el campo (propio o central) est formado por una fa milia de


ex lremales de cierto problema va riaciona l, se deno mi na campo de

extrema les.
6- 01387

82

CAP. 11 . EXTREMO DE F UNCIONALES


EJEMPLO

Consi deremos la funcional


i

lY (x)] =

/2 dx.

Sus extremales son las rectas y = C1 x


Ca. La familia de extremale
y = Ca forma un campo propio y la familia de extremales y = C1 x
forma un campo central con centro en el origen de coordenadas.

124. Determinar para la funcional


o

J [y (x)] =

Jo (y' +y) dx
2

los campos de extremales propio y central.


125. Lo mismo para la funcional
n/4

J Jy(x)J =

(y'2-!f+x3+4) dx.

o
Supongamos que la curva y

y (x) es la extremal de la funcional

XI

J fy (x)) =

F (x, y, y') ox

xo
que pasa por los puntos A (x0 , y0) y 8 (x~. y1 ) .
Se dice que la extrema! y = y (x) est Incluida en un campo propio
~extrema/es si existe una familia de extremales y = y (x, C) que forma
un campo y que comprende la extrema! y = y (x) para cierto valor
C = C0 y si, adems, esta extrema! y= y (x) no pertenece a la frontera
del recinto D en el que la familia y = y (x, C) forma campo.
Si existe un haz de extremales, con centro en el punto (x 0 , y 0 ) ,
que forma un campo en una vecindad de la extremal y = y (x) que
pasa por dicho punto, se dice que se ha encontrado un campo central
que incluye Ja extrema! considerada y= y (x). Como parmetro de la
familia y= 1J (x, C) se to ma el coeficiente angular de la tangente a
las curvas del haz en el punto (x0 , Yo)
EJEMPLO 5. Consideremos el problema variacional elemental para
la fun cional
2

J IY (x)] =

Jo

(y'3 + sen2 x) dx.

a) Sea y (0) ::..:: 1 e y (2) = l. La familia de extremales de nuestra


funcional viene dada por la ecuacin y = C1x
C2 La extrema! que
satisface las condiciones de fro ntera es y= l. Dicha ex trema! se
puede incluir ~n el campo propio de extremaJP3 y = Cs. donde e,
es una constan te arbitraria.

tAMPO bE EXTREMALES

i.

b) Sea y () = O e y (2) = 4. La extrema\ q i'sponde a estas


condiciones de frontera es la recta y = 2x que puede ser incluida en
el campo central de extremales u1 - C1x (C1 es una constante arbitraria)
con centro en el punto O (O, O).
y

)(

Fig. 9
EJEMPLO 6. Consideremos el problema variaconal elemental
1

J ( (x)} =

Jy' ( 2x-+ y')

dx;

-1

y ( - 1)=0,

La solucin de la ecuacin de Euler tit!ne la forma

+c2

La extrema! de este problema y =xi+

en el campo propio de extremalcs g=x 2

~-

+: +c

u=x2+c1x+

! ~puede

(lig. 9).

incluir

84

CA P . II. EXTREMO DE F UNCINALES

Probar que las extremale~ de los siguientes problemas


variacionales elementales se pueden incluir en un campo de
extremales (propio o central).
1

126. J(y(x)] =

J(y' 2

2xy)dx;

y(O)=y(1)=0.

o
1

127. J[y(x)] =

1
(2exy+y'
o
J
dx
2

y(O)= l ,

)dx;

y(l)=e.

J 28. J [y (x)J =

(y2-y'2)

(a :;6 kn);

o
y(O)=O ,

y (a)= O

2.

129. J fy(x)] =

J(y' +x ) dx;
2

y (O)= 1,

y(2)=3.

o
OEFI NI CION . Sea I> (x, y , C) =0 una familia de curvas planas. Se
llama C-dtscriminante de esta familia el lugar geomtrico de los puntos
determinado por el sistema de ecuaciones

<D (x, y, C) =0, }

oI> ex, u,
ac

C) _

- 0

( 1)

En el caso general, el C discrimlnante comprende la envolvent e de la


familia, el lugar geomtrico de los pu ntos rnl tiples y el lugar geom
trico de los puntos de retroceso.
la envolven te de la fam ilia <I> (x, y, C) = O es la curva que en
cada uno de sus puntos es tangen te a cierta curva de la famili a consi
derada y tal que cada una de sus partes es tangente a un conjunto nfi
nito de cur vas de la familia.
Si se tiene un haz de curvas con centro en el punto A (x0 , y 0), el
centro del haz pertenece al Cdiscrimnante.
EJEMPLO 1 Hallar el C-<l jscrim inan te de la familia de curvas
Y = (x - q2.
SOLUClN. Las ecuaciones (1) tienen en es te caso 1a forma
y- (x-C)2=0, }
2 (x-C) =0,

de donde y = O Es fcil ver que la lim~a y = O es la envol ven te de


esta familia. Efectivamente, en cada uno de sus puntos x = x0 la
lnea y = O tiene tangente com n con la curva correspondiente y =

p.

CAMPO DE EXTREMALES

85

x 0) 2 de la familia. Adems, si tomamos una parle de la lnea


por pequea que sea, habr un conjunto infnto de curvas de
la familia tangentes a esta parte. En el caso considerado el C-discrimi
nante consta de Ja envolvente nada ms.

= (x y

= O,

En los problemas siguientes hallar los C-discriminantes


de las familias dadas.
130. y = Cx + O.
131. y (C - x) - C2 = O.
132. (x - C) 2 + yi = l.
Si el arco AB de la curva y= y (x) liene un punto comn A,
distinto del punto A, con el Cdiscriminante del haz y = y (x, C) que
tiene su centro en el punto A y que. comprende la curva considerada, se
dice que el punto A* es conjugado del punto A .
EJEMPLO s. Consideremos la familia monoparamtrica de curvas
y= C sen x. El Cdiscriminante de esta familia se determina por las
ecuaciones

y-Csenx = O, }
-!len x = O,
o sea, representa un conjunto discreto de puntos (ktt, O}, k = O, 1.
2, ... (que son los puntos de interseccin de la sinusoide y del eje
y

X.

r!g. 10

e= 2, obtenemos la curva y= 2 sen X


que pertenece a haz de sinusoides con ceniro en el punto O (0, O}.
Si el otro ex tremo B (fig. l O) del arco de la curva y = 2 sen x tiene
Ja abscisa
E (n, 2n), el ar~o OB tendr otro punto (a parle del punto
O (0, O)) perteneciente al C-discriminante, a saber el punto
(n, O),
que ser conjugado del punto O (O, O). Si es o
n, en el arco 08
no habr puntos conjugado: del punto O {O, O).
1
Ox). Tomando, ror ejemplo,

<x <

86

CAP. ll . EXTREMO DE FUNCIONALES

133. Se tiene la familia de curvas y = C (x - l) x.


Hallar el punto conjugado del punto O (O, O).
134. Se tiene la familia de curvas y = C sh x. Hallar
el punto conjugado del punto O (O, O).
1 Condicin suficiente de Jacobl para poder Incluir la extrema!
en un campo central de extrcmales. Condicin suficiente para que el arco
A B de una extrema/ pueda ser incluido en un camfo central de extrema/es
con centro en el punto A (x 0 , y0 ), es que el punto A conjugado del punto A

no perunezca al arco A B.

EJEMPLO 9 . Consideremos la fu ncional


a

J[y (x)J =

(g'2- 9y2

+ e"-"

1) dx;

o
y (O) = O,

y (a) = O.
Analizar la posibilidad de incluir la extrema! y = O en un campo
central de extremales con centro en el punto O (O, O).
SOLUCIN. La ecuacin de Euler para la funcional considerada tiene
la forma !l + 9y =O y su solucin general esy = C1 sen 3.x +Cacos 3x.

Si a =F k;, donde

k es un

nmero en tero, la extrema! que satis-

face las condiciones de frontera es la recta !I = O. Consideremos la


familia monoparamtrica de extremales y = C1 sen 3.r; es fcil ver que el
Cdlscrlminante de esta fa mil la consta de Jos puntos {
es un n(lmero entero; por eso, si a

< ~,

k; . O), donde k

en la extrema! y = O no

habr punto conjugado del punto O (O, O) y entonces esta extrema! se


podr, obviamente, incluir en un campo central de extremales con
centro en el punto O (O, O). En cambio, sf a

> 3.n; .

en la extrema!

y = O habr como mlnimo un punto conjugado del punto O (O, O) y


no se cumplir la condicin suficiente de J acobi; en este caso las ex tri!*
males y = C1 sen 3x no forman campo.
FORMA ANALfTI CA DE LA CONDICIN DB JACOBI.

problema varlaclonal elemental


:lCI

J(y(x)J=

F(x, y, y')d.x;

Y(Xo)==~o.

Consideremos et

y(.x1)==1/1

:i:e

Si la solucin u= u (.x) de la l!Cuacin de Jacobi

(F1111--1r- F1111) u- :x

(F1111u')=O

(2)

que satisface la condicin u (xo) = O se anula tambin en algn otro


punto del Intervalo .r0
x < x1 , el punto A conjugado del punto
A (x 0 , y 0 ) pertenece al arco AB de la extrema! (el punto B tiene las
coordenadas (.rlr Y1

<

CAMPO DE EXTREMALBS

7.

87

Si extste una solucwn u (x) de la ecuacin de J acob que satisface


la condicin u (x0 ) = O y que no se anula en ningn otro punto del semlinteroalo x 0 < x ~ x1 , en el arco AB no habr puntos con/ugados del
punto A. En este caso el arco AB de la extremal se puede incluir en un
campo central de extrema/es con centro en el punto A (x0 , Yfl)
En la euacin (2) hay gue tomar en las funciones Fy 11 (x, y, y'),
F w' (x, y, y ) y F 71 , 11 , (x, y, y ) en lugar de y (x) el segundo miembro de
la ecuacin de la extrema\ IJ = y (x, C0 ).
EJEMPLO 10. Se cumple la condicin de Jacobi para la extrema!
de la funcional

r
a

J (y(x))=

(y'2+x2)dx

que pasa pot los puntos O (O, O) y B (a, 3}?


SOLUCl N. En este caso la ecuacin de Jacobi tiene la forma
u"= O. Su solucin general es u= C1 x + e,. De la condicin u (0) =
= O encontramos e, = O de modo que u = C1 x. Estas soluciones
u = C1x (C1 ::/=O) no se anulan para ningn valor de a > O. Por con
siguiente, en el arco OB de la extrema! no habr punto conjugado del
punto O (0, O). Es decir, este arco se puede incluir en un campo central
de extrema les con centro en el punto O (0, O). Es fcil ver que la extre-

ma! buscada es la recta y =

~
x que se puede
a

incluir, obviamente, en

el campo central de extremales y = C1x.


EJEMPLO 11. Se cumple la condicin de Jacobl para la extremal
de la funcional
a

J[y(x)]=

(y'2-4y2+e-:x:i)dx

(a-=F {n+}) n)

que pasa por los puntos A (0, O) y B (a, O}?


SOLUClN. La ecuacin de J acobi llene la forma u"
4u = O y
su solucin general es u = C1 sen 2x
C1 cos 2x. De la condicin

u {O)

O encontramos Ca = O de modo que u= C1 sen 2~. Si a

<

la funcin u no se anula para O< x..;;; a y la condicin de Jacobi se


cumple; en cambio, si a

> ; ,

la solucin u= C1 sen 2x de Ja ecua-

cin de J acobi se anula en el punto x

= ;

pertenecien te al segmento

{O, al y en el arco de la extrema! y = O (O

a) hay un punto
conjugado del punto A (O, O). Por consiguiente, si a > ~ , no existe
campo central de extremales que comprenda la extrema) dada.

En los problemas siguientes analizar si se cumple o no la


condicin de J acobi.

88

CAP. 11 . EXTREMO DE FUNCIONALES


1

135. J [y (x)] =

J (l 2xy + y' +x2) dx;


2

-1

y(-1)= -2,

y(l)=O.

136. J[y{x)I=

J(y' +9y -3x)dx;


2

y(O)=O,

y(a)= O.

137. J fy(x)] =

J{l -f-y' )dx;


2

y(O)=Y(l)=O.

o
Q

138. J[y{x)]=

Jy'ev'dx ;

y(O) = 1,

y(a) = b.

o
2n

139. J(y(x)] = ) (y' 2 -y'l.)dx;


o

y() = O,

y(2n)=l.

140. Demostrar que. si la funci n integrando de la fun-

cional

J fy(x}J =

JF(x, y')dx
a

no contiene y explcitamente, cualquier extremal puede ser


siempre incluida en un campo de extremales.
OBSB~VACION. La condicin de Jacobi es necesaria para que la
runcional J
(x)I alcance su valor extremo: o sea, cuando la extrema!
A 8 realiza e extremo, el punto conjugado de A no puede estar en el
intervalo xi)< x < x1 Por ejemplo, la funcional

ly

a
J[y(x)J = .\ (y'4 + 1)dx;

y(O)=y(a)=O;

o
alcanza su valor mnimo en la exlremal y e O. En esta extrema! no

hay puntos conjugados del punto O (O, 0).


EJEMPLO 12.

La funcional
~

;-n
J{y(x)}=

Jo

(V2-y'2)dx

V(O)=O,

u(: n) ==o

89

CAMPO DE E X TREM A LES

7.

no alcanza extremo en la e.x tremal y == O porque en el intervalo


{O,
est el punto o (n, O} conjugado del punto O (O, O) (ya que
Ja solucin de la ecuacin de Jacobi que se anul a en x = O es u=

! Jt)

nE (O, ! n)).

=C1 sen x y u se anula tambin en el punto x =


Efectivamente, tomemos como curva
sen
!In (x)

; para esta curva se cumplen C>LP1 iamt>nte las con-

y~ (x} = 5~

tenemos J [O) = O y

sen : 11xJ ~ n
=

n 2.

a y = O la curva

nx

n<l

5
diciones y (0) = y ( : ) = O e

prxima~

Jo

cos n

x.

Entonces

dx-

T"

- Jr (-5n4- ) 2 cos2 (~
x) t!X =
5
o

Sn

8n2

(-1121--~)
25

< 0

para todo entero n ~ 2. Por consiguientc, la extremal y =O no realiza


el mnimo de la funcional cons iderada ya que existen curvas prxi mas
ay= O en las que son negativos los valores de la funci onal. Considere
mos ahora 1a familia de cur vas !In (x) = +sen ~ x cercanas a la curva
y s O en el sentdo de proximidad de orden cualquiera. Es facil ver que
5

J IYn (x)j =

T n sen2~x
5

Jr

n2.

Tn

Jr

dx -

Por consiguiente, la extrema! 11


de la funcional considerada.

16
2 4
9n
252ros 5 xd."t:= 40n2 >O.
~

O tampoco realiza el mximo

141. Supongamos que en la funcional


b

J fy (x)] =

JF (x, y, y'} dx
a

la funcin integrando F tiene derivadas parciales acotadas


res~~cto a las variables !/ e y' en cualquier

de tercer orden

90

CAP. 11. EXTREMO DE FUNCIONALES

recinto acotado de variacin de y e y'. Sean y = y (x) e y =


= y (x) + 11 (x) dos extremales cercanas. Demostrar que 1a
funcin fJ (x) satisface la ecuacin de Jacobi

f 111/YI

+ F1111'lJ

1
-

:X

(F1111'fJ

+ F1111TJ

1
)

= 0

salvo una infinitsima de orden superior respecto a la distancia de primer orden entre estas extremales.
2. Condiciones suficientes de legendre. Condicin suriciente para
que la extremal de la funcional
J [y {x)l ~

xi

JF (x, y, y') dx;

!J (xo) =!lo.

xo
se pueda incluir en un campo de extremales es que se cumpla la condi
cin reforzada de Legendre. Esta consiste en que Ja desigualdad
F 1111, >O

se cumpla en todos los puntos de la extrema! considerada (o sea, para


todos los x E [xo, x1 ]).
EJEMPLO 13. Consideremos Ja funcional
2

J[y(x)]=

y(O)=l,

(y''+y'")dx;

y(2)=5.

Sus extremales son las rectas y= C1 x


C2 La extremal buscada que
satisface las condiciones de frontera es la recta y = 2x
l.
En este caso F11 ,11 , = 12y' 1
2 y en todos Jos puntos de la extrema! y= 2x
1 tenemos F11 , 11 , = 50 >O. Se cumple la condicin
reforzada de Legendre y, por consiguiente, la extremal y = 2x
1
se puede inc.Julr en un campo de extremales.
Esto se ve tambin directamente. La extrema! y = 2x
1 queda
comprendida en la familia monoparamtrica de extremales y ""'
;:::: 2x
a. (<X es el parmetro) que forma un campo propio.
EJEMPLO u. Consideremos Ja runcional

J (Y (x}) =

(xlly'll

+ l 2yll)

dx;

-1

g(-1)=-1.

y(l)=l.

La ecuacin de Euler para esta funcional tiene la forma

x.'ly''

+ 2:ty' -

y su solucin general es
!1

C1x'

12y

+Cx
1

5 8.

CONDICIONES SUFICIENTES DE EXTREMO

91

La extrema! que satisface las condiciones de rrontera consi deradas es


y= x8.
No puede ser incluida en un campo. La nica familia monoparamtrica
de extremales que la contiene es y = ax'. Pero esta familia no cubre
el recinto que contiene el punto de abscisa x = O (porque las extremales
de esta familia no pasan por los puntos del eje Oy con ordenadas distintas de cero).
En este caso tenemos F'll''ll' = 2x2 y Ja condicin de Legendre no
se cumple para x = O.

Analizar la posibilidad de incluir la extremal en un


campo para las funcionales siguientes

i
1

142. J (y(x)]=

(y' 2 -yy' 3 )dx;

y(O)=O,

y(l)=O.

o
a

143. J[y(x)J=

Jy' dx;
3

y(O)=O,

y(a)=b>O.

o
:r1

144. J (y (x)I

=) n (y) V 1 +y'

2 dx;

Y (xo) =Yo

Y (x1) = Yt,

n (y) > O.

145. J[y(x)J= ~ (6y' 2 -y")dx;

y(O)=O,

y(a) =b,

b>O.

8. Condiciones suficientes de extremo de una funcional


Se considera el problema varlaclonal elemental, o sea, se considera

la funcional

:t:t

J [y (x)) =

F (x, y, y') dx

(1)

:ro

con las condiciones de frontera


{2)
Y (xe1) = Yo
11 (x1) = Yi
to. Condiciones suficientes de Welerstrass. Se denomina funcin
dt Weiers/ra.ss E (x, y, p, y') la funcin defnlda mediante la igualdad
E (x, y, p, y')= F (x, y, y') - F (x, y, p) - (y' - p) Fp (x, y, p)

donde p = p (x, y ) es la inclinacin. en el punto (x, y), del campo de


extremales del problema variaclonal (1) y (2).

92

CAP. 11. EXTREMO DE FUNCIONALES


CONDICIONES $UF ICIENH.S DE EXTREMO DflBIL.

La curva C realiza el extremo dbil de la funcional ( l) si;


1) la curoa C es una extremal de la f u11clonal ( 1) que satisface las
condiciones de frontera (2), o sea, es la solucin de la ecuactt& de Euler
para la funcional ( l) que satisface las condiciones {2);
2) {a extrema/ C puede ser incluida en un campo de extrema/es
(esto tendr lugar. en particular, si se cumple la condicin de Jacobi);
3) la fur1cin de \reierstrass E (x, y, p, y') conserva su signo en
todos fos puntos {X, y) prximos a la extrema[ C y para valores de y'
prximos a p (x, y). La funcional J Jy (x)) tendr mximo en C si E ~ O
y mnimo si E ~ O.
CONDICIONES SUFICIENTES DE EXTREMO FUERTE.

La curva C realiza el extremo fuerte de la fun cional (1) si:


1) la curva C es una extrema[ de la funcional (1) que satisface las
co11diciones de frontera (2);
extrema[ C puede ser incluida en un campo de exfremales,
funcin de Weierstrass E (x, y, p, y') conserva su sigtta en
todos los puntos (x, y) prximos a la extrema/ C y para valores cualesquiera
de y'. S E ~ O, se tendr mximo y, si E ~ O, se tendr mnimo.
2) la
3) la

OBSE'RVAClN. La condicin de Weierstrass es necesara para que


exista el extremo en el sentido siguente: si para ciertos valores de y'
la funcin E tiene signos opuestos en los puntos de la extremal, no se
alcanza el extremo fuerte si esto ocurre para cualesquiera valores de
y' por prximos que sean a p, tampoco se alcanza el extremo dbil.
EJEMPLO 1. Analizar el extremo de la funcional

Jy(x)J =

J(y'+y')dx;

y(0)=0,

y(l)=2.

o
s0L u c 1~. La ecuacin de Euler para la funcional considerada tiene
la forma y' y" = O de modo que las extremales son las rectas y =
= ;G.i_;f_ C,. La extrema! que satisface las condiciones de frontera
Ta recta y= 2x. La inclinacin del campo en los puntos de esta
t~!remal es p = 2. Es evidente que la extrema\ y = 2x se puede
inclu ir en el campo central de. extremales y = Cx con centro en el
punto O (O, O). Es fcil probar as mismo que en este caso se cumple la

es

condicin de J acobi. La ecuacin de J acobi es en este caso -

:X (6y' u')=

O, donde, debido a la ecuacin de la ex trema!, hay que poner y' = 2.


Por consiguiente. la ecuacin de Jacobi toma la forma u"= O de modo
que u = C1x
C2 De la condicin u (O} = O obtenemos C2 = O.
Puesto que para C1 =i!:: O esta solucin u = C1 x no se anula en ningn
punto a excepcin del punto x = O, la condicin de J acobi se cumple.
Formamos la funcin de Weierstrass

~ (x,

y.

p,

t) =

y''+ y' -

p3

p -

(J/ -

p) (3p~

= (t -

1)

p} (V'

+ 2p).

93

CO Nbl lO Nl:.S SUF ICIE NTES DE EXTREM

El primer facto r es no nega tivo cualesquiera que sean los valores de

y' y el segundo es positivo para valores de y' prximos a 2. Por con-

siguiente, se cumpkn todas las condiciones de mnimo ddl il. Adems,


es fcil ver que para y' < -4 la funcin E ser negativa y no s e cum
plir la condicin suficiente de e xtremo fuerte ya que para el extremo
fuerte se ex ige que la funcin de Weierstrass E conserve.> su signo para
valores cualesquiera de y' . Si se
tiene en cuenta la observacin de
v
la pg. 92, se puede llegar a la con
clusin de que en este caso no hay
extremo fu erte.
EJEMPLO 2. Analizar el extremo de la funcional
l

J(g (x)]=

J(x+2y + ~ y'2)

dx;

o
y(O)=O,

y(l)=O.

SOLVClN.
La ecuacin de
Euler paTa esta fu ncional tiene la -0-4------~------/(
for ma y" = 2. Las extremales son
C1.x
C2
las parbolas y = x2
La extrema\ que sa tisface las
condiciones de frontera es y =
= x 2 - x. Formamos la ecuacin

de Jacobi

O.

-:x

u' =- O,

sea,

sol ucin general es


C2
La condicin
u (0) = O da C2 = O y la cond~
cin Jacobi se cumple ya que
u = C1 x con C1 -:fo O no se anula
en ningn punto del segmen to (O, 1]
a excepcin del punto x = O; es deFig . 11
cr, la ex trema! y = x2 - x se
puede inclu ir en un campo central
de extremales con cen tro en el punto O (0, O), a sal>er, en el campo
y = x2
Cx (ig. 11). La funcin de We iers trass tiene 1a fo rma

u = C 1x

Su

E (x, y, p , y') =

=~

(y' - p) 2

(y' -

p)1 . Se puede ver

de aqu l que E=

O para cualesquiera v alores de y' Por consiguiente,

la funciona l considerada alcanza en la extre ma! y -

ruerte igual a J

(x11 -

x]

~ ~

x2

x mnimo

CAi> . 11. EXTREMO DE FUNCINLt:S

AnaJizar eJ extremo de las funcionales siguientes.


1

146. J[g(x)] =

(u2+;y'2)dx; y(O)=l, y(l)=e.

o
1

147. J (y (x)] =

Jo eVy'2

dx; y(O)=O, y(l) = ln4.

148. J [y\x)J =

J;,

dx; y(l)= 1, y(2)=4.

i
Cl

149. J [y(x) )=

J:~;

y(O)=O, y(a)=b, b>O.

o
1

150. J (y(x)]=

J(l+x)y' dx;
2

y(O)=O, y(l)=l.

o
n/2

151 . J[y(x)l=

J(y2-!/ )dx;
2

y(O)=l, y

( ~)=L

o
2

152. J(y(x)J=

y'(l+x2g')dx; y( -1 )== 1, y(2)=4.

-1
t

153. J [y (x)l =

J(y'+ y') dx~ y(- 1) = -1, y(l) = 3.


-1

20. Condiciones sufici entes de Legendre. Supongamos que la


derivada parcial F'J'IJ' (x, y, y') de la funcin F (x, y, y') es continua
y que la extrema! C est incluida en un campo de extremales.
SI en la extrema/ C se tiene F1111 , > O, en la curva C se alcanza
mlnimo dlbll; si en la extrema/ C se tiene F11 11 <O, en ella se alcanza
el mximo d~bil de la funcional (!). Estas condiciones se llaman oondi

ciones reforzadas de Legendre.


En el caso en que F'll' IJ' (x, y, y') ;;:;;:. O en todos los puntos (x, y)
prximos a la extrema/ C y para cualesquiera valores de y', se tiene mrzi
mo fuerte y en el caso en que F11 (x, y, g') ~ O para estos valores de
los argumentos, se fierie mximo uerte.

.Y,

t 8.

ONDJCIONES SUPICIENtM DE EXtREMO

9$

EJEMPO 3: Analizar el extremo de la funcional


1

J(y (x}l=

J(y'l-(J.y')dx;

g(0)=0,

y(l)=-2

o
(a. es un nmero real cualquiera).
SOLUCiN. Puesto que la (uncin integrando depende slo de y',

las extremales son las rectas y = C1 x


C9 La extrema! que satisface las condiciones de frontera es la recta y= -2x que se puede incluir
en el campo central de extremales y= Cx. La inclinacin del campo
en esta extrema! es p = -2 . A continuacin, calculamos F11 , 11 , = 6y'.
En la extremal tenemos F'll'U' = -12 <O, o sea, la funcional alcanza
mximo dbil en la linea y= -2x. Para valores arbitrarios de y'
el signo de F1111, no se conserva y, por consiguiente, no se cumplen las
condiciones suficientes de mximo fuerte.
En este caso la funcin de Welerstrass E (x, y, p, y') tiene Ja forma

E (x, y, p, y'} = (y' -

p)~ (y'

+ 2p)

y para determinados valores de y' tiene signos o puestos. Teniendo en


cuenta Ja observacin de Ja pg. 92, deducimos que no hay mximo
fuerte.
EJEMPLO 4. Analizar el extremo de la funcional
2

lu (x}] = ~

(e?'' +3) dx;

y {O)= O,

g (2) = l.

o
SOLUCJON . Las extremales son las rectas y=C1 x+Ci. La extrema! que satisface las condiciones de frontera es Ja recta y= ~ ; puede

ser incluida en el campo central de extremales y = Cx. Tenemos, ade


ms,. F 11 , 11, (x, y, y') = e'I' > O para cualesquiera valores de y' . Por

consiguiente, Ja funcional tiene mnimo fuerte en la extrema! y=~.

vr

EJEMPLO 5, Analizar el extremo de la funcional

J(y (x)}=

Jo

dx;

y(O)=O,

y(a)=Yt

!I

SOLUCIN. La funcin integrando no depende expllcitamente de


x y, por eso, tenernos F - y' Fu' = C1 o, en nuestro caso,

y '2

CAP . 11. EXTREMO OB FUNCIONALES

de rlonde
1
.......,.=-...,...,,===
lfylfl+y' Z

1
donde Cs = ( ~

~1

-C
1

y ( 1r y'2>,._. c.,

)2. Pongamos y' = ctg

t
2 . Tendre mos y =

Ciseni

(1-cost), Adems,

dx = ......!!J!.._= Cs sen t dt

ctg2

C1seni..!..dt

2ctg 2

e, integrando, encontramos
- C

X-

Tenemos pues

( 1- cost)dt

(e = ~I

C1

=T(t + senl) +

x =e (f - sen t) + C2
y=e (1-cos t),

o sea, las ecuaciones paramtricas de una familia dt> cicloides. De la


2trR
J(

Fig. 12
condicin y (O)

O encontramos Ci = O. El haz de

x = e (t-sen t),

u= e <1-

icloides

cos t)
forma un campo central con centro en el punto O (O, O) que comprende
la extrema!
x= R (t-sen t), }
y :=.R(l-cost),
donde R se determina de la condicin de que la cicloide pase por el
segundo pu nto frontera B (a, y 1), s a < 2nR (fig. 12).

CONDICIONES SUFICIENTES DE EXTREMO

Empleamos la

~ondici n

F11'111

97

de Legendre. Tenemos

3 >O

VY.o +u'

para cualesquiera valores de y'. Es decir, para a


considerada tiene mnimo fuerte en la cicloide

< 2nR

la funcional

x=R(t-sent), }

u=R (1-cos t).

Empleando la condicin de Legendre, analizar el extremo


de las funcionales siguientes:
1

154. J[y(x)]=S(y' 2 +x2 )dx;


o

y(0}=-1,

y( l) =l.

155. J (y (x)J =

J x~ dx;

y (2) = 4,

y (3) = 9.

2 y
2

156. J{y(x))=

5(xy'.,-2yy' )dx;
8

y( l) =O, y(2) = l.

1
a

157. Jly(x)) =

J(J-e - lf')dx ; y(O) = O,

y(a)=b.

o
1

158. J(y(x)]=) yy' 2 dx; y(O)=p>O, y(l)=q>O.


o
159. Analizar el extremo de la funcional
l

J [y(x)}=) (ey' 2 +y2+x2)dx;


o

para dis tintos valores del parmetro

y(O)=O,

y(l)= J;

t:..

EJEMPLO 6 (problema de .Euler). Una barra vertical de longitud l


se somete a una carga axial P. Para un valor determinado de P (fuerza
crtica de Euler) la barra se comba. Se pide determinar el valor mnimo
de la fuer za P que provoca la ftexl6n longitudinal.
SOLUCIN. Sea E el mdulo de elasticidad, sea I el momento de
inercia mnimo de las secciones transversales de la barra, se p el radi
de curvatura y sea cp .el ngulo entre la tangente y el eje.
1-01387

9R

CAP. 11. EXTREMO DE FUNCIONALES

La ener:a potencial de la flexin se determina mediante la

f rmula

El J-p2
l

Ut=2

dS

o
Si el ex tremo de la barra desciende en
1

a=

Jo

(1-cos cp) dS,

la energa potencinl cte la barra disminuye en


l

llz= Pa = Pl-P) coscpdS.

o
Si antes de la delormaciu la energa potencial es igual a cero,
despuP.s de la delormacin estar dada por la frmula
l

U=U 1 -U,.=

Jo (~l p~+P<;oscp)dS-Pl.

dS

Puesto que p = dq> y (para pequeos valores de q>) cos <> ~

cp
1-T,
9

se tiene
1

U=

t o

[El (

~;

) -

Pipll] dS ~ ~

Jo

[El (

~~ )

2
-

pq2] dx.

En el caso de equilibrio, la energa. potencial toma su valor mini


mo. Por eso . el problema se reduce a la determinacin del mlnimo de
la integral

En este caso
F=EI (

!~) -Pe>~
2

y la ecuacin de Euler tiene la forma

q>"

+aiq = O,

donde el'=

:r .

La !\Olucin general de esta ecuacin es


e> = C1 sen cu
C2 cos cu.

f&.

CONDICTONES SUPICIE:NTES DE EX1REMO

99

Puesto que tg q> ~ q> para valores pequefios de q> y como, adems,
tg q> = y', se tiene

y' = C1 sen ax+ C1 cos ax,


de donde
C, cosax+C2 senax C.
a
a.
+

11=

Si el extremo inferior de la barra est en el origen de coordenadas,


ser y = o para X=
o sea, C1 =
O:

e=
e
y= _z. sen<u.
cr.

Veamos si se cumplen las condiciones de Legendre y de J acobl.


Es obvio que la condicin de Legendre se c umple :
(}Zf

oq>'

= 2El >O.

La ecuacin de J acobl tiene la forma


E 1z
Pz = O z"
a'z = O
con la particularidad de que z (O) = O. Por eso, la solucin de la
ecuacin de J acobi ser
z = A sen ax.

La funcin z se anula para

x,. =..!!!:_
(k =
a.

condicin de J acobi se cumplir si l

1, 2, . . . ) de modo que la

~ ..!!..
a . De aqu

ni

P4:---El.
El valor mlnimo de la fuerui critica de Euler ser
ns
Pmtn=""j2EI
y la ecuacin de la curva de ilexin ser

nx

Cz.

y=-;:;:-sen - 1- .
3. figuratrl2. Sea dada la funcional
b

IY (x)J=

F (x, y, y') dx.

Tomando x e y como parmetros, consideremos la funcin y =

F (x, y, y') en tanto que funcin del argumento y'. El ~rfico de


esta funcin en el pla no de las vi1riables (y', Y) se denomina flguratriz.
7

100

CAP. 1i. EXTREMO DE FUNClNA LES

Es fcil probar que la funcin de Weierstrass E (x, y, p, y') representa


la dilerenca entre las ordenadas de la figuratriz y las ordenadas de la
tangente a la liguratriz trazada por el punto de abscisa y' = p. Si la
funcin de Weerstrass conserva su signo para ciertos valores de y',
ello significa que la figuratriz est por encima o por debajo de la
tangente para esos valores de y' . En este caso hay mfnimo dbil. Si
la figuratriz est a un lado de la tangente para todos los valores de
y' y para los valores de los parmetros x e y prximos a los puntos de
la extrema!. hay extremo fuerte.
La condicin suficiente de Legendre se expresa en estos trminos
as: si para todos los puntos (x, y) prximos a la extremal la ftguratriz
es cncava llacia las Y positivas o negativas, hay extremo fuerte.
EJEMPLO 1. Analizar el extremo de Ja funcional
(l

J(y(x))=

y'"'dx;

y(0)=0,

y(a)=b,

b>O.

SOLt:CiN- Las extremales son las rectas y= C1 x


C2 La extrema! buscada viene dada por la ecuacin y = ~ x. Puede ser i!'duida

Fig. 13
en un campo central de cxtremales. La figuratriz es la parbola
Y = y'm (f ll- 13). Es fcil ver que toda la figuratrz est por encima

s 8.

101

CONDICIONES SUFICI ENTES DE E XT RE MO

de Ja tangente trazada a la misma en el punto p =

.!!.a cualesquiera

que sean a y b (a * O) . Por consiguiente, la unclonal considerada tiene


mnimo fu erte en la extremal y
EJEMPLO

= ~ x.

s. Analizar el extremo de la funcional

Io
o

JJ y (x)l=

y'Sdx;

y (O)=O,

y(a}= b,

b >O .
h

La extrema! buscada es la recta y = - x que se puede


a
incluir en el campo central de extre males y = Cx con centro en el
SOLUCIN.

Fig. 14

punto O (O, O). La fu guratriz. es la parbola cbica Y = y' 3 (fig. 14).


Para valores de y' suficientemente prximos al valor p =

.!.a la

ratriz est sobre la tangente a la misma en el punto de abscisa y'

igu-

= ~
a

De la lig. 14 se puede ver que la figuratrlz corta Ja tangente en el punto

de abscisa y'

= -

2
b y a la izquierda e este punto aparece por debajo
a

C:AP. JI . f.XTRf.MO J>E PUNCION ALL: s

102

de la tangente. Por" lo tanto. hay mnimo dbil en la t>xlremal


b

= - x.
a

Ntese que para p = O (esto corresponde al caso b "" O en el que


Ja extremal es un segmento del eje Ox) la tangente a la figuratriz es
el eje Oy' y el punto O (O, O) es un punto de inflexin de la figuratriz.
Teniendo en cuenta la observacin de la pg. 92, vemos que en cualquier vecindad del punto O (O, O), por pequea que sea, la figuratrz
tiene ordenadas tanto positivas como negativas. Por lo tanto, la
funcin de Weierstrass E tiene s ignos opuestos para valores de y'
tan prximos a p = O como se quiera y, por consiguiente, en esto caso
no se alcanza ni siquiera el extremo dbil.
EJEMPLO 9 . Probar qui! la extrema! y = O del problema variaciona l
1

J\y(xiJ=) (y'i - yy'3)dx;

y (O)=y ( l) = O;

o
realiza el mnimo dbil de la funcional.
SOLUCION. En este caso la condicin de Legcntlre da

F1111 , 111 -o=(2-6yy') 111 ... 0 -=2>0.


o sea, se alcanza mnimo M bil en la extrema! y = O. Demostremos
1.ue en esta extrema) no se alcanza el mnimo fuerte. Consideremos la
y

y'

Fig. 15
figuratriz Y = y' 2 - yy'' para los valores y > O (fig. 15). De la fig. 15
se ve que la tangente a la figuratriz en el punto de abscis a p = O corta
la figu ratriz en el punto y' =
con y

> O,

~.
y

Es decir, para los puntos (x, y),

prximos a los puntos de la extrema! y= O, la funcin de

Weierstrass E es positiva para valores de y' menores que

~ y es negati!/

8.

!03

CONDIC IO NES !'UPICJENTES D E fX TIHoMO

va para y' > 2.. Segn la observacin de la pg. 92, no hay mlnlmo
y
fuerte. Una situci n semejante se tiene tambin para y < O.
Lo que destaca este ejemplo es que en l la condicin F 11 , 11 , > O
se cumple en la extrema! para cualesquiera y' y, sin embargo, ello
no implica la existencia de extremo fuerte.

Empleando la figuratriz analizar el extremo de las fun cionales siguientes:


1

J
o
Jly(x)] = Jy'(l+x
_,

160. J[y(x )] =

(l+x)y'Zdx;

y(O) = O,

y(l)=-2.

161.

2y')dx;

y(- l) = y(2)= 1.

162. J[y(x))=

J(l-e - 11' )dx;


4

o
y(O)=O,

y (a )=b,

(b>O).

163. J (y (x)J =

J(6y' -Jl' +yf/)dx;


2

y(O)=O,

y(a) = b

(b>O).

ORSERVAClN. La no negatividad de la segunda variacin es con


dicin necesaria, pero no s uficiente, para que la funcional J [y (x))
al cance mn imo en la \: urva.
t:JEMPLO 1o. Consideremos la funcional
1

J[g (x)l =

Jyi

(x-y) dx

o
en el espacio C [O, J} . La ecuacin de Euler tiene la forma F 11 = O
/1 = O. Para O~ x ~ 1 la segunda variacin de l a funci onal en la
extrema\ y = o
l

{J2J

(O, 6y1 =

x (y)s dx

o
es positiva para todo ~!I =6 O. Sin embargo, en cual quier vecindad del
cero la funcional J [y (x)l toma tambin valores negativos; basta fijar

l04
e

<.:AP. ll. EXTREMO DE FUNCIO NA LES

>Oy

considerar la funcin

!Jg (x) =

-x+e,

Entonces tendremos J f!le (x)] = -

O,

e'

< O para cualquier e > O.

DEFINICJN. Se dice que la funcional cuadrtica L 9 (h] definida en


un espacio normado es fuertemente posi/lva si existe una constante

>O

tal que

para todo h.
CONDICIN SUF ICIE NTE DE MtNIMO. Condicin suficiente para que
la funcional J ly (x)) definida en un espacio normado tenga mnimo
en el punto estacionario y = !lo es que su segunda variacin sea fuerte
~nte positiva en y = y 0 , o sea, que se cumpla la condicin
fJ'J !yo. fJy) ~ k 11 fJy ue.

donde k = const, k > O.


4. Supongamos que se busca el extremo de la funcional
%i

J [!ltt IJz, .. , Yn]=

F (x, !11t !IZ .. ., Yn

y;_, y;, ... ,

y;.) dx,

(3)

%Q

q ue depende de n fun ciones y 1 (x), Yt (x), .. ., !In (x), con las condi
ciones de frontera
Yk (xo)

= IJkO

!11< (x1) = Yin

(k

l , 2, ... , n).

La condicin refonada de Legendre consiste en que las desigualdades

F1111
1 n
F1111
1

>0

{4)

F11'11'
n 1 F1111
n ll F,,.,,.
n 11
se cumplan en todos los puntos de la extrema! considerada de la fun
cional (3).
La condlc1on reforzada de Jacobi consiste en que el segmento
f.i0 .i1J no contenga punto conjugado del punto x0 .

f 8.

CONDICIONES SUFICIENTES DE EXTREMO

!05

La condicin reforzada de Legendre (4) conjuntamente con la


condicin reforzada de Jacobl garantizan por lo menos la existencia
del mlnlmo dbil de la funcion al (3).
EJEMPLO 11. Analizar el extremo de la funcional
1

J [.!/ (x), z (x)] =


y (0) =O,
y(l)= l ,

(y'+2' 2) dx;

o
z (0) =

O, }

(5)

(6)

z ( l )=2.

SOLUCION. Las ecuaciones de Euler para la funcional (5) son


y*

O,

O,

de modo que
y=C1 +C2x. }

(7)

z=Cd C,x.
Empl eando 1as con diciones {6}, obtenemos
C1
C1 =O,
La extrema) buscada

I,

C3

=O

Ct. = 2.

y=x, }
Z=2X

(8)

representa una recta que pasa por el origen de coordenadas.


Tenemos
Fy y = 2,
Fy'z' = O,
F z'y' = O
y
F Z'%' = 2.
La condicin reforzada de Legendre se cumple:
F11"11'=2>0.

' F:r.'y'
Fy'' F11~1=12
1=4 >0.
F%'%'
1O 2

Veamos si se cumple la condicin reforzada de Jacobi.


Una de las defin iciones de punto conjugado es la siguiente (vase

[3)).

Supongamos que se tiene una fa milia de ex tremales de la funcional (3) que arrancan del punto inicial (x 0 , y 10 , . . , .!/no) en direcciones prximas r ero linealmente independientes. Se dice que el
punto x E [x0 , x1 es conjugado del punto x 0 si existe una sucesin
de extremales, que arrancan todas del punto inicial y que son t an
prximas como se quiera a la ex tremal considerada, tal que cada una
Cle estas extremates corta la extrema! considerada con la particu laridad de que las abscisas de los puntos de Interseccin convergen haci a
el punto x.
En nuestro caso las extremales son las rectas (7). Todas las extremates que arrancan del punto (O, O, O) cortan la ex trema! (8) en t>ste

C.'\I' , 11. EXTnEMO DE FUNCIO ../.l\LES

106

punto ~olaml'nte P or lo tan to, el segmento (O 1) de varacin de x


no contiene punto conjugado del punto x0 = C. Es decir, se cumple
tanto la condicin reforzada de Legendre como la condicin reforzada
de Jacob de modo que la extremal (8} realiza el minimo dbil de la
funcional (5).

Analizar el extremo de las funcionah.!S siguientes:


t

164. J[y(x).z(x)I=

j v1 +y

12

+z' 2 dx ;

o
y(O) .,., o, y(l) = 2, z(O)=, z{1)=4.
1

165. J IY (x), z (x)l = ) (y' 2 +z' 2 + 4z) dx;

o
y(O)=O, y(I)= l, z(O) = O, z(l) = O.

9. Extremo condicionado
l. Problema isoperimtrico. Sean F (x, y, y') y G (x, y, y') dos
funciones.
El problema isopermtrico consiste en Jo siguiente.: entre todas
las curvas y= y (x) E C1 [x 0 , x1] a lo largo de las cuales la funcional
:lei

K[y(;c;)J=

G(.1; 1 y, y1 )dx

llene un valor fij o ! hallar la curva en la que la funcional


:11:1

J fy (x)i =

F (x, y, y') dx

xo

alcanza su valor ex tremo.


Suponemos que las fu nciones F y G tienen derivadas parciales
continuas dr primer y de segundo rdenes para x 0 ~ x ~ x1 y para
valores cualesquiera de las variables y e y' .
TEOREMA DE EULER . Si la curva y = y (x) realiza el extremo de la
funcional
XI

J IY (x) I-= } F \X, y,


Xo

y'> dx

p.

EXT REMO CONDICIONADO

107

con tas condiciones


Xt

G(x, y, y')dx=l, y(xo)=Yo y(x1}=Yt


:ro
y si g = y (x) no es extremal de ta funcional K existe una constante f.
tal que la curva y = y (x) es e;ctremal de Ja funcional
K[y(x)l=

XI

L (y (x))=

[F (x, y, y') +W (x, y, y')] dx.

:ro
EJEMPLO 1 (problema de Dido). Entre todas las curvas cerradas
de longitud 21 hallar la curva que comprende el rea mxima.
SOLUCIN. Observemo.s , ante todo, que dicha curva debe ser con
vexa. Efectivamente, de lo contrario encontraramos una recta l

Fig. 16
(fig. 16) tal que, al reflejar en ella la parte BCD de la rontera, obtendramos un re.cinto de rea mayor que el inicial siendo la longitud
de la rontera la misma que antes.
ObseJve mos tambin que toda recta que divide por la mitad la
cerrada que comprende el rea mxima ha de dividir por la mitad la
propia rea. Efectivamente, supongamos lo contrario y sea L 1 una recta
que no cumple esta propiedad. Reflejando en l 1 Ja parte de la figura
de rea mayor, obtendremos una curva de idntica longitud pero que
comprende un rea mayor.
Tomemos como eje Ox cualquiera de las rectas que dividen por
la mitad la curva; llegamos entonces al problema siguiente.
Hallar la lnea y = y (x), IJ (-a) = y (a) = O, de longitud fija
l > 2a que conjuntamente con el segmento -a~ x ~ a del eje O:c
encierre el rea mxima. Por lo tanto, el problema $e reduce a hallar
el extremo de la funcional
(1

J[y(x))=

Jydx;
-a

y(-a)=y(a) = O;

108

CAP. 11 . EXTREMO DE Pl:INCIONALES

con la condicin complementarla de que


a

K (y (x)) =

JVl+y''dx= l

(l>2a).

(1)

-a

F ormemos la funcin auxiliar

H =F+W= y(x}+'.).. lfl +y'2(x}

y consi deremos l a [uncional au xilia r


a

l IY (x)) =

JH

(x, y, y') dx.

(2)

-a

La ecuacin d e Euler para la funci onal (2) tiene la for ma


-d (
dx

')..y'

V1 +u' -- 1,

de donde
'J...y'

Vi + u'
Resolviendo la lti ma ecuacin respecto a y', encontramos
dy

di =

x+C1

(3)

V h.~-(x+ C1)i .

Integrando la ecuacin (3) , obtenemos


(x+C1)i+ (y+ c 2)2=J..2,
o sea, la circunle.rencia de radio~ con centro en el punto (-C1 , -C1 ) .
Las constantes C1 y e~ as como el parmetro A. se determinan de las
condiciones de frontera y (-a) = y (a) = O y de la condicin isoperi
mtrica (1). T enemos
C~ =A.2 - (C 1 -a)2, }

C! =V-(C1+a)2,
de donde
de modo que

Y = V~ -v~

y' = - --====V !.2-xZ

~XtlUiM CNOICIONADO

109

La condicin (1) da entonces


o

l=

1v

').

dx

)..2. -

xz

=A arcsen-;-

x-a

,.. x--ci

= 2J..arcsen-:,..

-o

o sea,

-,;=sen 21..
Resolviendo respecto a A. esta ecuacin trascendente, encontramos un
valor determinado A.= A.0 y despus encontramos el valor de C2 =
= y).g - '
E.s fcil persuadirse de que Ja ecuacin ~ = sen ;').. tiene siempre solucin. Efectivamente, tomando

~J.. = t,

esta ecuacin quedar

t, donde ~ =<I por hiptesis del problema.


La inclinacin de la tangente a la funcin y = sen ten el punto t = O
reducida a sen t

es

= ~a

Z mientras que la inclinacin de la funcin y =a.I es menor. Por

consiguiente, los grficos de estas funciones tienen un punto de inter.


seccin como mlnimo, a parte del punto O (O, O).
PRlNCJPlO DE RECIPROCIDAD EN EL PROBLEMA ISOPEJUM~TRI CO.

Las extrema les de la funcional


J

IY (x)I = "' F (x,

y, y') dx

con la condicin complenuntarla


:Xt

K l.Y (x)]= ) a (x, y, y') dx= consl


~

colnclden con las extremales de la funcional K. [y (x)I con la condicin


J (y (.x)] = const.

Basndonos en el principio de reciprocidad, deducimos del proble


ma de Dido el resultado siguiente: entre todas las curoas cerradas que
comprenden un drea fiia, la circunferencia es la curva de longitud
mlnima.

Es cil obtener este resultado directamente si se recurre a, la


forma paramtric.a de\ problema variacional.
Sean
x=x{t), x(to)=x(t1).}
!I =Y (t), Y (to) =y {t),

t 0 ~t ~ t 1,

110

CAP. 11. EXTREMO

oe

FUNCIONALES

las ecuaciones de una curva cerrada. El problema consiste en hallar


el extremo de la funcional
1

f (~2+y2)I
con la condicin

dt

J(xy-y~) dx = C.

Introduciendo la funcin
1

F=(~2+yz)2+

>..

(xy- y~),

encontramos (vase Ja pg. 68} que la curvatura

de Ja curva que

realiza el extremo es constante:

_!.=l..
r

Por consiguiente, la extrema) buscada es una circunferencia.


Utilizando el principio de reciprocidad, se pueden resol ver, sin
realizar clculos, a l ~unos problemas cvarlacionales de la Geometria
elemental.
EJEMPLO 2. Demostrar que: 1) entre todos los tringulos de base
y permetro fijos, el de rea mxima es el tringulo issceles; 2) siendo
fijas el rea y la base, el tringulo issceles es el 'tringulo de permetro mlnimo.
SOLUCIN. 1) Tomemos una elipse cuyos focos son los extremos
de la base de los tringulos considerados (fig. 17). De la propiedad
y

c0 (o,b)

Fig. 17

de la elipse deducimos que lodos los tringulos ACB tienen el mismo


permetro. Es evidente que el rea mxima corresponder al tringulo

EXTREMO COND ICION1\DO

111

de altura max1ma lo que s ignifica q ue el ver tke del t ring ulo debe
coinc idi r con el vrt ice C0 de la elipse. El tringul o AC0 B es Issceles.
2) Segn el princpio de reciprocidad, siendo Jijas el rea y la
base, el per met ro m n imo cor respo nde al tringulo issceles.
EJEMPLO J. H allar el mni mo de la integral

J [y(x)) =

"

y'2dx

lt

con las condic iones

y2dx= 1, y (O)=y ln).=0.

SOL.UClN. For memos la funcional auxiliar

l [y (x)J =

J"'

(y'2 + J..y2)

ru:

o
f

consideremos su ecuacin de Euler

ZA.y -

:x

(2y') =0,

o sea.

Y~ - 'A.y = O.

(4)

lfl.
+

Su ecuacin ca racterstica es r 2 - A. = O, de don de r 1 , 2 =


Est claro que J. de be ser menor que cero: s i 11ct.>ptamos que J. > O,
l a solucin general de l a ecuacin (4) tend r la for ma y = C 1eV~x

+ c,e- V;:x, las condiciones de frontera y (O) =


rn slo p ara C1

O, C2

en este caso la co nd icin

O, o sea, resulta r y

11

Jy

dx

y (n)
o se cumpli=
O y no se c umplir
=

1; de la mis ma for ma, s i J..= O,

la solucin de la ecuacin de Euler (4) q ue sa t isfa ce las condiciones

O. Por eso, cons ide ramos que


de frontera t a mbin ser la fu ncin y
A. <o de modo que ' 1.a = i
y l a soluci n genera l de la
ecuacin (4) es y = C 1 sen
- 1..x
C2 cos
'A.x. La condicin
y (0) = O da C1 = O y la condicin y (n) = O da - /.. = k1
(k = \ , 2, . .. ). Es decir, y = C1 sen kx, don de C1 no se ha deler mi-

v =x1

1f

nado a n. Utilizando la con di cin

y 2 dx

o
1'I

JCf scn2kx dx=


o

1,

I, obtenemos

CAP. 11 . EXTREMO DE flU NClO NALES

112

V! .O sea, V! senkx. Todas las extremales


V! senkx por los puntos y pero
slo para dos, a saber, y=V! sen x, se cumple la condicin
de donde C 1 =

11==

11=

pasan

(O, O)

(Jt, O),

de Jecobl. En estas dos extremales se tiene


J ly (x)I =

y'Zdx=

1-J "'i"cos2xdx=I.
2

o
EJEMPLO 4

(problema de Kelvin . Supongamos que en el plano

xOy est distr ibui da una masa de densidad continua {x, y) y supon

gamos que se tiene en el plano una curva C suave a trozos y dos puntos
P 1 y P, sobre la misma . En tre todas 1as curvas de longitud ja l
que unen Jos puntos P 1 y Pz hallar la curva que conjuntamente con
el arco P1 P2 de la curva C forme un recinto D de ma!>a mxima. Los
puntos P 1 y P.1 pue den coincidir.
SOLUClN. ~nsiderem os Ja funcin
V (x, y)=

{x, y) dx.

Segn la frmula de Green, tenernos

JJtx.

y)dxdy=

JJ ~~

dxd!J """' f Vdy,

donde el contorno f i.e compone de la curva L y de la parte P1 P 2


de la curva C. A lo largo de esta ltimu parle la integral toma un
valor determinado que designaremos por K. Aceptando que la curva
L est dada en forma paramtrica

x=x (t),

Y= y(t),

lo< t ~ ttt

tendremos entonces
t

JJ

(x, y)dxdy =

V(x, y)ydi+K.

t~

Por .;onsiguiente, el problema ha quedado reducido a la determi

nac in del mximo de la funcion al


tt

Je,= ) V (x,
ti

y)

ydt

s 9.

EXTREMO CONDI CIONADO

113

con la condicin de que


tt

~ Yxz+Ji2dt=1.

Consideremos la funcin auxil iar

F=Vy+X V~z +yz


y empleemos la forma de Weierstrass de la ecuacin de Euler. Tenemos

av

F = iJx '

F = O,

XII

yx

F..
.
F, =---==-=--- 3

y2

2
(x2 + y2)

de modo que la ecuacin de Euler en la forma de Weierstrass <:i.

av

7=-;a;0,

rerordando la expresin de la funcin V (x, gi.


1

7=

(x,

y)

A.

donde r es el radio de curvatura de la curva pedida.


En el caso en que 1-l (x, y) = const resulta que la curvatura de
la curva pedida es constante y, por consiguiente, las extremales son
circunferencias. Queda claro que realizan el mximo de la funciona l
JL.
Tambin se denominan problemas isoferimtricos los problemas
variacionales en los que se pide hall ar e extremo de la fu ncional
Xt

I (Y11

!/21 . ,

YnJ=

JF (x, Yt. yz, ., Yn y\, Y

. . . , Y) dx

(5)

:ico

con las asl llamadas condiciones isoperimtr icas


Xl

G (x, !11 !Jz, . , Y n

y~.

Yt .. .,

y~)dX=l

t6)

xo
(i= 1, 2, . .. , m),

donde 11 son unas consta ntes.


Para obtener la condicin necesaria fundamental en el problema
soperimlrico sobre la determinacin del extremo de la funcional (5}
8 - 011187

CAP

114

11 . EXTREMO DE FUNCIONALES

COr1 las Condiciones (6) hay que formar Ja UOcionaJ auxiliar


Xt
m

cI> IYt !h. - .. , Ynl

1( + ~
F

xo

,G) dx,

i>=t

donde At son unas constantes y escribir sus ecuaciones de Euler. Las


constantes arbitrarlas C1 C2 , . , Cin de la solucin general del
sistema dt' ecuaciones de Euler as como las constantes A.1 , A.2 ,
. . . , Am se determi nan de las condiciones de frontera
!J1t, (xp) = Y1t,u .

!lk

(x, )

= Ykt

(k = 1. 2, ... , n)

y de las condiciones isoperi mtricas (6)


XI

~O;dx = lt

(i=l,2, ... ,m).

xo
EJEMPLO

s. Hallar la extrema! en el problema isoperimlrico

sobre el ex tremo de la runcional

I
1

J IY (x) , z ( x ) ) =

(y'2

+zi _

4xz' - 4z) cfx;

ll

y(0) =0,

z(O)=O,

z(J) = I;

y(l )= l,

con la condicin
1

(y'2 - x/-z'2) dx = 2.
u
SOLUC IN. Formame>s la funciona 1 auxiliar
1

<D fy (x), z (x)j =

J [y'2 + z 2- 4xz'-4z + J.. (y'2-xy' -z'2))


(1

y escribimos para ell11 el sistema de ecuaciones de Eu ler

d~

(2y' + 2Ay' -A.x) = O, }

_ 4 _ !!:_ (2z' - 4x-2A.z') = O

dx
r esolvie ndolo, encontrnrnos

'

dx

s 9.

EXT REMO CO NDICIONADO

115

Las condiciones de frontera dan


_311.-j-4.
Ct 2

Ca=2 (1 -A.}

C2= 0,

de modo que
.A.x2 + l3"'+ 4)x
}
4 (1 +"-)
,

y=

Z=X.

Para determinar l. recurrimos a la condicin isoperimtrica (7). Puesto


,
, 2"-x+3A.+4
que y = 4 (l +"-) y z = l, obtenemos
1

r [(2A.x+-3). + 4)2

16(l + A.)2

(21..x-f-3A. -\- 4) x
4 (1+1..)

1] d:c= 2,

de donde. despus de unos clculos sencillos pero voluminosos, obte


nemos pa ra A. la ecuacin sigu iente:

(23A.2-H6A.+24)=48(A.2+21.+ 1).

:~ y

De aqu resulta 1..1 = que A.1 =

- :~

A.2

~~

= -

satisface y 12 = -

!~

. Introd uciendo en (7), vemos

no satis face la condicin iso-

perimlrica.
La extrema! buscada se determina por las ecuaciont-s

y = 7x-;_5x2 , }

Z=x.
Hall ar las extremales en l ~ siguientes problemas isoperimtricos.
166. Entre todas las curvas planas <le longitud l con
extremos en los puntos fijos M 0 (x0 , Yo) y M 1 (xJt y 1 ) hallar
la curva con ordenada mnima del centro de gravedad (pro
blema sobre la forma de equilibrio que toma un cable pesado
homogneo por accin de la gravedad).
167. J{y(x)) =

.l' y'
o

condicin

J' y dx =
o

3.

dx;

y(O) = 1,

y(l) = 6; con

la

CA P. ll. EXTRE MO DE FUNCIONALES

116

l68. J [y(x)J=

J(x + y' )dx;


2

y(O) = O, y(l)=O; con

o
1

Ja condicin

io !f

dx = 2.
1

169. J[y(x)) =)

y' 2 dx;

y(O)=O,

y (l) =

! ; con

la

o
t

condicin

(y- y' 2) dx =

12

20. Tambin es un problema variacional de extremo condicionado


el problema de Lagrange en el que se pide ha llar el extremo de la
funcional J [y1 , y 2 , ., Yn1 con la particularidad de que se imponen
ciert as condcones de enl ace a las funciones de las cuales depende
la fu ncional J.
E l problema se plantea as. H all ar el extremo de la uncional
Xt

Jfy. YZ Ynl=

F(x, Y1 Y2 . ., Yn Yt y2 , .. ,

y~) dx;

(8)

:xo

YJ(Xo) =Y;o.

Yj (Xt ) =y

(j=I. 2, ... , rt);

con las condiciones

CJl1 (x, Y1. Y1. .. Yn)


(i

1, 2, .. ., m; m

(9)

< n)

que se consideran independ ien tes.


TEOREMA. Las furiciones y1 , y 2 , . . . . Yn que realizan el extremo
de la funcional (8) con las condiciones (9) satis/acen, siempre que los
factores '-t (x) (i = 1, 2, . . . , m) se escojan debidamente, las ecuaciones

de E uler de la funcional

Xi

J =

Pl

J[ f + ~ l.cp ]

xo

dx.

..,. 1

Para a breviar pondremos F

Y .. . ,

y~).

+ 2: A.tq =et> (x,


i=1

Ytt Y2 . Yn Y

Entonces las. funciones A.i (x) e y (x) se determinan de

s 9.
la~

EXTREMO CONDICIONADO

11 7

ecuaciones de Euler
<D'

llJ

-~ <11',=0
dx Vj

(J = l, 2, .. ., n)

y de las ecuaciones
q> (x , y1, Yi .. ., y,.)= O (i = I, 2 . . . , m).
Las ecuaciones q>1 = O tambin se pueden considerar como ecuac iones
de Euler para la funcional J"' si se acepta que los argumentos de esta
funcional son tanto las funciones y1 , !h. . ., !lri como las funcio nes
A (x), /..1 (x), .. . '-m (x).
EJl?MPLO 6. H allar la distancia mnima entre los puntos
A (1, -1, O) y B (2, 1, - 1) que pertenecen a la superficie 15x -

- 1y

+z-

22 =

o.

SOLUCIN . C-Omosesaben

la distancia entre dos puntos A (x 0, y 0 ; z0)


= O se determina por la

y 8 (x1 , Ytt z1 ) en la superficie q (x, y, z)

frmula

Xi

l=

Jy1 +u2+z'2dx,

= "

donde y = y (x), z
z (x).
Se pide, pues, hallar el minimo de l con la condicin q> (x, y, z) =
= O. En nuestro caso,

.to= 1,

XJ=2,

<p (x,y,z)= J5x-7y+z- 22.

Formamos la fu ncional auxiliar


2

JlYI +1/2+z'2+ )..(x)

(\5x- 7y +z-22)l dx

y escribimos pa.ra sta las ecuaciones de Eule r


). (x)( - 7)-

ddX (-.Vr 1 +u'Z+z'2


y'
)=

d {
). (x}-l --d-x

V l + z'y'2+z'2 ) =O.

(10)
( ll)

Resol vamos el sistema de ecuaciones (10) y (11) empleando la condi


cin de enlace
l 5x - 7y
z - 22 = O.
(12)
Las fun ciones incgnitas y= y (x) y z = z (x) satisfacen las s guientes
condiciones de fronter a:

!I (1) = - l,
!I (2) = 1,
t ( 1) =
z (2) = -1.
( 13)
Multiplicando por 7 la ecuacin (11) y agregndola a (1 O), obtenemos
~(
dx

y' -1- 7z'

V 1+u'i + ri

) _ 0

CAP. 11 . EXTREMO DE FUNCIONALES

11 8
de donde

11'+7z'

(14 )

yt+y'' +z2
De (12) te ne mos

z' = 7y' - 15.

(1 5)

In trodu ciendo este valor de 2' en (14) y resolviendo la ecuacin dife


rencial obtenida, encontramos y = C1x + Ci Las cond icio nes de
fr ontera (13) dan 1 = 2 y e~ = -3 de modo que
(1 6)
y= 2x -3.

Teniendo en cuenta (16) , de (15) resulta

l - X
(1 7)
(es obvio que la funcin (l 7) s.at isface las condiciones de frontera).
O. La distancia buscad a es
De (10) o de (11 ) obtenemos /,,
=

l=

Jlfl + y'' + z'2dx=\f6 .


1

Este resultado se puede obtener inmediatamente de consideraciones geomtricas eviden tes.

30, Lneas

geod~sicas.

Sea

(1 8)

r (u, v)

la ecuacin vectorial de una superficie.


Se llama lnea geodsica la linea de menor longitud que per tenece
a la superficie considerada y que une dos puntos ijos de la misma.
Las ecuacione!; de las lneas geodsicas se pueden ob tener como
l as ecuaciones de Euler correspondie ntes al p roblema v ariacional
sobre la d istancia minima en la s uperficie entre dos puntos fijos ..
Toda lnea perteneciente a la superficie r = r (u, v) se puede
representar por las ecuaciones paramtrcas
U =

U (/),

V _., V

(t).

La longitud de su parle comprendida entre los pu ntos corre5pondien tes


a los valores 10 y t 1 del parmetro t es igual a
ti

J[ u(l) , u(t )]=

J VEu'2+2Fu'u

-j- Gv'2 dt,

(1 9)

41
donde E, F y O son Jos coeficientes de la primera forma cuadrtica
de la superficie (18), o sea,

ar

or )

auTu.

ar Tv,
or )
au
1

=(.E!...
au , ~
av ).

Aqu (a, b) es el producto escalar de los vectores a y b.

5 9.

ll9

f. XTREMO CONDICI ON ADO

Para la funci o nal (19) el sistema de ecuaciones de Euler tiene


la forma

Euu' 3+ 2Fuu'v' + Guv' 3


d
Eu": + 2Fu'u' -', Gv'2 - dt

E11u' 2 -! 2Fvu'r.l+Gvtl 2 - ~
Eu':I+ 2Fu'u'-+- Gv'2
dt

2 (Eu' + Fv' )

v Eu'~ -- 2Fu'u' +Gv

= O
1

'

2(Fu'l Gu')
= O.
Eu'<i + 2Fu'v' + Gu'2

EJEMTLO 1. Entre todas las cur vas que es t n sobre una superficie
esfrka de rado R y que unen dos puntos f ijos de la misma. halla r la
curva de longitud mnima (la curva ~codsica) .
SOLUCJN . Sean qi y O las coordenadas del punto en l a esfer a
y sea cp ""~ <p (fl) la ecuaci n de la curva pedida. Tenemos entonces

= r (<p. 0) = x (<p. 0) l + g (<p. 6) j

+ z (q;. El} k ,

donde

R cos q sen 0, y ..,,. R sen


z = R cos 0.

Por t>so,
E = (r ip, r qi) = R.<J. sen 9 0; G = (re. r e)
De aqu. segn la frmula (19). tenemos
61

J[q(0)1 = R

e.

q; sen

= R. 2 ,

F = (re. rqi) = O.

61

Vd6z +sen 2

Jy1 +sen~Oq>' Z (6)d6 .

Udq': = R

El integrando no contiene la func in incgnita

qi

(O) y, por eso, la

ecuacin de E uler ser

donde

sent O<i' (A)

f IP' = -:-;====::;::===-

V 1+se ns 0<f"2 (0)

'

de modo que

sen2 O<p' ( 0)

v 1+ seas0q>'2 <>
de donde
<>' (!:l) =

C1
sen aysenz a - q

e,

t i

~~~-;:-::::::==:;::;::=-

sen2

1-q-

sen

0 V(t-CU-Cf

st>n20

Ctd ~dg 0)

C1
ctg2

6 -vc1-q)-q ctg1 o

Integrando , obtenemos
q> (6) = arccos ,

C1ctg6

1-C:

+ C2

120

CAP. 11

EXTREMO D E FUNCIONALES

6
<p (6) = arccos (C ctg 6} + C 2 ,

donde

De aqu

C ctg 0= cos (<> (0) - C:?J


6

ctg 0 =A cos <p (O) + B sen cp (O),

(20)

donde

A= co~C:>.

= se~C2

Multipl icando por R sen 0 ambos miembros de (20). obtenemm.

R cos 0 = AR cos cp sen 0 + B R sen <p sen O


o. pasando a las coordenadas cartesianas,
z =Ax+ By.
Esta es la ecuacin de un plano que pasa por el centro de la esfera
y que corta su superficie segn un cfrculo mximo. Por consiguiente,
la lnea ms corta (linea geodsica) es el arco del circulo mximo.
EJEMPLO 8 . Demostrar que para una superficie de revolucin
es constante en cada uno de los puntos de las geodsicas el producto
del radio del paralelo por e.\ seno del ngulo entre la geodsica y el
meridiano (teorema de Clairaut).
SOLUCIN. En coordenadas cilfndricas la ecuacin de una superficie de revolucion tiene la forma
x = p cos cp, y = p sen cp, z = f (p).
Determinemos los coeficientes E, F y O:

E = 1
f2, F = O, G = p'.
Por eso, la dHerencial ds de la longitud de arco en la superficie de
revolucin tiene la forma

ds ==

V p1 + (l + f,t)p' 2 dq>.

En la superficie de revolucin las lneas geodsicas sern extremales


de Ja funcional
q>

) V P'+(l +f~)p'Zdcp.
qlo

Como Ja funcin integrando no contiene explcitamente cp, obtenemos


Inmediatamente
pi
i.r

- const,

... p2+(l+fi>)p'

1o.

o sea, p

PROBLEMAS CON

~~

FRONTERAS MVI U:'.5

121

=Const. Observando que p :: =senw (fig. 18), obte-

nemos psenca>=const que es lo que se quera demostrar.

170. Hallar la distancia ms corta entre los puntos


y B (O, -1, 1) en la superficie x +y+ z = O.
171. Hallar las lineas geodsicas del cilindro circular

A (l . O, -1)
r =R.

JO. Problemas variacionales


con fronteras mviles
10. Problema elemental con fronteras mviles. Sea F = F (x , y, y ' ) una
funcin diferenciable tres veces respecto a sus argumentos y sean
y = q> (x)
e
y = "1 (x)
(1)
donde cp (x) E C1 fa , b] y "1 (x) E
E C1 la, b], dos curvas en el plano xOy .
Consideremos la funcional

J [y (x)J =

P (x, y, y') dx

(2)

Fig. 18

'V

definida para las curvas suaves y = y (x) cuyos extremos A (x0 , y 0 )


y B (x1 y1) se encuentran en las curvas (1) de modo que !lo= <p (Xo)

e y 1 = '1)1 (.x1). Se pide hallar el extremo de la funckinal (2).


TEOREMA. Supongamos que en la curva Yo : y =
(x) se alcanza
el extremo de la funcional

J(y(x)J=

F( x, y, y')dx

entre todas la curvas de la clase C1 que unen dos puntos arbitrarios de


dos curvas fijas y= q> (x) e y = ip (;e). En tonces ta curva 'Yo es una
extremal y en los extremos A (x0 , y 0 ) y B (x1 , y 1 ) de la curva 'Yo se cumplen
las condiciones de tran.sversalidad

fF+(q'-~Fll, f:c-xo = O,
fF+(ip'-y')F11 .J lx~x 1 =0.

(3)

Es decir, para resolver el problema elemental con (ronteras


mviles es preciso:
1) Escribir y resolver la ecuacin de .E~ler ~orrespondiente.
Como resultado, se obtiene una famllla de extrernales /J = f (x, C1 , Cz),
que depende de dos parmetros C1 y Cz.

122

CAP. 11. EXTREMO DE FUNCIONALES

2) D eterminar las constan tes C 1 , C2 x 0 y x 1 de las condiciones

de trans versalidad (3) y de las ecu aciones

f(x 0 , Ct. C2) = q>(xo), }


C , Ci) = \I> (xi)

( )
4

f (xi,

3) Calcular el extremo de Ja funcional (2).


EJEMPLO 1. Hallar la con dicin de transversalidad para la funcional
XI

Jf

J (y (x)J =

(x. y) earctg !I'

V 1 +u~ dx,

f (x,

Y)

* o.

XI)

SOLUCIN. S upongamos que el extremo de la izqu ierda de la ex trema! se ha fijado en el punto A (x0 , y 0 ) mientras que el extremo de Ja
d erecha B (X, y,) puede desplazarse por una curva y = 11> (x) . Tendremos entonces

En nuestro caso es
F = f (x, y) e~1rctg r/ V 1 + yi

l +y'
F ti'= f (x , y) earc.tg y---''-=-1 + y'~

La condicin de transversalidad se represe nta as

ft <x, u>

ercti: 11'

Vi +!1'2+
+ C'1>'-y')/(x,y)e11rctg11'

l + u'.

V1 + u~

J1x=x1 = 0.

De aqu obtenemos, debido a la condicin f (x. y ) :::/= O,

'1>'-y'
1 + ip'y'

-1.

(5)

Desde el punto de vista geomtrico, la condicin (5) gnifica que las


extremales y= y (x) deben cortar la curva y = q, (x) por la cu al
se desplaza el pu n to extremo B (xl> y 1) de modo que el ngulo entre
es tas curvas sea de

n
T

Efectivamente, la r elacin (5) se puede tran!'.formar del m o do


s iguient e: s upon gamos que Ja tangente a la extrema! en el punto
B (x1 , y 1), que pertenece a la curva y = '11' (x), for ma ngulo a. con
el Ox y que la ta ngen le a 1a curva ija y = <p (x) forma ngulo ~
(fig. 19); entonces, se tiene tg a.= y', lg ~ = 'lj>' y el primer miembro

d e la frmula (5) da tg

~ - a
trar.

= -

(~

- a); pero como -1

, de do n de a =

tg ( -

: ), resulta

~ que es lo oue se quer a demos

f 10,

PROBLEMAS CON FRONTERAS MVILES


EJEMPLO 2. Hallar la distancia de la

x-y = 5.

SOLUCION.

parbola g

123

X'- a la recta

El problema consiste en hallar el valor extremo de la

integra l
!lt l

J [y (x)J =

JV 1+

11' 2 dx

%1)

con la condcin de que el ex tremo de la izquierda de la extrema! se


puede desplazar por la curva g = x mientras que el extremo de la

Fig. 19
derech a, por la reda y = x - 5. Por consiguiente, tenemos en este
caso <p (x) = x2 y \Ji (x) = x - 5. La solucin general de la ecuacin
de Eul er ser y = C1x + C2 , donde C1 y C, son constantes arbitrarias
que deben ser determinadas.
Las condiciones de transversalidad (3) tienen la forma

donde y' = C1 Las ecuaciones (4) t ienen en nuestro caso la forma


C1xo +C2 =x~,

C1X1 +C2=X1-5.

C.AP. ll. fX TREMO DE FUNCIONALES

124

Tenemos, pues, un sistema de cuatro ecuaciones con cuatro incgnitas C1 , C2. Xo y x1:
C1
l/ 1+q+c2xo-C1) -. r
= 0,
V 1+q

-V1 +Cf+O-C1) V e,

1+q

=O,

C1Xo+C2 = x~,
C1X1 +C2=X1-5;

resol vindolo, obtenemos

C1= - I.

xo= 2

C2=4,

23

Xt=g

Es decir, la ecuacin de la extrema! es y = -x +

! y la distancia

de la parbola a la recta es igual a


Z3

8
l=

23

Jlf1 +(- 1) dx=-V2x l: = 19 f 2.


1

172. Hallar la distancia ms corta del punto A (1, 0)


+ 9y-z = 36.
173. Hallar la distancia ms corta del punto A (-l. 5)
a la parbola y 2 = x.
174. Hallar la distancia ms corta de la circunferencia
x2 + y 2 = 1 a la recta x +y= 4.
175. Hallar la distancia ms corta del punto A (-1, 3)
a Ja recta y = 1 - 3x.
176. Demostrar que en el caso de la funcional
a la elipse 4x2

J[y(x)] = ) h(x, y)V1+y'2 dx ,

donde h (x, y) ::F O en los puntos frontera, las condiciones


de transversal idad tienen la forma
y' (x) = -

1
qi'

(x)

y' (x) = - il>' 'ex)

o sea, las condiciones de transversalidad se reducen a las


condiciones de ortogonalidad.

S JO .

PROBLEMAS CON FRONTERAS MVILES

125

2. Problemas con fronteras mviles para funcionales de la forma

XI

J [!/ (x), z (x)J =

F (x, y, z, !I', z') dx.

(6)

:ro

Al analizar el extremo de la funcional (6) aceptamos que por lo


menos uno de los puntos frontera A (x 0 , y0 , z0) o B {x1 , y1 , :z1) se
desplaza por una curva fija.
El extremo de J l!I (x), z (x)} se puede alcanzar slo en las curvas
integrales del sist ema de ecuaciones de Eulcr
F11 - -

dx
d

F , = 0,
11

F,-..!!_F.
=0.
d,t ~
Supongamos que el punto A (x0 y0 , z0) est lijo mientras que el
otro punto frontera B (x1 , y 1 , z1 ) se. desplaza por una curva definida
median te las ecuaciones

<p (x), }
z =, {x).

!I

En este caso, la condicin de transuersolidad tiene la forma

[F+(cp'-y') F11 ,+ (1jl'-z') Fzl lx-:ti =0.


Anlogamente se escribe la condicin de transversalidad para el
extremo de la izquierda (si .ste tambin se desplaza por una
curva g= ~ (x) }) :
Y=1'> (x)

[F + fq' -y') Fi.i+(~'-z')Fi,J lx-ro =O.


EJEMPLO 3 .

a la re.eta

Hallar la distancia ms corta del punto M (x 0, y 0 , z0)

g=mx+p,}
Z=nx+ q.

SOLUCtON. El problema se reduce a la determinacin del extremo


(mlnimo) de la integral

JV + y'+ z'dx

"1

J [g (x), z {x)J=

::ro

(7)

126

CAP. 11. EXTaEMO DE FUNCIONALES

con la condicin de que el extremo de la derecha de Ja extr ema! puede


desplazarse por 1a recta

y=mx+p,}

(8)

Z=nx+q.
~

o sea, en nuestro caso las funciones rp y


la forma

q> (x)

= mx + p y

'!> (x)

tienen, respectivamente,

nx

q.

La solucin general del corr espondiente sistema de ecuaciones de


Euler ser

y=C1x+c2, }
z =C3 x+C4,

(9)

donde C (i = 1. 2. 3 y 4) deben ser determinadas.


La condicin de lransversal idad (en el extremo de la derecha)
tiene la forma
y'
+
[ VI +Y''+z'2 +(m-y') Vi+u't+z'i

z'
J1x-x1
2 +i' 2
Vt +Y'

+Cn-z')

=0,

de donde, puesto que y' = C1 y z' = C3 , obtenemos

(10)
l -1- mC1
nC3 = O.
La relacin (10) expresa la condicin de perpendicularidad en tre la
recta buscada (9) y la r ecta dada (8).
Empleemos el hecho de que la recta buscada (9) pasa por el punto
M (xo. !lo zo}:
(11)

y tambin el hecho de que el extremo de l a derecha se desplaza por


la recta (8):
Clx1+Cz = mx1+P }
C3x1-J-C4=nx 1-!-q.
De las cinco ecuac iones ( 10, {l I)

e,, e, y X1 (xo. Yo Zo.

y (12)

debemos determinar CJo

(12}

c.,

m. n, p y q son nmeros dados). Para calcul ar

la integral (7) basta conocer Xi. C1 y

e,.

Tenemos

xo + m (110 - P}+n (io-9)


X
i+nz+mz
'
C _ mxo+mn (zo-9) -(1 +nZ) (Yo- P}
t - m(110- P)+n(zo - q)-(m 2+ n2)xo'
C _ rtxo+ mn(yo-p)-(l+mZ)(zo- q)
3
- m(y0 -p)+n(z0 -q)-(mz + nz}Xo
- ~..;..._~...=.,-...-;,-=..,...:--::-'~_.;.:.

1o.

PROBLEMAS CON FRONTERAS MVILES

127

Introduciendo estos valores en (7), obtenemos


h = mn .1 (y (x), z (x)I =

_ , / x2+ (y -p)Z+ (z _

(xo+m (go-P)+n (z 0-q)J2


1+ n2 + m2

Si el punto front era A (xo. Yo Zo) est fijo y el o tro pu nto rron tera
B (x1 , y 1 , z1 ) puede des plazarse por una supt'rl icie z ~..::. <p (x, y) , las
condicion es de l ra nsversalidad sern
-

q) 2 -

[F- y' F11 ,+(<p~ -z' ) Fi, ll x...x1

=_ }

fF11 , + F 2; <p 11J lx-:ci -

( 13)

O.

Las condiciones (1 3) conjun ta mente con la ecuacin z = <p (x , y )


permiten determinar, hablando en trminos generales, dos constantes
arbitrarias de la solucin gen eral de l s ist ema de ecuaciones de Euler
(las otras dos constantes se determinan de la condi cin de que la
extrema! ha de pasar por el punto fi jo A (x 0 , y 0 , z0)).
Si el punto mvil es el pun to frontera A (x0 , y 0 , z 0 ) . obtenemos
para x =-= x0 unas condiciones anlogas completamen te a las condiciones (1 3).
EJ EMPLO 4 . Hallar la distancia ms corta del punto A ( 1, 1, 1)
a la superficie esfrica
;x2 + yZ
zZ = J.
(14)
SOLUC tON. El problema consiste en analizar el extremo de la
runciooal

I
j

J[y(.l:). z(x}J =

v1+y'2+ z'2dx.

(15)

Xt

donde el punto B (x11 y 1, z1 ) debe estar en la su perficie es[rica (14) .


Las ex lremales de Ja funcional (1 5) son las rectas

y = Csx+Cz, }

z = C3 x+C4.

( 16)

De la condicin de que Ja extremal (1 6) pase por d punto A (1,. 1, 1) ,


obtenemos
C1+C2= 1, }
(1 7)
Cs+C~= l.
tienen la orma

128

CAP. (l.

EXT~EMO

DE

FUN~IONALES

t enlendo en cuenta (16), despus de unos clculos sencillos encontramos de aqu


Z1-C3X1=0,

(18)

C1z1-C31J1 =0,
donde x,. y1 y z1 son las coordenadas del punto buscado B.
De la condicin de que la extrema! (16) pasa por el
B {x1 , y 1 , z1 ) tenemos

punto

Y1=C1x1+Cz, }

(19)

z1 =C3 x1 +C4
De (17), (18) y (19) encontra mos

C1 = I,
C~= O,
C3=
y
de modo que la ecuacin de la extrema] es

e~ =

!J =X , }

(20)

Z=X

Puesto que el punto 8 (x 1 , y 1 , z1 ) debe estar en la superf icie esfrica

(14). obtenemos, tomando t:>n consideracin (20), que x~

1, o sea. que x,

va .
1

+ x~ +

xf

Por consiguiente, obtenemos dos puntos

(-1 _J _1 )
' -V3' V3' ~r3

B (

- y 3

--v13 --va1 )

Es fcil ver, por razones geomtricas, q ue en la ex trema! (20) que


une los puntos A y 8 1 la funcional (15) alcanza su mnimo igual a
1

Jmtn =

JV1+1 +1dx=V3-1
t

va
mien tras que en la extrema! (20) que une los puntos A y B~ esta fun
cional alcanza su mximo
1

Jmx=

V3dx= VJ+ l.

- -V3
OBSERV ACI N 1. Al deducir las condciones de lransversalidad (18)
hemos considerado que <> (x, g)
xi - y 2 Es fcil ver que
las condiciones (18) subsisten si q> (x, y} == - V i - x'l - y9
OBSERVACIN 2 . Queda claro, por razones geomtricas, que la
extrema) (20) es ortogonal a la s uperfic ie esfrica x2
y'J
~ = l.

=yt -

+ +

PROBLEMAS CON FRONTERAS MVJ LES

10.

129

EJEMPLO 5. Consideremos el mismo problema sobre el extremo


de la funcional (15) pero tomando como A el centro de la esfera
O (O, O, O) .
SOLUCIN Las exlremalcs de la funcional son las rectas (16) y la
condicin de que la extrema! pase por el punto O (O, O, O) da inmediatamente C9 = e, = O.
Las condiciones de transversalidad sern las mismas

Z1-C3X1=0, }

(21)

C1z1-Ca!J 1 =0,

y las condiciones en el extremo mvil sern


!/1 =C1x1. }

(22)

Zs =C3X1,

Por ltimo,
(23)
Para determinar las cinco magnitudes C, e,. Xi. Y1 y Z1 tenemos
einco relaciones (21), (22) y (23) de las cuales solo tres son indepen
qientes:
(24)

Empleando las relaciones (24), encontramos

Xt=l(l-1-C:+q'
zl =

Y1=v1+q+q'
1

":'":'-;:::::::::;;:;;:=::;::;-

V 1-1-q+q'

donde C1 y C 3 son unas constantes arbitrarias.


Consideraciones geomtricas aclaran esta arbitrariedad: la distan
cia del punto O (O, o. O) a la superficie esfrica (14) es la misma en
cualquier direccin, o sea, para cualesquiera valores de C1 y C3
El valor de la funcional J [y (x), z (x)] en las extremales

y=C1x. }
z=Csx
es igual a

Jy(x),z(x)J=
9-01387

Vi+chci
~
v1+q+qdx=I.

130

CAP. 11. EXTRf!MO DB f'UNCIONAL e.S


EJEMPLO 6.

Hallar ta condicin de transversalida d para Ja fun

eion al

Jf(x,
%j

J[u(x), z(x)J =

!f, z)"J( I +u'z+z'2dx,

(25)

si el punto A (x 0 , g 0 , z0 ) est fijo y el punto B (x1 , y1 , 2 1 ) se encuentra


en 1a superficie z = q> (x, y).
SOLUCJON. En este caso las condiciones de transversalidad sern

(l + q~z') lx-xt =0, }


(y' + <i>vZ') l;l;lal%J = 0,

o sea,

'I

q>; x ....:ic =

~ 1X'o:o.ltl =-=-r
~ l cpv

:ic1.

~epr esentan ta condicin de paralellsmo del vector -r {I, y', z'}


tangente en et runto B (x., fl' Zt) a la extrema! buscada y del vector
n {cp;, q>~, - 1 de la norma a la superficie z = q> (x, y) en este mismo
punto. P-or consiguiente, para las funclonales de la forma (25) las
con,diciones de transversal ida d se reducen a las condiciones de ortogohalidad.

177. Demostrar que, si la condicin de transversalidad

y la condicin de ortogonalidad coinciden para todos loo


datos iniciales, la funcin integrando F tiene la estructura
siguiente:.
P = f (x. y, z) VI+ y' 2 +z'2 ,
donde f (x, y, z) es una funcin diferenciable cualqu iera de
!J y Z.
178. Ha!Jar la distancia ms corta del punto M (O, O, 3)
a la superficie z = x 2 yi.
t 79. Hallar la distancia ms corta del punto M (2, O, 5)
a la superficie z = ~
y".
180. Hallar la distancia ms corta entre las superficies

X,

+
+

~ + r~ + ~ = 1

x2+!f+z2=4.

181. Analizar el extremo de la funcional

/ [y(x) 1 z (x)I=

~ (y'2 +z'+2yz)dx

J
o

s 10.

PROBLEMAS CON PRONtERAS MOVI LES

131

si y {O) = z (O) = O y el punto B (x1 , y 1 , z1) se desplaza por


el plano x = x1
3. Distancia geodsica, El valor de la integral
B

J [y (x)) =

F (x, y, y') dx,

(26)

ca1culada segn una linea


desde el punto A hasta el punto B , se
denomina J-longitud de la nea y. Si y es una extrema! se dice que
J [y (x)] es la distancia geodlsica entre Los puntos A y B, o simplemente
Jdistancla, y la propia extremsl se demomina J-recta.
EJEMPLO 7. Hallar la distancia geodsica del pun.to A (O, O) al
punto B (1, 1) si esta distancia se define mediante la funcional
B

J [y (x)) =

J!Pr/2

dx.

SOLUCION. La distancia geodsica del punto A al punto B es Igual


al valor de esta funcional en la extremar que une dichos puntos. La

ecuacin de Euler es
2yy'Z -

:X (2g2y') = 0

!JY' +y'2= 0.

Es Hici 1 ver que

llJI"

d
+ g'2 = dX
(gy'),

de modo que 2yy' = C1 e y1 = C1 x


C9 Empleando las condiciones
de frontera !J l:r=o = O e y 1x=i = 1, obtenemos C1 = 1 y Ca = O.
Por consiguiente, la extrema! que une los puntos A y 8 es la parbola
yl =X.
Tenemos ahora 2yy'

1, yy'

!11. por consiguiente,

(yy') =

!.

Por defi nicin, la distancia geodsica entre los puntos A y B es igual a


f.

l (A, B)=

J! !.
dx=

Supongamos que se tiene una linea Z: q> (x, y) = O.


la distancia geodsica entre un punio B que no pertenece a :C
y esta linea se defi ne como la distancia geodsica del punto B a un
punto A E :C que se obtiene calculando la funcional (26) segn Ja
extremal 'i que une los puntos B y A con la particularidad de que y
corta tra nsversalmente Ja lfnea :C en el punto A.

CAP. 11 . EXTREMO OE FUN lONALS

Se denomina !circunferencia (o circunferencia qeodtsica) la lnea


formada por los puntos que esUn a una misma distancia geodsica
de u n punto ijo. Anlogamente se definen los conceptos de / elipse
y de J hiphbola.
EJEMPLO s . Hallar la / circunferencia de radio R y con centro
en el punto O (O, O) si la distancia geodsica se define mediante la
fun cional
B

J (y (x)J

y'2/2 dx.

SOLUCION. Las exlremales de la funcional corlan transversalmente la circunferencia geodsica. Para las extremales tenemos (vase
el ejemplo anterior)

C1 x,

2yy'

C1

y, por consiguiente,
,,1 _

" - 2x
De la condicin de transversalidad

y'y' (2q>' - y') = o


encontramos que el coeficiente angular de la tangente a la Jcircun
ferencia es q>'

= y;

ferencia es y'=

y, por eso, la ecuacin diferencial de la J-circun

fx , de donde resulta la ecuacin de la / -circunfe-

rencia: y' = Cx. Para determinar el valor de C observemos que el


punto (to, C) est en la circunferencia geodsica y que la ecuacin del
radio geodsico (o sea, de la extrema\) que pasa por dicho punto es

!f = ~ .

De aqui tenemos yy'

=~

es

es
R=

y, por lo tanto,

Jo

(yy')2dx=

Jo 4~2

dx=

~.

Es decir, C = 4R y la ecuacin de la circunferencia geodsica de ra


dio R. y con centro en el origen de coordenadas es !!' = 4Rx.
EJEMPLO 9. Hallar la / circunferencia de radio R. y con centro en
el runto O (0, O} si la distancia geodsica se define mediante la funciona
B

Jy(x)J=

Jlft+y'2dx.
A

10.

PROBLEMAS CON FRONTERAS MVILES

133

Las exlremales de la funcional son las rectas y=


De la condicin de que la extrema! deba pasar por el
punto O (O, O) encontramos C9 = O de modo que y= C1 x y, por consiSOLUCIN.

= C1X + e,.
guiente, y'

= JLX

La condicin de transversalldad coincide en este caso con la


condicin de ortogonalidad y, por eso, el coeficiente angular de la
tangente a !circunferencia es -q>' = - :, . Por consiguiente, la
ecuacin diferencial de la /-circunfenca es y'= - ~ . De aqur resulta
y
la ecuacin de la !circunferencia: x2 +y" = C'. E l punto (C, O)
est en dicha circunferencia. La ecuacin del radio geodsco que pasa
por este punto es y = O de modo que y' = O y

R= ) dx -c.
o
Es decir, C = R y la ecuacin de la circunferencia geodsica buscada
de radio R es la ecuacin de la circunferencia corrien te x.2
y2 = R2
OBSERVACION. Los conceptos Introducidos permiten hablar de la
Geometrla no euclidea con la diferencial de arco

ds

F (x, y, y'} dx.

Yl

Si F =
+ y'2, las /-rectas se convierten, como hemos visto, en
las rectas corrientes y nuestra Geometra se convierte en la euclldea
corriente.
Si F es una funcin arbitraria, que slo satislace las condiciones
habituales de ser continua y derivabl e respecto a los tres argumentos,
1a Geomelria construida muy poco recuerda 1a corrien te: no siempre
se puede trazar una J -recta por dos puntos y puede suceder que por
dos puntos pasen varias Jrectas y, por consiguiente, que la J distancia
entre dos puntos no sea una fun cin unvoca de las coordenadas.

182. H allar la distancia geodsica del punto A (O, O)

al punto B (1, 2) si esta distancia se define- mediante la


funcional
J(y(x)J=) (!f+y' 2 )dx.

183. Hallar la distancia geodsica del punto A (O, l) al


punto B (1, l) si esta distancia se define mediante la funcional

J [y (x)J = ) (12xy+ !/ 2) dx.

184. Hallar la J-circ:unferencia de radio R = 8 y con


centro en el punto O (O, O) si Ja distanci a geodsica se define

134

CAP. 11. EXTREMO DE. FUNCIONALES

mediante Ja funcional

Jy' dx.
3

J [y (x)J =

t t. Problemas discontinuos. Variaciones unilaterales


10. Problemas discontinuos. La extrema! y=y (x) de la funcional
::<

J (y (x)] =

F (x, y, y') dx

(1)

xo

es una funcin que tiene dos derivadas continuas siempre que la deri
vada F V'll' (x, y (x). y' {x)) sea diferente de cero. Sin embargo, existen
problemas variacionales en los cuales el extremo se alcanza en una
curva suave a trozos solamente.
o) PROBLEMAS DISCONTINUOS DE PRIMERA ESPECIE. Consideremos
el problema sobre la determinacin del extremo de la funcional (l)
aceptan do que las curvas admisibles satisfacen las condiciones de
frontera
(2)

<

y pueden tener un punto angular en el punto de abscisa e (x0 <e


x1).
Este punto angular puede darse slo all donde F 11 = O (vase el
11

teorema 2 de !a pg. 53). En el punto angular Ja extremal debe satisfacer las con.diciones de Weierstrass - Erdmann
F11 ,f:1:=c-o-F11 ,/~=c+o=O, }

(F-y' F11 ,) /~ ...c-o-(F-y' F11,) b:=e+o=O.

(3}

Conjuntamente con las condiciones de continuidad de la extremal


buscada, estas condiciones permiten determinar las coordenadas del
punto angular.
En cada uno de los seg mentos [x0 , e) y [e, xi] la extrema! debe
satisfacer la ec.uacin de Euler, o sea, una ecuacin diferencial de
segundo orden. Al resolver estas dos ecuaciones se obtienen cuatro
constantes arbtrarias que, hablando en trminos generales, se deter
minan de las condiciones de frontera (2) y de las condiciones (3) en el
punto angular.
EJEMPLO 1. Hallar las extremales quebradas (si es que e~isten)
de la funcional
a

JI!! {x)) =

J
Q

(y'2- y2) dx,

P~O BLEM AS

11.

DISCONTIN UOS

1. 35

SOLUClN. Escribimos la primera de las condiciones (3) que deben


cumplirse en el pun to angular:

F ll' b:-c-o = Fv' l:r~e+o

(O <e < a).

En nuestro caso tiene la forma

y' (e - O) = y' (e

+ O)

y significa que la derivada y' (x) es continua en x = c . Por conslgulen!e, no ha y puntos ang ulares. Esto se puede ver ta mbin de que en
nuestro caso F 11 , 11 , = 2 >O en todo punto. Por lo ta nto, en el problema considerado ~~extremo puede alca nzarse slo e n curvas suaves.
EJEMPLO 2. ttallar las exlremales quebradas de la funcional
2

J(g(x}J=

S(g'4-6g'2)dx;

y(0)=0,

y(2)=0;

o
aceptando que y' puede ser discontinua en el punto correspondiente
a la abscisa x = c.
SOLUCION. En este caso F , , = 12y'2- I2 se puede anular, y , por
11 11
eso, puede ocurrir que la extrema! tenga puntos angulares. Puesto
que la funcin integrando depende slo de y', las extremales son las
rectas
Pongamos

< <

<

y_ = mx
/1 (O
x
e) e Y+ = px q (e x ~ 2).
De las condiciones de frontera encontramos n = O y q = - 2p de
modo que
Ymx e IJ+ = p (x - 2).
La condicin de continuidad de la extremal da

me= p (e - 2).
Escribamos las condiciones de Weierstrass -

F 11 ,=4g'S-t2y'

(4)

Erdma nn. Tenemos

F- y'Fv, = - 3r'' + 6y'2.


Puesto que y:_=

u: = p,

obtenemos

4m3-12m=4p3-12p,
}
-3m' +6m2= -3P'+6p2.,

o sea..
(m - p) (m2+mp+p2- 3)= 0, }
(m'l. - p?.) (m2+pZ-2~= 0.

(5)

136

CAP. 11. EXTREMO DE FUNCIONALES

La segunda de las ecuaciones (5) da inmediatamente m = p, m = -p o


m' + p1 - 2 = O.
La solucin m = p debe ser excluida: en este caso la extrema\ tiene
derivada continua y de Ja condicin (4) obtenemos m = O, o sea, la
extrema! es un seg mento del eje Ox.
Por consiguiente, para resolver el sistema (5) hay que resolver
dos sistemas de ecuaciones:

m = -p, }

6)

~ +~+ P2 =3

y
m2+p2=2, }

m2+mp+P2=3.
sistema (6) es: m=Y3,

(7)

p=-113

La solucin del
y m = - y3,
=
La solucin del sistema (7) es m = p y debe se.r excluida.
Es decir, m ::: -p y la condicin de continuidad (4) da e '= 1.
Por consigu iente, las extremales buscadas son:

y3.

Y= {

"V3x,

-V3 (x -

2),

u= {

--V3x,

Y3(x-2), 1 ~ x ~ 2.
185. Hallar las extremales con punto angular para la

funcional
2

J( y(x)] =

Jy' (y' 2

l)2 dx;

y(O)=O,

y(2) = 1.

o
186. Hall ar la solucin c.on un punto angular en el probl ema sobre el mnimo de la funcional
4

J(y(x)] =

J(y' -

1)1 (y'+1) 2 dx;

y (O)= O,

y (4) = 2.

o
187. Existen soluciones con puntos angulares en el

problema sobre el extremo de la funcional


x1

J (y (x) 1=

J(y'2+ 2xy- !/') dx ;


.lCg

Y (Xo) = Yo1 !J {x1) = Yt?

PROBLEMAS DISCONTINUOS

11.

137

188. Hall ar la solucin con punto angular en el problema

sobre el extremo de la funcional


1

J {y(x) J=

J y2(1-y' )dx;
2

y( - 1) =. 0,

y( l )=l.

- 1

189. Hallar la solucin con punto angular en el problema


sobre el mnimo de la funcional
:l<1

J(y'' -

2y'2 ) dx.

%1

190. En el problema sobre el extremo de la funcional

(:l<'r 111)

sen y' dx

(0, 0)

hallar ta solucin continua y la solucin con punto angular.


OBSERVACIN. Las condiciones (3) de Weierslrass-Erdmann
admiten la siguiente interpretacin geomtrica.
Consideremos la fguratriz o sea, la curva Y = F (x, y, y') cm
tanto que funcin de y'.
Las condiciones (3) signHican entonces que para los valores de los
parmetros x = e y y = c1 , que corresponden al punto angular, la
figuratriz debe tener una misma tangente en Jos puntos de abscisas
y:._ = y' (e - O) e yi, = g' (e
O).
Al mismo tiempo se obtiene una Interpretacin cl ara de la condl
cin F 1111 =PO que excluye la posibilidad de puntos angulares en las
extremales. Efectivamente, si, por ejemplo, es F 11 , 11 > O, la fguratriz
es cncava hada las Y positivas y no podrn coincidir sus tangentes
trazadas en dos puntos distintos. Es dec ir, en este caso la extrema!
no podr tener punto angular.
Consideremos de nuevo el problema sobre la determinacin de las
ex trema les quebradas de la funcional del ejemplo 2 de este pargrafo.
Tenemos

Jly(x)]=

Jo

(y'' -6y'2)dx;

y(O) = O,

y{2) = 0.

Las extremales son rectas. En este caso la figuratrlz Y = y' 4 - Gv' 2 no depende del punto (x , y) . Tiene una tangente comn eJl

138

CAP. JI . EXTREMO DB PUNCIONALES

Jos puntos de abscisas y'= 1(3 (fig. 20). Por eso, las condiciones
de Weierstrass-Erdmann quedarn cumplidas si como extremales
y

Fig. 20
quebradas toma mos las quebradas cuyos lados formen ngulos de

con el eje Ox.

En la quebrada y 1 con un punto angular (fig. 21) la funcional


toma el valor J fyd = - 18. Este mismo valor tendr J (y (x)] en la
y

Fig. 21
quebrada y 11 con dos puntos angulares, (ig. 22), en la quebrada y 1
con tres puntos (fig. 23), etc.
b) PROBLEMAS DISCON TINUOS DE SEGUNDA ESPECle. Se denominan
problemas dist;onlinuos de segunda especie los ~robl emas sobre el extre-

s 11.

PROBLEMAS DtSCONTI NUOS

139

mo de la funcional
XI

J[(x)J=

F(x, y, g')dx;

g(x1)=Y

y(x2)=v2;

(8)

;X:f

en la que la funcin Integrando es discontinua.


Supongamos, por ejemplo. que F (x. y, y') es discontinua a lo
largo de la curva y = <!> (x) y sea F (x, y, y') igual a F 1 (x, y, y')
a un lado de la lnea y = di (x) e Igual a F2 (x, y, y') al otro lado.

Fig. 22
SI existe la extrema\ quebrada, deb.er componerse de los trozos
de las extremales y = y 1 (x) e y = Ys (x) que tienen un punto comn
(e, <!> (e)), e E (x1 , xa), en la linea de discontinuidad. Para hallar la

Flg. 23
extremal quebrada obtenemos dos ecuaciones diferenciales de Euler
cuyas soluciones generales contienen cuatro constantes arbitrarlas C1,
C9 , C1 y
Para determinar estas constantes as! como la abscisa e
del punto en el que la extrema! encuentra la curva g = <!> (x) tenemos:
1) dos condiciones de fronteras (8), 2) dos condiciones segn las cuales
las ordenadas de los extremos de las extremales en el punto de juncin
han de ser iguales a la ordenada de la curva .Y = <l> (x) y, por ltimo,
3) la condici6n de la jun.cin

e,.

F1 +(q>' -() f

w lii:-c-o """.F2+ (<I>' -y') !'211 1~-e+o:

{9~

140

CAP. 11. EXTREMO DE FUNCIONALES

Hablando en trminos generales, estas condiciones alcanzan para


determinar la exfremal quebrada.
EJEMPLO a (problema de la refraccin de un rayo de luz). La luz
se propaga con una velocidad constante u1 en el medio I y con una
ve.Jocidad cqnstante v 2 en el medio l l. La curva y= <I> (x) separa los
medios I y I l.
Deducir la ley_de refraccin del rayo de luz que va desde el punto
A del medio l hasta el punto B del medio 11 si se sabe que es minimo
el tiempo durante el cual el rayo de luz recorre este camino.
SOLUCl N. El .problema consiste en hallar el mlnlmo de la inetgral

J [Y (x)J=

r Vf+Y'2

Jo

dx+ Jb

Ot

lt'T+vz dx,

(10)

V2

t:

ya que la primer.a y la segunda integrales de (10) representan, respecti


vamente, el tiempo que necesita el rayo de lu.t para llegar del punto A
a la lfnea de separacin y de la linea de separacin al punto B.
Tenemos un prot>lema discontinuo de segunda especie siendo

Para determinar los trozos de las extremales debemos hallar las extremales de la funcional

JV

l+y'Zdx

que, como se sabe, son rectas. Por consiguiente


!lt =mx+n

y 2 =px+

q.

Escribamos la condicin (9). Tenemos


F _
1

lf ~

aF t _
!li iJy~ -

, i1F2
Fz-y,-,

au,

t11

Yi'
uqf 1+Y'

Ut

V 1 +Y'

V'+ y;

Introduciendo estas expresiones en (9), encontramos

1+ <ll'Y
v1-V1 +u1

1+a>'y
oi-V 1+y;2

(11)

Sea y el ngulo que forma con el eje Ox la tangente a la lfnea de s~pa


racin en el punto de abscisa e, sea a el ngulo que forma con el eje
Ox el rayo de la izquierda y sea f} el ngulo que forma con el eje Ox
el rayo de la derecha. EntQnces, se tiene ~ - tg y, Yi. == tg a, yp =

s t l.

P~OBLEMAS OISONTINUOS

141

= tg ~ y la condicin (11) toma la forma


1

+ tg a tg i'
V1 + tgz a

l
~

cos (y-ex)

cos(y-~)

v1

+ tg ~ tg -y
V l + tg2~
Vz

Ut

donde y - e.t y i' - ~ son los ngulos entre los rayos y la tangente
a la llnea de separacin. Tomando en lugar de ~slos los ngulos cp y 0
entre la normal a la lnea de separacin y los rayos, incidente y refractado, obtenemos
sen q>
v1
--=-=consl,
sen a
Clz
o sea, la ley de refraccin del rayo de luz.

20. Variaciones unila terales. Se pide hallar el extremo de la


fun cional
~s

J [y (x)) =

F (x, y, y') dx;

a;

con la condicin
y -

q> (x) ~ O

( y -

cp (x) ~ O)

(12)

(las condiciones de limitacin pueden ser de forma ms compleja).


En este caso Ja extrema! busca da puede estar formada por trozos
de extremales que pertenecen al recinto (12) y por trozos de la fronte
ra y = q> (x) de este recinto. En Jos puntos de j uncin de estos trozos
la ex1remal buscada puede ser suave y tambin puede tener puntos

angulares.
La condicin en el punto de juncin tiene la forma

[F(x, y, y')-F(x, y, ip')-(q' - g')F11 .(x, y, y')J I

- = O.

;ll:mo2:

_Si_F..,,11 , ::FO, la extrema! es tangente en el punto de juncln


M (x, y) a la frontera y = q> (x) del recinto.
EJEMPL O 4. Hallar en el recinto y ~ x 2 el camino ms corto del
punto A (-2, 3) al punto B (2, 3).
SOLUCION. El problema consiste en hallar el extremo de la funcio
nal
2

1 IY <x) 1=

jV

+ui

(13)

d.x

-2

con las condiciones

!J ~ x',

y (-2) = 3,

y (2)

3.

i42

CAP. 11. extREMO DE PUNC!ONALS

Las extremales de la funcional (13) son las rectas

y= C1

En nuestro caso

+ C x.

Fvv =

:1+0
(l+y'2)2

y la extrema! buscada se compone de los trozos AM y NB de las rectas


tangentes a la parbola y= xi y del trozo MON de esta parbola
(fig. 24). Representemos las abscisas de los puntos de tangencia por
y -i (utilizamos la simetra del problema). En el punto de tangencia

A(-2,J)

)(

Fig. 24
coinciden las ordenadas y los coeficientes angulares de la recta y de la
tangente a la parbola de modo se tiene
C1 +Cai=~:

(l)

Cz=2x.
Por otra parte, la tangente debe pasar por el punto B (2, 3) y, por
consiguiente,

C1

+ 2C, =

(15)

3.

x' - +

Eliminando C1 y C, de (14) y (15). encontramos


4.i 3 = O,
de donde i 1 = 1 y i, == 3. E l segundo valor de no sirve. Es decir,
i = 1 y C1 = -1 y C, = 2. La extrema! buscada (nica) es

-2x- l

11 =-

x?.

2x-l

si
si
si

- 2<;:x<-l.

<x<

-1
l,
1 < x<;:2.

Queda claro que la funcional (13) alcanza en ella su minimo.

TE.ORfA DE

HAMIL tON .. JACBl

143

191. Hallar las curvas en las cuales puede alcanzarse el


extremo de la funcional
10

J[y(x)]=

Jy' dx;
3

y(O) = ,

y(lO) = O;

o
si las curvas admisibles no pueden pasar por el interior del
crculo que limita la circunferencia (x - 5)2 + yz = 9.
192. Entre las curvas que unen los puntos A (a, y 0 )
y B (b, y1 ) hallar la curva que ofrece el valor extremo a la
funcional
l>

1 y <xH ==

j y V 1-

b'2u'2 dx

con las condiciones

y;;::r:o, 1- y2y'2 ;;;:r:o.

12. Teora de Hamiltoo-Jacobi. Principios variaclo


nales de la Mecnica
I. Forma cannica (hamlltoniana) de las 1:.cu1tiones de Euler.
Las ecuaciones de Euler para la funcional
J [Y1t Y2 .,

Ynl=
F (x, Yt. !h., ., Yn Y1 y;, ... ,

y~) dx

( 1)

Xt

tienen la forma
d
F 11 - - d F
ll

,-o

lllt.

{k= 1, 2, .. ., n).

(2)

En e.I caso en el que el determinante


F11'11'
F 11'111'a ' ' ' F 11'v'
11
in
F11' 11'F11'11'
a1 t2

pondremos

F , = Pk
111

(k

F11'u '
zn

l , 2, .. , n).

De las ecuaciones (4) se puede expresar yi en trminos de x, JJi, y,,


Yn Pi. Pi. ., Pn=

g =

CJ>1t.

{x, Y1, Y ., Yn P11 Ps. Pn)

(3)

(4)

i 44

CAi'>. JI. EXTREMO OE FUNCIONALES

La runcin H de las variab les x, y 1 , y,, . . ., y71 , p 1 , p 2 ,


definida media nte Ja igualdad
ff = [-F(x, !lt Y'2. ... , Yn Y

u2 ..... u,.. u;..

Y1.

u~ .. . , Y~)J 1 ,

~~

se denomina hamtltoniano de la funcional ( 1).


E l h amilton iano satisface las relaciones s iguientes
iJH
dg11,
iJH
dp1t

= ""iiX,

{Jph

iJy1

Pn

y;, ... , y,)+

+ ~1
~ u.,F , (x,
~

, ,

=-

(k = l, 2, . .. , n).

dx

(5)

Se dice que las ecuaciones (5) son el sistema cannico o hamilfoniano


de las ecuaciones de Euler (2); las variab les y 1 , !lt . . , Yn Pt> P i .. .

. . . , Pn llevan el nombre de oariables cannicas.


OBSER.VACION 1. La condicin (3) en el caso de la fun cional
oca

J (y (x)l =

F (x, y, y') ) dx

da

Fll'v'

=F O en

[x i. x2).

:i;

Hablando en trminos generales, las ecuaciones


yk en todo el segmento lxir x1 ) . Si se cumplen las condiciones del teorema de existencia
de la funcin implicita, las ecuaciones (4) a dmiten solucin unvoca
localmente.
EJEMPLO 1. Formar el sistema cannico de las ecuaciones de Euler
para 1a funcional
O BSER.VACJON 2.

(4) no se pueden resolver unfvocamente respecto a

;(

J [Y1 !12)=

J(2Y1Y2-2u~+ Vi. -Yi )dx.


3

11

SOLUCIN.

F
Ponemos
Entonces

En nuestro caso

F (x,

Y1.

Yi. Y. Y2) = 2YiYs - 2y ~

+ Y2 -

F = P1

F ,

Ps

Ps

-2y ;.

lit

P1

2y.i

112

Y*

Aqui el determinante
F1111 Fll'll'
l 1
l s
Fll"ll'
Fll'V
'
2 1
8 2

= 12o -

O1
2 = -4-:;.

o.

Resolv!end.o respecto a Y e y2 las relaciones obteni das, encontramos


,_ Pt
Y1 - 2

P2
,=-2

y'

12.

145

TEORfA DE HAMILTO N JACOBI

Formamos el hamlltoniano de la funcional conside rada

H = (-F +11i.F11 + yFu)


1

Pt

111- 2

11'=-P

=(-2Y1112+zut + 11~-11i9>

, 111
111...T

p:

PI

=2g,-2111112+4-- 4

11--~

:a

Empleando las relaciones (5), obtenemos el sistema cann ico de las


ecuaciones de Euler
dy t = .!!l..
dx
2 '

dp,
dx

Aqu y1 =

Y1

= -4Yt +2112;

(x), Ya = /l:i (x),

P1

Incgnitas de x .

dyz=-~
dx
2 '

dp2 _ 211
-;t

=Pi (x) Y Ps = Pa (x) son funciones

EJEMPLO 2 . Formar el sistema cannico de las ecuaciones de Euler


para la funcional

1b'11 Yz)=

yfyl(x2 + Yi+ll)dx.

SOLUCIN. Aqui

+ + u).

F = 11f11I (x11
Y
Determinamos las derivadas parciales

F11 = Y?YI.

F11 = Y?YI

Ponemos

Pt = Y~YI Y Pz=Y~Y=
Estas relaciones no comprenden las derivadas vi e Yi de las funciones
Incgnitas y 1 e y 2 ; por eso, no se puede expresar y ' e Y en trminos
de p 1 y p 1 Por consiguiente, no se puede formar e\ hamiltoniano de
esta funcional. En este ejemplo no se cumple la condicin (3):

\-1 1-= .

F llll Ft1lf
F , , F , -

111111

11s11

O O
O O

EJEMPLO s. Formar el sistema cann ico de las ecuaciones de


Euler para la funcion al

J [y (x) 1=
10- 01581

xgy' 3dx.

146

CA P. 11 . EXTREMO De FUNCIONALES
SOLUCIN.

Tenemos
F

Pongamos p

= xyy'3

3:r11y' 2 ,

F 11 ,

3xyy'!.

de donde

Y' = -JI-~

1/=Va~g

.e

La funcional considerada tene dos hamlltonianos

.111-

H1=(-F+y'F 11

vp-=
~

=2xyy'3

I
11

"'-l P = 3V3 V
1"

,rv = -~
sva ..V/
V 3xV

3
P

xg

;;a

fl1 = (-F+y'F11'

, __

:cy,

ll:tl/

En concordancia con esto obtenemos dos sistemas cann icos de las


ecuaciones de Euler:

dy
dx

l ..

dp

r /p
3xy.

/7
ax11a

"dX=s V

Formar los sistema cannicos de las ecuaci'ones de Euler


para las fun cionales s iguientes:

JX!J V y' dx.


J [y (x)J = Jxyy' dx.

193. J [y (x)} =
194.

195. J [y(x)J =

JJ/x + y J(l +x':>. dx.

t 96. J[y1 . Y2} =

J(y;2+ y: + y~ ) dx.
2

s 12.

'l'EOIHA t>E HA MILTON-J ACOBI

J(x +YtY;'+!k!1~ )dx.


J l!ltt Y21 = J(2.xgt - y~'+ y. y;) dx.

197. JLyt. y 2) =

198.

2. Ecuacin de Hamllton-Jacobl. Teorema de Jacobl. El


sistema cannico (5) de las ecuaciones de Euler es el sistema de ecuaciones de Euler para la funcional
J

fYt U2, ., Ynl =


=

r[~ Pk!I~
ot(

-H (x, Yh flz, .. , !In Pt /Ja, . , Pn)] dx

l-t

si Yi !Jt .. , !Jru P1t p 1 , , Pn se consideran en tanto que fun


clones incgnitas de x.
Esta funcional J es la solucin de la ecuacin en derivadas par
dales de primer orden

aw

-iJ + H
x

( x,

aw

ow )

iJW

Yt. yz, ., Yn -0 , ~, ., -iJ = O,


!lt
'-'112
':In

que se denomina ecuacin t Hamilton-Jacobi.


Tl?OR.EMA DE JACOBJ . Supongamos qu.e W es la integral completa
de la ecuacin de Hamllton- Jacobi y satisface la condicin

J2W
iJ2W
02W
Yt 8Ct iJy , 8C2
lit iJCn
iJ2W
a2w
01-W
iJy20Ct iJy2iJC2 . iJ112iJCn

.. .

+o.

iJ2W
o:>.W
02W
iJyn iJC t iJyn lC2 ... aun iJC,.
Entonces las igualdades

aw

iJC11. =B1t.,

aw

iJy" =P1i

(k-1, 2, . .. ,n),

don.de C11. y B1c son unas constantes arbitrarias, determinan u.n a solucin
MI sistema cannico (5) dependiente M 2n constan/es arbitrarias.
EJEMPLO 4. Hallar las extremales de la funcion al
J

IY (x)] =11:J Y x2

+ !12 V l + y'

2 dx

:ti

utilizando la solucin de la ecuacin de Hamilton-J acobi.


10

CAP. 11. EXTREMO DE FUNCIONALES

148

sotu c10N. Para obtener la ecuaci n de Hamilton- J acobi forma


mos el hamiltoniano de la funcional considerada. Tenemos

H= - lfx2+y2- p2.
La ecuacin de Hamilton-Jacobi tiene la forma

aw - l .-/, xz+ yz- (w


- )2=O

iJx

iJy

ax + (W)2
Tu = xz+yz.
( oW)2
~epresentemos

(6)

la ec uacin (6) en la forma

(~~ r

-xZ

+( ~~ )2 - yz =o

y apliq uemos el mtodo de separacin de variables. Queda claro que


la ecuacin (6) se verHca si se exige que

)2 _
( aw
ax

xz =

_e

aw ) 2 _
ay

yz =e 1

donde C es una constante arbitraria. De aqu encontramos

aw
ax =Vxz- c

aw
ov =1fy2+c.

La integral completa de la ecuacin (6) ser

W ==

V !!2+ Cdy=
c-.E..1n)
x+V xz- cl+..!..y
2
2 Y y2+c+

lfx2-C dx+

=..!_x,l'xz2 V

+ ~ ln (y+ VY+c l+Co,


2

donde C y C0 son constantes arbitrarias.


De la relacin

~~

={, donde A es una constan le arbitrara,

'l /-, +- (x+

determinamos la solucin general de la ecuacin de Euler. Tenemos


X
11 x+ v x2-C
---=-.........,-- -

4 V x2

++ 11;+c+-}

C
4

VxZ-C)

+
1fx2-C

ln ly+VY2 +cl+

A'
+--(y+ -V y2+c) V yz+c T
e

TEORA DE HAMlLTONJACOBI

12.

149

Despus de unas simplificaciones sencillas, ohtenemos

In

u+
VY2+"C l=A,
x+Vx2-C

o sea,

u+Vyq:G =A
x+lfx2-C

(A= eA.),

de donde resulta definitivamente

es decir, una familia de hiprbolas.

Hallar las extremales de las funcionales siguientes:

:x:2
199. J [y(x)] =

j xyV y' dx.

;q

200. J{y(x)J=jxyy''<-dx;

y(l)=O,

y(e)=l.

r
1

201. J [y(x)J=

G (y)Vl +y'2 dx.

202. Hallar el mnimo de la funcional


1

J {y(x)] =

J(f y' + yy' +y'+ y) dx


2

o
si se desconocen los valores en los extremos del segmento.
203. Hallar la funcin del campo p (x, y) y el propio
campo de extremales que pasan por el origen de coordenadas
para la funcional

1:Y'

c:x:.11>v--2
Jfy(x)J=)
dx

(y>O).

(0 , 0)

204. Entre las lneas que unen el punto x = O con el


punto M1 (xlt y1), donde X1 >O e Y1 > O, hallar la lnea
en la que alcanza su mnimo la funcional

150

CAP. H . EXT'REMO DE FUNCIONALES


XI

J [y (x)] = ) "Vr+?2 dx
o
!I

(y> O).

Supongamos que se tiene Ja funcional


%

J [Y (x)[ =

JF (x, y, y') dx

Xf

= cp (x, C). Entonces en todo


punto del campo se conoce la direccin de la transversal del campo que
pasa por este punto. Todas las transversales del e.ampo se obtienen
como las soluciones de la ecuacin diferencial de primer orden
y se conoce su campo de extremales y

F11 , [x,

qi (x, C),

q>~ (x,

C)]

:~ = H {x,

q> (x, C),

<Px (x,

C)),

donde en lugar del parmetro C, que determina las exlremales del


campo, hay que introducir su expresin en trminos de las coordenadas
de los puntos del campo. Aqul H (x, y, p) es el hamiltoniano.
EJEMPLO 6. Hallar las transversales para el campo de extremales
y = Cx de la funciona l

J [y (x}] =

y'2 dx.

:Et

SOLUCIN. Formamos el hamiltoniano de la funcional considerada.

Tenemos

y'2

{F 11 , 11 ,2 =I= O).

F '//' = 2y'

Poniendo P=F11 ,, encontramos

y'=r~

H= C-fl'2+2y'g') 1

,,,_

Las transversales se obtienen resol viendo la ecuacin diferencial


F 11'

donde en lugar de

l11-C%

e hay

:~

IJ>-2'//' -

=H
zc t
que tomar su expresin en trminos de las

coordenadas de los puntos del campo: C = ..f!_.


Tenemos
)(,

2y'

lv-cx :~ = ~ /p-2c

Puesto que C *O, se tiene 2

:~ =C,

2C

:~ =

o sea, 2

C2.

~= ~ .

De aquf

eo~ontramos cue ta f~mi)ia de transversales son las parbolas 112=== e~.

12.

TEOR!A

DE

HAM!L TON- JACOBI

151

205. Hallar las transversales del campo de extremales


y = Cx de la funcional
~t

JF (y') dx.

J [y (x)J =

y =

206. Hallar las transversales del campo de extremales


X+ C de la funciona!

J(xy'' -2yy'

:1:2

J (y (x)) =

3)

dx.

:q

207. Hallar las transversales del campo de extremales


x2
y= x - c de fa funcional
:r2

J [y(x)]=

JVy(l-y' )dx
2

(C>O, x>O, y~O).

Conociendo la ecuacin de Hamilton-J acobi

aw
ax +H {x, u. aw)
au =O
de la funcional
:x:a

J [!I (x)J =

F (x, y, ;y') dx,

=1

se puede reconstruir la funcin integrando F (x, y. y'). Esta es solucin de la ecuacin diferencial de primer orden

F - zF; = -H (,t', y, F~.


(7)
donde H (x, IJ, p) es el hamiltoniano de la funcional considerada
y F (x, y, z) es 1a funcin incgnita (se considera que x e y son par
metros). Despus de determinar F (x, y, z) hay que tomar en ella la
derivada y' en lugar de z.
OBSERVAC.I(>N. La ecuacin (7) es la ecuacin de Clairaut. Como
regla, la solucin general de la ecuacin de Clairau t se omite pues
en este caso la funcin integrando F (x, y, y') es lineal en y' y el
problema variacional no siempre tiene solucin (vase el 4). Por
eso, se toma slo la solucin ingular de la ecuacin de Clairaut que
ser precisamente la funcin buscada F (x, y, z).
EJEMPLO 6. La ecuacin de Hamilton-J acobi en el problemji
sobre el extremo de la

funclon~l

.:X-1

l [y (x)l = } F (x, y, $1') 4x Hem1


:114

C AP. 11 , E X T REMO D I! FUNCIONALES

152

la forma

oW)2 +(W)2
7i = xz+ y2.
(OX
Hallar la Funcin F (x, y, y').
SOLtJCION. ~esolviendo la ecuacin dada respecto a la derivada

oW
ax ,

tenemos

aw . /
-=V

x2 + 1J2-

( ay
aw )2 .

o sea,

Por consiguiente, el ha milton iano es


H = -yx2+y2-p2.

La ecuaciou (7) para la determinacin de la unci6n F tiene la


forma
dF
F- z
-=
dz

x2+ y2- _dF


di

)2

(8)

Derivando respedo a z ambos miembros de la ecuacin (8), resulta

dZF

Dejando a un !arlo el caso dzZ =O (que da la solucin general, tenemos


dF

dz

z = -,---::
/:=====d==F::=2;::-

V :cz+uz-( t1Z)

Resolviendo esta relacin respecto a la derivada


dF

_zV~

-z -

v1+t

~:

, encontramos

(9)

12 .

TEORA DE H MAILTQN.. J ACOB I

153

Introduciendo (9) en (8), obtenemos

F=z z Vii'+Y2 + .. / x+ 2_ z2 (x~+Y2>


"JI+zz
Y
Y
l+zi

vx+ yi y1 +zz.

Por consiguiente, la funcin Integrando buscado tiene la forma

F="Jx2+!12 Vl+ y'z.


En los problemas que siguen hallar las funcionales a partir
de sus ecuaciones de Hamiltoo-J acobi:

ow ow _
og -

209. 4 iJx

..2.,,2
.-!:f

aw )2=0.
ax+ (<f

210. 4xy aw
211.

)2=X2+!f.
aw )2 + (y aw
( X;iJy

so. Principios variaclonales de la Mecnica.


a) PRINCI PIO oe HAMIL TON-OSTROO RADSI<J. Supongamos que se
tiene un sistema den puntos materiales M11. (x1P Yk :z11.} {k = 1, 2, ... , n)
con masas respectivas mk (k = 1, 2, . . . , n). Suponga mos que el
movimiento del sistema est sometido a enlaces
'PJ (x, y, z, t) = O {/ = 1, 2, . .. , m;
m ~ n)
(10)
y se realza bajo la accin de.las fuerzas P-,., (X1., Yk, Z11 ) (k = 1, 2, ... , n)
que tienen el potencial (funcin de fuerza} U = U (xA, Y1u z11.. t) :

au

Xh=;;-,
vX

La energa cintica de este

T=21

au

au

ZA=~ ..

Y 11. =:;--,
v1jR,

V'k

sist~a

ser igual a

"

""'
L.J

m11 (x1t+Y.11+.t1t).

Jt... J

Supongamos que este sistema pasa de cierto estado A correspondiente


al momento del tiempo t = t0 a otro estado B correspondiente al
momento de tiempo t = t1 Entre todos los desplazamientos posibles
del sistema de A a B se escoje la clase de movimientos admsibles que
concuerdan con los enlaces dados y que hacen pasar el siste.ma del
estado A a l estado B en el intervalo de tiempo dado ( 10 , 11 ).
El principio de Hamilton-Ostrogradski consiste en lo siguiente:

entre todos los moulmiento.s admislbles que hacen JJasar el sistema

CAP- JI. E XT REMO DE F UNCIONALES

154

del estado A al estado B, el movimienio real se caracteriza por el


cumplimiento de la Cflfldicidrt necesaria (JJ =0 de ex/remo de la
funcional
t1

J=

(11)

(T-i-U)dt .

to
A cada moYimiento admisi ble del sistema le corresponden 3n
funciones X1t (1), 1111: (t), :z11 (I} (k = l , 2, . . , n) que estn definidas
en el intervalo [10 , / 1], que satisacen las ecuaciones (10) y que toman
determinados valores en los extremos del Intervalo [t0 , t1 J. Por consiguiente, tenemos un problema Yariacional con los enlaces (1 0} y con
fronteras fij as.
Para resolver este problema formamos la funcin auxiliar de
Lagrange

F= T + U +

2}

'-1 (t) 'PJ

i=-1

y escribimos para cita el sistema de ecuaciones de Euler- Ostrogradski:

Ucpj

m1iY11. - Y1t-

(12)

X1(t) 0!11i = 0,

i- 1

'PJ

m11,z11,-Z11.-

~/..1(1) ozh = 0.
j=1

El ss tema ( 12) conklde con las ecuaciones diferenciales del movlmlen


to real del sistema.
b) PRINCIPIO DE 1.A ACCIN

~MI N IMA BN 1.A FORMA DI! LAGRANOB.

Supongamos que los enlaces q> y el potencial U no dependen del


tiempo t. En este caso tiene' lugar la integral de energa T - U
= h = const. La integral

11

se

denq rnin~

=} Tdt
'

!C<;(<'!p. De la integral (11) se ded uce


IJ

J(T-!-U)41=~ ~~
t9

11

T dt- J hdt.
IQ

qu~

l :Z .

TEOR fA DE

15.5

HAMI L TONJ ACOBl

El principio de la accin minima en la forma de Lagrange consiste


en lo siguiente: para el movimiento real, la integral de la accin debe
tomar su valor min.imo, o sea,
lt

I=

JTdt=min.

to
El principio de la accin mlnima puede ser representado en la
forma de Jacobl

JV2(U+li) ds = mn
y

(ds es la diferencial del arco y) en la que no interviene el tiempo.


OBSERVACIN t . Aqui se consideran admisibles los movimientos
que satisfacen las ecuaciones de enlace IJ>J (x, y, z) = O (J
1, 2, , m}
y la ecuacin T - U= h con el mismo valor de h que para el movi-

miento real y que tienen los estados inicial y final ijos, siendo tambin fijo el momento Inicial /0 del tiempo . El momento final del tiempo
no se fija para estos movimientos.
OBSERVACIN 2 . La energa potencial figura no en Ja Integral sino
en la condicin complementaria T - U= h. Formamos la funcin
auxiliar de Lagrange
m

"1

= 2 T+2 (U+h) + Ll

),,JIJ>J

;-1
Despus escribimos las ecuaciones de Euler-Ostrogradski para nuestro
problema

m11,X11,

<JU

oq>

=~+2
""' 'A.J ~,
VX]l.
LJ
v:CJt
;- 1
m

au
iJ<p1
mhYk=-::i- +2 "1 A.1 ;;-- ,
vflft
Ll
v/lf1.
;-1

"

m11z~ =

iJU
OZ1t

+2 ~ A.1

ocp
ozk

;-1

que representan las ecuaciones del movimiento real.


EJEMPLO 1 . Basndose en el principio de la accin mlnima, hallar
la trayectoria del punto material (de masa unitaria) que se mueve
por accin de la gravedad.
sOLUClN. Tomando ~I eje Oy hacia arri ba, el potencial de la fuer
ia de \~ gravedad es
(l3)

156

C AP , 11 . EXTREMO DE F U NCIONAt.l!S

Segn el principio de la accin mnima, para la trayectoria bus


cada 'Yo la integral
J ....

sV2(U+ ~)ds

(14)

debe alcanzar su valor mnimo. Por consiguiente, la trayectoria ser


una extrema! de la funcional (14). Introduciendo (13) en (14), obtene
mos

J
i.:1

J=

V2<h-eY>Vt+y '2 dx .

(15)

La ecuacin de Hamilton-Jacobi tiene la forma

ow
ax - ,,V-2h-2gy - ( ow
au )2=
o sea,

( ~~) 2+ ( ~~

=2 (h-gy).

Su integra 1 completa es

r
1
W =Ax+ J V2h-2gy-A2 dy=Ax- ag { 2h -

2gy-A2)

-2

+ B,

donde A y B son constantes arbitrarias.


Determinamos las exlremales de la funcional (15) :
t

A
2
x + -g(2h - 2gy- A2) =C,
o sea,
!1=

~-

:; -

2~2 (x-C)2;

C constantes.

En particular, las extremales que pasan por el oriien de coorde


nadas se determinan de la condicin y {O) =0. Obtenemos una fami
lis monoparamlrica de parbolas
Y= -

2A2 X

v~
A

x.

212. Hallar en el plano ta trayectoria de un punto que


se mueve por efecto de una fuerza repulsiva que acta desde
el eje Ox en direccin del eje Oy y que es proporcional a Ja
distancia del punto al eje Ox aceptando que la integral de
la fuerza viva tiene la forma ~ - ~s = O y basndose en

12.

TEORtA DE HAMI l. TONJACOBI

i57

la integral de la accin
X2

J [y (x)] =
=

Jy Vl + y'

dx

(y> O).

213. Un punto material describe la circunferencia p =


2R cos q> (P. cp son las coordnadas polares) de radio R

bajo la accin de una fuerza central : 6 inversamente proporcional a la quinta potencia de la distancia al centro que se
encuentra en el origen de coordenadas. Demostrar que la
integral de la accin alcanza minimo fuerte en cualquier
arco de esta circunferencia ( - ~ < <p1 ~ tp ~ (f>\l < ~ ) .
214. Analizar el movimiento de un punto material por
efecto de una fuerza central de atraccin proporcional a la
dis tancia al centro O basndose en el principio de la accn
mnima y aplicando el mtodo de 1-familton-J acobi.

Captulo i 11
MTODOS DIRECTOS EN EL CLCULO VARIACIONAL

13. Mtodo de diferencias finitas de Euler


Consideremos el problema variacional elemental: hallar el extremo de la funcional
b

J[y(x)] = 1F (x,g,g')dx;

y(b) = B .

y(a) = A,

(1)

Segn el mtodo de Euler, los valores de la fun cional (l) se toman


no e.n las curvas arb itrarias que admite este problema varacional,
sino en las quebradas compuestas por un nmero dado n de segmentos
rectllneos cuyos vrt ices tienen abscisas fijas
a+x,

a+2x, .. . ,

a+(n- l )x,

donde

b-a

Ax=--.

En estas quebradas la funcional J [y (x)} se convierte en una funcin


<D (Yi. !12 . ., Yo-1) de las ordenadas YI> y,, . .. , Yn-1 de los vrtices
de la quebrada. Las ordenadas y 1, y, . .. , Yn - i se escogen de modo
que la fu ncin I> (!/i. y 2 , . , Yn-i) tenga extremo, o sea, se determinan del sistema de ecuaciones
oI>

iJy2 =0, ... ,

~o
Yn-t - -

La quebrada as obtenid a es la solucin aproximada del problema


variacional (!).
EJEMPLO. Hallar la solucin apro ximada del problema sobre el
mlnimo de la fu ncional
1

J(y (x)l=

J(y 2+2y)dx;
1

y(O)=y(l)=O.

o
SOLUCIN.

!lo
Ya

=
=

1-0

Toml'mos Ax= - 5
y {O) = O,

Y1

Y (0,6),

!14

!1 (0,2),
y (0,8),

0,2 y pongamos
y~

y (0,4),

!15 = g ( 1)

= o.

13.

~TODO

159

DIFERENCIAS FINITAS

01$

Sustituimos los valores de la dervada segn la frmula aproximada

, _y' (x ) ,...,
!Jlt -11 ,..,

Y1t+t -

l\x

Yk

Entonces

g' (O 2)- !h.-Yl

y' (O)- Yt-0

-----ay-,

' -

0,2

y' (O oi)- Y4 - !13


' . 0,2 '

y' (O 8) - O-Y+.
.
' 0,2 .

Sustituimos la integral por una suma empleando la [rmula de los


rectngulos:

Jf

(x) dx ~ [f (a)+ f (x1) +

f (x2} + ... + f (Xn.-tH l\x.

(l

Tendremos

<D (Ytd'2, !13.!/i.) = [

+ ( !!3;;!12 )

( ~~

y+(!!~~ )2+ 2Yt +


Yl

;;!1

+2u2 + ( 11

2
)

+ 2us+ ( - %,~ ) +2y,J 0,2.


2

Formamos el sistema de ecuacones para determinar las ordenndas


y,. y,. Ya e y, de los "'rtices de Ja quebrada buscada:

0:2 ~: = 0~2 - - !/~~%' + 2 = 0


o<D

!12-Yt
0,02

y3 -!J2
0,02

_ 1_ o<D
0,2 i'Jy3

Ya-Y2

y,-y~
0,02

_l_
0,2

oy2

0,02

!J4-Ys
1 i'JCD
0,2 iJy4. = 0,02

o sea,

!14

0,02

+2=

0,

+2 = 0 ,

+ 2 =

2!11 -!/2 =-0,04,}


-1J1+2!12-!!s= -0,04,
-u2+ 2Y3 -!J4 = -0,04,
-ua+2y,
= -0,04.
La soluci n de este sistema es y1 = -0,08, y 2 = -0, 12, y 3 = -0,12
e y 4 = -0,08. Estos valores de la solucin aproximada coinciden
con los valores que t iene en los puntos respectivos la solucin exacta

x2-x

y= - 2 -

GAP . 11 1. MTODOS OIRl!CTOS

160

Hall ar las soluciones aprox imadas de los problemas sobre


el mnimo de las funcionales:
1

21 5. J{y (x)}=) (y' 2 +y2 +2xy)dx;


o
SUGERENCIA. Tomar X=,2.

y ()= y( l)=.

216 .1 [y (x)] =

J(y' + l) dx;
2

(O) = O,

y (1) =O;

b) y (O) = O,

y(l)= 1.

a) y

14. Mtodo de ftz. Mtodo de Kantorvich


10. Mtodo de Rlb. La idea del mtodo consiste en que al hallar
el extremo de la funcional J [y (x)) se consideran, en lugar del espacio
de las funciones admisib les, slo fas funciones que se pueden representar como combinaciones lnealcs de las funciones admisibles:
n

Yn (x)=

2j a11P1 (x),

( 1)

}col

donde IX son unas constantes y el sistema {q> (x) }. llamado sistema


de fu nciones coordenadas, est formado por funciones cp1 (x) que son
linealmen te independientes y que constituyen un sis tema completo
de funciones en el espacio considerado.
H ablando en trminos generales, cuan do pedi mos qut:' las (un
ciones Yn (x) sean admisibles, Imponemos a las funciones coordenadas
cp 1 (x) ciertas condiciones complementarias co mo, por ejemplo, limita
ciones en cuan to a la derivabilidad o en cuanto a la verificacin de
las condiciones de frontera .
En estas combinaciones lineales la funcional J fy (x)) se convierte
en una funcin de los argumentos a 1 , as, ... , an:
l IYn (x)I = <D (et., ct2, ., ctn)
Determinamos los valores a 1 , a 2 , , an que ofrecen extremo a la
funcin <D (1X1 , ct 2, . , 1Xn); para ello resolvemos el sistema de ecuaciones

-a()<D
=0
a

(i= 1, 2, ... , n),

no lineales, como regl a, respecto a IX1 , a 2 , , ct1u e introducimos


en (1) Jos valores encontrados para IX. La suct>sin {Yn (x)} que asl
res ulta P.S una sucesin mi nimlzante, o sea, la sucesin de los valores
de la luncional {J (y11 {x)I } obtenida a partir de ella converge hacia

J 14 .

MaTODO DE RITZ. M!TODO DE KANTOROVICl t

tfH

el mnimo o hacia la cota inferior de la funcional J fy (x)}. Sin embargo,


de

lim

n ... oo

no se deduce an que

IYn (X}]= min J 111 (x)J

lim Yn (x) = y (x). La sucesin rninimizante

n-+oo

puede no converger hacia la funcin que realiza el extremo en la


clase de las funciones admisibles.
Se pueden indicar las condiciones que garanticen que el mnimo
absoluto de Ja funcional exista y se alcance en las funciones {l/n (x) }.
En el caso en el que se trata del extremo de la funcional
:i:s

J (Y (x)! =

F (x, g, y') dx;

Zt

fl (x1)-= Ytt
11 (x2) = U2:
estas condiciones son:
1) la funcin F (x, y, .t) es continua respecto al conjunto de sus
argumentos para cualquier t y para (x, y) E D. donde D es un recinto
cerrado del plano xOy al que pertenecen las lineas Jln (x);
2) existen unas constantes <X > 0, p > 1 y P tales que

F (x, y, z) ;;;;;a 1 z IP

+P

cualquiera que sea z y para cualquier punto (x, y) E D;


. 3) la funcin F (x, y, %) tiene la derivada parcial conlinua
Fz (x, y, z) y esta derivada es una funcin no decreciente de
z (- oo < z < +oo) cualquiera que sea el punto (x, y) E D.
En particular, las condiciones enunciadas se cumplen para las
funcionales

J (y (x)J=

(p (x) y'z+q (x} y2-t-2r (x) yl dx;

if.(x1)

=a,

y (xa)

=b;

donde p (x), q (x) y r (x) son funciones dadas, continuas en lxt, x 1 ),


con la particularidad de que existe la derivada continua p' x) de
p (x) y de que p (x) >O y q (.x) ~ O.
Si por este mtodo se determina el extremo absoluto de la fundo
nal, el valor aproximado de su mfnlmo se obtiene por exceso y el
valor aproximado de su mximo, por defecto. Al aplicar este mtodo,
el xHo depende en gran medida de la eleccin adecuada del sistema
{cp1 (x)} de funciones coordenadas.
En muchos casos basta tomar la combinacin lineal de dos o tres
funciones q>1 (x) para obtener una aproximacin bastante satisfactoria
de la solucln exacta.

Si hay que determinar el extremo aproximado de la funciona)


J (z (xi. x,, ... , xn)) que dependen de las funciones de varias varia
H-01381

t62

CAP. 111. MeTODOS DIRECTOS

bles independientes, se escoge un sistema de funciones coordenadas


IJ>1

(xi.

X2, , Xn),

y la solucin aproximada del problema variacional se busca en la


forma
m
Zm (X1' X2 1

Xn) =

~ akQ>lt. (X1t Xz, .. ,

Xn).

k ... 1

donde los coeficientes ak son unos nmeros constantes. Para determinarlos se forma, por analoga con lo que hemos explicado, el sistema
de ecuaciones

!> = O (k =

vcik

1, 2, .. ., n), donde Cll (ai. et11 ,

a n)

es el resultado de introducir Zm en la funcional J (z (x1 , x11 , , x~)).


EJEMPLO t. Hallar la solucin aproximada del problema sobre
el mfnmo de la funci onal

J [y (x}} =

(y'2 - y1.

+2xy) dx;

(2)

o
y (O)

=y

(l)

O;

y compararla con la solucin exacta.


SOLUCIN. Como sistema de funciones coordenadas q>k (x) tomamos

cpk (x)

(1 - x) xll.

(k

l, 2, ... ).

Es. evidente que las funciones 'Pk (x) satisfacen las condiciones de
frontera 'Pk (O) = q>,1 (1 ) = O, son linealmente independientes y for
man un sistema completo en el espacio C1 IO, 1).
Para k = l tenemos !h (x) = et (x - x'). Introduciendo esta
expresin de y 1 (x) en la funcional (2), obtenemos
i

J IVt (x)J =

(af {1-2x)2+af (x-x2)2+2a 1x (x -x2)J dx=

o
1

J(a.l(l-4x+4x2- -'2+2x3-x') +2a.1 (.~2- x3))dx=


o
3

=10 a.f + 6
E1 coeHciente at se determina de Ja ecuacon

Mi

t =s1 +6= 0

ci1

= <D (at).

.METODO DE

14.

de donde result" a 1 =

R I TZ. MBTODO

5
- 18
.

oe

KANTOR OVICH

163

Por consiguiente,

y, (x) = --x+-xz
18
18 .
SOLUCIN EXACTA.

rada es

La ecuacin de Euler de la funciona l considl'

y-+ y=

x.

Resolviendo esta ecuacin lineal no homognea, encontramos


y = C1 cos

x + Ca sen x + x.

Empleando las condiciones de frontera y (O)


definitivamente
sen x

= y (1) = O,

obtenemos

y=x-seT.

Comparemos las soluciones exacta y aproximada:


Solucl6n exada

IC

Solucin aproxln1ada

0,25
0,50
0,75
1,00

- 0,044
-0,070
-0,060

-0.052
-0,069
-0,052

Hallar la solucin aproximada de la ecuacin no

EJEMPLO 2.

lineal

3
y= -z y2
que satisfaga 1as condiciones y (0) = 4, y (1) = l.
SOLVC.10N. A este problema de contorno le corresponde el proble
ma variacional
1

J lu(x)J=

(y'2+y3)dx;

g(0)=4,

y(l)= I.

o
Buscaremos la solucin en ta forma
Y1 (x) = 4 -

3x

+a

(x -

x2);

es evidente que y1 (x) satisface las condiciones de frontera dadas cual


quiera que sea el valor de ~
T enernos

J [yt(x))=

J'

(lat(I -2x)-3}2+ [4-3x + a 1 (x-xZ)]} dx,

{)

CAP. Ill .

164

Merooos

DIRECTOS

de donde
1

oJ(y(x)} =

~ {(l-2x)2[a.l(l--2x)-3J+
o

+3 (x-x2) (4- 3.x+a.1 (x-x2)l2) dx.


La condicin i)J [Yi (x)) =0 toma la forma
iJa1

9et.~
490a.1
1407 = o
-3,0413 obtenc.mos la solucin del problema
Yi (x) = 3,0413x' - 6,0413x
4
posi tiva en lodos los puntos.

y para a..1

Hallar las soluciones aproximadas y compararlas con las


exactas en Jos problemas que siguen sobre el mnimo de las
funci onales:
1

217. J[y(x)l =

J(y' + 2y)dx;
2

y(O)=y(l)=O.

o
2

218. J[y(x)] =

J(2xy+ y + y'
2

2 ) dx;

y(O)=y(2)=0.

o
219. Hallar la solucin aproximada del problema sobre
el mnimo de la funcional
1

J (y(x)}=

J (y' 2

k2if)dx;

y(-l )= y(l)= O;

-1

con Ja condicin complementaria

J if' dx = 1.
-t

EJEMPLO 3. Hallar la solucin aproximada del probll'ma sobre


el extremo de la !uncional

J [z (x, g)J=

" - 2zJ

az ) 2 + ( az ) 2
Jr Jr l- (ax

dx dy.

donde D es el cuadrado - a ~ x ~ a, -a ~ y ~ a. siendo z = O


en la frontera del cuadrado.
SOLUCIN. Buscamos la solucin aproximada en la for ma
z0 (x, y)

ct0

(xi -

ex) (!/' - a).

'

METOOO DE RITZ. M~TOOO DE

l 4.

165

KANTOROVIC H

Es evidente que esta funcin z0 (x, y) satisface las condiciones de


frontera planteadas. Introduciendo 20 (x, y), Zx (x, y) Y Zy (x, y) en
Ja funcional e integrando. obtenemos
256
32
45
Ct~a-9 aoa 8 =

J (zo (x, y)) =

Tenemos despus

o<D

s12

32

45

- - -aoa--a~-0

de donde cto= l6aZ de modo que z0 (.~. y) =

(J)

(ao).

'

5
1642

r_x2 - a2) (y2-a2).

220. Hallar la solucin aproximada del problema sobre


el extremo <te la funcional
J

(z (x, y)J =

j J[ ( ~: -y)

+ ( ~~ + x) j dx dy,
2

t:

donde D es el recinto limitado por la elipse ~: -+


= l.
221. Hallar la solucin aproximada z3 (x, y) del problema sobre el mnimo de la funcional

JJ[ (:: ) + ( ~;) Jd x dy,


2

J [z (x, y) J=

donde D es el recinto: x > O, !J > O y x + y < 1, si la funcin z (x, y) satisface en la frontera f: x = O, y = O y x +


+y = 1 la condicin z Ir= x2 +y?..
20. Mtodo de Kantorovlch. Este mtodo ocupa una posicin
intermedia entre la resolucin exacta y el mtodo de Rilz y se aplica
para analizar el extremo de las funcionales
J (z (x1, x 1 ,

., Xn)]

(3)

que dependen de funciones de varias variables independientes (11 ~ 2) .


Igual que en el mtodo de R itz, escogemos un sistema {<pk (x 1, x2, .. , Xn)
de (unciones coordenadas y buscamos la solucin aproximada en la
forma

(4)
pero considrrando los coeficientes a~ (xJ) como fu nciones incgnitas

de una de las variables independientes.

CAP. 111. M2TOOOS DIRECTOS

166

En las funciones (4) Ja funcional (3) se convierte en una funcional


J[a.i (x1), a 11 (x1), .. , ~ (x1)] que depende de m funciones
a:1 (x), a 1 (x1), ... , cim (x1). Estas funciones se escogen de modo
que la funcional l alcance el extremo y se determinan de las condicio
nes necesarias de extremo para la funcional 7.
Empleando el mtodo de Kantorovich, se obtiene una solucin
aproximada, como regla, ms exacta que la solucin que da el mtodo
de ~ itz con las mismas funciones coordenadas 'Pk (x1 , x 1 , , xn)
y con eJ mismo nmero m de trminos en la aproximacin.
EJEMPLO 4. Hallar la solucin aproximada de la ecuacin de
Poisson
A
.,.z
=

{ -a<
x <a,
-b <;y<;
b

t en e1 rec t.angu 1o D :

si z = O en la frontera.
SOL.UCiN. La ecuacin Az = -1 es la ecuacin de Euler-Oslro
gradski para la funcional

JJ[( ~: }+ ( :; ) -2z]
2

J[z(x, y}J =

dxdy.

(5)

Buscamos la solucin en la forma


z1 (x, y) = (b' -

y1 ) a (x);

es evidente que la funcin z1 (..t, y) satisface las condiciones de frontera

z = O en las rectas y = +b.

lntroducierido esta expresin de z1 en la funcional (5), encontramos


a

J [zt(x y)]=

'

J (~
-o

15

b5cz.'2+! b3a,2

.!
ba.) dx.
3

(6)

La ecuacin de Eu ler para la runcional (6) es

"-

2b2.

= -

4b2

(7)

La ecuacin (7) es una ecuacin lineal no homognea con coeficientes


constantes y su solucin general es

Las constantes C 1 y Ci se determinan de las condiciones de frontera


a (-a)-a (a) =0

s 16.

LOS

VALORES

lo queda C2 =0yC1 =

Y LAS FU NCIONES PROPIOS

2ch

a(x)- -

Es decir, obtenemos

t 1 (x y) =

1-

b2-yz
2

5 a
-2 b
ch

167

de modo que

X
V/S
2"b

, /5 aj

chy '21'

ch

1-

,/sx l

V 2b"
V

ch

5 a
-2 b

Para obtener una aproximacin ms exacta se puede buscar la solucin


del problema en la forma
i2

(x, y)

(b' - y) a 1 (x)

+ (b'

- y') a, (.r).

222. Hallar en el recinto D la solucin aprox imada de


= -1 que se anule en su frontera
si D es el tringulo equiltero formado por las rectas y =

la ecuacin de Poisson 6 z

= ~a X

b.
223. Hall ar en el recinto D la solucin aproximada de
la ecuacin ~ z = - 1 que se anule en su frontera si D es el
y

X=

trapecio issceles formado por las rectas y


y X= 3.

~ 3 x,

15. Mtodo,, variacionales para la determ;nacin


de los valores y de las f un clones propios
La ecuacin de Sturm-Liouville
d
- dx (p (x) y'J +q (x) g = Ay,
donde p (x)
condiciones

> O tiene

l ll

derivada continua y q (x) es continua, con las

y (a) = O e y (b) = O
(2)
tiene la solucin nula (trivial) !I s O cualqulera que sea el valor rea
o complejo de A.

CAP. ll l. MeTO DOS DI ~ECTOS

168

El conjunto de la ecuacin (1) y de las condiciones de frontera (2)


se denomina problema d coniorno /U Sturm - Liouoil/e (1)-(2).
Los va lores de>.. para los cuales el problema de contorno (1)-(2)
tiene soluciones no triviales y == O se denominan valores propios
y las soluciones mismas llevan el nombre de /unciones propia; del

probletn11 de contorno.

La ecuacin (1) es la ecuacin de Euler para el slfulente problema


sobre extremo condicionado: hallar el mlnlmo de a funcional
I>

lu (x)J =

(py'2+ qg2.) dx

(3)

con las condiciones (2) y la condicin


b

(4)

y2 dx= l.

()

Si y = y (x) es una solucin de este problema variacional, tambin


ser una solucin del problema (1 )-(2) distinta del cero Idntico en
vi rtud de la condicin (4). Por eso, los valores propios y las funciones
propias del problema de contorno de Slurm-Liouville se denominan
tambin valores propios y runciones propias de la funcional (3) con 1as
condiciones (2) y (4).
La funcin propia y = !I (x) se denomina normada si

EJEMPLO 1. Hallar los valores propios y las funciones propiae

de la funcional
3

J [y (x)J =

((2x

+ 3)2 y'2 -

y2 dx

o
con las condlcones
y (0) = O,

y (3)

= O,

(5)

g2dx= l.

SOLUCION.

La ecuacin de Sturm-Liouville tiene la forma

-yo sea,

(2x + 3)1 y'

:x

((2x+3)2 y'] = >..y,

+ 4 (2x + 3) y' +

(A+ 1) N = O.

(6)

su.

LOS VALORES

Y LAS PUNCIONES PROPIOS

169

Mediante la sustitucin 2x + 3 = ti la ecuacin (6) se reduce a la


ecuacin lineal con coeficientes constantes
d2y

_. dt'-

dy

+ 4dT+<:i..+nu=O.

(7)

Su ecuacin caracterlst ica

4k'

tiene Jas ralees

+ 4k + ),, + 1 = o
t

,r-

kt,2=-22 v -A..
Consideremos tres casos.
l} A. < O. Entonces la solucin general de la ecuacin (7) es
y = C1e11 ' 1

+ C1e112t,

donde ki y k 1 son nmeros reales; en consecuencia, la solucin general


de la ecuacin (6) es
!I == C1 (2x + 3)11'

+ Cs (2x + 3)111

las condiciones de frontera (5) dan

Ct3"'+C2 3111 = 0,}


C191u+c2~-o.
de donde C1 = O, C2 - O e y s O.
2) A. = O. Entonces

y, por consiguiente,

De las condicione de fron tera obtenemos

C1+C21n3=0,
C1 +C2 ln 9 =0,
de donde C1 =0 y C2 = 0, o sea, ysO.
3) A.> O. Entonces k1 , 2 =
de la ecuaci.n (7) es

- 21

y=e-f ( c,cos ~

vx

i -

- y la solucin general

t +C2sen

v:~ t)

170

C.l \P. 111 . M8TODOS DI RECTOS

Pasando a la variable x, obtenemos

C 1 cosf~ ln(2x+3)J +C2 sen[ ~ ln(2x+3}]


Y

v 2x+a

Las condiciones de frontera t5> dan


c,cos

c~i

ln 3)+c2sC?n (

v
C cos ( -- ln 9) +C sen
1

~1
VX

tn3)

=0, )
<9>

(2 - In 9} =0.

El sistema (9) tendr soluciones no triviales s i su determinante es


igual a cero

v- In 3) sen ( T
v - In 3)
(-f
=0;
v~
-veos (-i- Jn9} sen(-+ ln9)
eos

por

lo

tanto,

sen

(V~ In 3- ~:

In 3 ) =O,

es decir,

S<.'n X

-VI

X(2- In 3) =- 0, de donde - 2- In 3= ruc, Los valores propios sern


4n2n2
An '""'i''23 (n= 1, 2, .. ).

Tomando cualquier ecuacin del sistema (9), por ejemplo, la pri mera,
e introduciendo en ella An en lugar de A., obtenemos
C1 cos nn

C1 sen nn = O,

o sea, C1 ( - t}n = O. de donde C1 = O. Tomando en (8) C1 = O y


41i2n2
= JnZ , ob tenemos las funciones propias del problema conside3
rado
AA In (2x+3)
sen [ .
In 3
(n=I , 2, ... ).

"- 11

Los coeicientes Cn se determinan de la con dicin de normacin


3

Jo y~1

(x) dX= 1

5 15.

lo que

LOS VALORES Y LAS PUNCIONES PROPIOS

171

da

Cn=,r-

y, por consiguiente,
2
&en
Vn(x)=ylii3

ln3

rm In (2.x + a)
ln 3

Y 2 x+s

(n=l,2, ... ).

Hallar los valores propios y las funciones propias en los


problemas que siguen:
1

224. J(y(x)l=

Jo (y +y' )dx;
2

y(O)=y(l)=O;
t

Jo !f dx= J.
2

225. J[y(x)J=

Jx y'dx;
2

y(l) = y(2)=0;

J!/'dx= 1.
i

226. J {y (x)I =

J(6y + x y'
2

dx;

y(l) = y(e)=O;

j ydx= l.
1

2n

227. J (y (x)J =

J(y -y') dx;


2

y (n) =y (2n} = O;

"

2:n

Jtfdx= J.
n

228. J (y (x)} =

J(3y2-(x+ 1) y'
2

dx;

o
1

y (O) =y ( 1) =O;

Jo y dx = J.
2

CA P. 11. MtTODOS DIRECTOS

172

Los valores prof los y las funciones propias del problema varia
cional (3), (2) y (4 tienen varias propiedades importantes.
1) Si A,,. y A.n son dos valores propios diferentes de la funcional (3)
con las corioiciones (2) y (4) y si llm (x) y !Jn (x) son las funciones pro
plas que les corresponden, estas !unciones !lm (x) e Yn (x) son ortogo
nales, o sea,
b

!lm {x) !In (x) dx =O

+ n).

(m

2) Todos los valores propios A.n de la funcional (3) son reales.


3) Si An es un valor propio de la funcional (3) e lln (x) es la fun
cl6n propia normada que le cotresponde, se tiene

J l!ln (x)]

= "-n

4) El menor de los valores propios coincide con el mlnimo de la


funcional (3) con las condiciones (2) y (4) .
EJEMPLO 2. Demostrar la desigualdad
n

Jo

>

y'Z (X)

Jo

y (0) = y (n) = o.

yz (x) dx,
n

Determinemos el mfn

:>OLUCION

g't. (x) dx con las condiciones

lt

Jy2

(x) dx= \,

y (0) = Y (n) =O.

()

La

~y"~1on

de Euler para la funcional


n

J (U (x}J =

(g'2 - f..g2) dx

o
tiene la form a

+ 1.y =

y (O)

O;

O,

y (n) = O.

Las funciones prop ias de este ltimo problem a son !In (x) = sen nx
A.n = n1
E l valor propio mlnlmo es A.i = 1. Por eso, en virtud de la propie
dad 4),
y los valores propios son

mn

J
o

y'2 (x)dx= l.

!5.

LOS VALORES

Y LAS F UNCI ON ES PROPIOS

En consecuencia, para cualquier funcin y (x) tal que

yi (x) dx = l

lenemos
n

Io

lt

y' 2 (x) dx :?-

yZ (x) dx.

Esta desigua !dad no se puC'de precisar ya que para Y1 (x) =

sen x

lf

se tiene

JYi

(x) dx =

yf (x) dx= l.

1(

OBSERVAC IN. Si

Jy 2(x)dx= k2 + 1, el problema se
o

anterior introduciendo la func in z (x) =

y~)

reduce al

Empleando la definicin extremal de los valores propios, sealemos cmo pueden ser calculados aproximadamente a partir del mtodo
de Ritz. Debe tenerse en cuenta que el mtodo de Rit:i da una aproxi
mac n por exceso del valor propio.
EJEMPLO 3. Hallar aproximadamente el primer valor propio del
problema
y" + 'J..'y =
!J (-1) = y (l) = o.

SOLUCIN. El problema sobre el mlnimo de la funcional

r
t

y'2 dx

-t

con las condiciones


1

g(--l)=y(l)=O

y2dx =I

-1

es un problema isoperimtrico y se reduce al prob lema sobre el mini


mo de la funcional
1

J {y (x)] =

J
-1

(y'Z - 'J..ZyZ) dx

CA P. 111. M~ TOD OS DIR ECTOS

174

cuya ecuacin de Euler coincide con la ecuacin dHerencial considera


da y"+ "J..y = O, y (-1 ) = y (1) = O.
La solucin general de la ecuacin es y = C1 cos 'Ax
Ca sen 'Ax.
De las condciont>s de frontera encontramos

C1COSA-Czsenf.. = O, }
C1 cos A.+ C2 sen A. = O,

(10)

de modo que Ja condicin de existencia de una solucin no nula del

sistema (10) es la condicin de que sen2A. =0, o sea, l. = n~


i..~ =

Por consiquiente, para el primer valor propio tenemos


= (

)2 y la pr imera armnica

de la cuerda viene dada por la solu

.. exac t a g =cos nx
c1on

.
"'1
2 , '= T; a

segun da armomca
es y=sennx,

. . es y=cos3nx
, 3
t
11.~ = .n; 1a tercera armonica
- , 11.=2n, e c.
2
A ttulo de comparacin, busquemos las soluciones pares (armnicas pares de la cuerda} aproximadas en forma de un polinomio segn
las potencias de x. Tomando las funciones coordenadas en la forma
q:11 (x) =x2112- x211 (k= 1, 2, .. . } minimicemos la funcional J en las
m

funciones llm (x) = ~ c.1<p1t (x).

Limitndonos al

11-t

=c 1rp1(x), tendremos J (y1 (x)J=-c~


c1 obtendremos

iJJ (.!lt (X)]


OC1

-- Ci

( :-

!~

trmino .111 (x) =

,,2) y para determinar

(!3 __!!
2) =0.
15 "'

Puesto que debe ser c1 =rf:: O, resulta X' = 2,5. Tomando para y

Y2 (x) = c1cp1 (x)

+ Ci'Pll (x),

encontramos

J [Y2(x)]=cl (

~ - !~ t.2 )

+2c1c2 { 15 -

1~5 X )+
2

+cf (
y para determinar

Ct

1: -

y c2 obtenemos el sistema

oJ [Y2(x))
_
(..!!-~ ,,2) + (J!-~ ,,_2 )
Oct
-Ci
3
15
Ca
15
105
oJ l.!12 {X))
OC2

Ct

3\~ 1.2),

=O

'

'\
1

(~32 ,,2 )-j 2 ( 176 _2!_ A,2) =O J


15
105
c
I05 ,315
,

S IS.

LOS VALORES

LAS FUNCIONES PROPIOS

175

La condicin de existencia de soluciones no nulas r1 y e, de este ltimo


sistema es que su determinante sea igual a cero; esto da ).4 - 28>..2
63 = O. de donde hJ = 2,46744 y hl = 25,53256. Comparemos los
valores aproximados obtenidos para hf y A.f con sus valores exactos.

El valor exacto de >..? es (

(~) ~ 22,20661

~) ~ 2,46740
2

y el valor exacto de

A.~ es

de modo que el va lor aproximado obtenido para

A.f es

de gran exactitud mientras que para el segundo valor propio se


obtiene una aproximacin tosca.
EJEMPLO Hallar el primer valor propio del problema
yw
A. (1 x') y = O, y (-1) = y (l) = O.

SOLUCtN. Tomemos como funciones coordenadas las funciones


q>Ji (x)
1 - x2 1t (k
1, 2, ... ) que satisfacen, obvamente, las

condiciones de frontera. Pon iendo

c1 (1 - x2} + e, (1 - x'),
planteemos el problema sobre la minimiz.acin de la funcional

Y2 (x)

J (!I (x)) =

J [y'2-I. ( \ + xZ)

yZ) dx

- 1

que tiene la ecuacin dada como ecuacin de Euler. Tendremos

J [!1'2(x)) =el

( ~ - : ~~ ~.) +2c1c2 (

1
:-

~:A.) 7
2 (

32

+c T 2

588~
3465

"-J

Para determinar c 1 y c'l obtenemos el slstema

iJJ[!12(x)]_ 2
OC

aJ [y2 (x)J 2
OC2
C

Ct

(!128 '- )..L 2


3
104
c
1

(~-~-)
+2
5
45

(~-~i.) - o
5

45

'\

, 1

( 32 _ 5888 ') - o
7
3465 "' -
La condicin de existencia de solucin no nula de este ltimo sistema
da
52~}.I - 1068).
2079 ,.,,
de donde. lomando la ra lz menor. encontramos 1.1 = 2, l 775.
PRINCIPIO oe RAJLEIGH. Supongamos que se tiene el proble ma de
valores prop os
C2

/>.

:~J-j q (x)y=~.r(x)y,
a,y(a) t~tY' (a)= O,
cxf+ ~l>O, }
a2Y (b) + ~211' (b) =O, ex~+~> 0 ,

L(y) s -:x [p(x)

donde p (x), p' (x), q (x) y r (x) son continuas en (a, b}; p (x)
en [a, bl.

(ll"I

( l 2)

>O

176

CAP 111 . Me TODOS 01 'RECTOS

Diremos que la funcin y (x) es admisible (y E D) si tiene dos


derivadas continuas y satisface las condiciones de frontera (12).
Supongamos que para toda funcin admisible y (x) se cumple
la condicin
b

yl (y) dx:;;:. O.

En este caso el problema'de contorno (l 1) - (12) tiene solamente valores propios reales A..
Podemos poner en correspondencia a este problema de valores
propios el siguiente problema variacional:
entre todas las funciones admisibles y (x) tales que
b

Jr

(x) y2 d.x >O.

{13)

a
b

S yl (y) dx
Q

hallar aque lla para la cual

= min.

sr (X)gZ dx
a

Sea y = IJ>1'(x) la solucin de este problema.


Si t..1 es el vafor mnimo, o sea, si
b

S gl (y) dx
l..1=mn:
llED

~ "'tl ('1> 1) dx
-,....
b -- -

S r(x) yz dx
a

! rl!l~ dx

entonces f. 1 es el menor valor propio positivo y ~ 1 (x) es la funcin


propia que Je corresponde.
Si a las funciones admisibles se impone, a parte de la condicin
(13), una condicin ms
b

npydx=O

(condicin de ortogonalidad), el problema


b

~ yL(y) dx

-..,...-- - =mio
~ ry2dx
CI

tendr de nuevo una solucin

Wa (x).

lli.

LOS VALORES

Y LAS FUNCIONES PROPIOS

177

'Si >., es el valor mnimo correspondiente, entonces h., ser e 1


siguiente, en cuanto a la magnitud ().1 ;;;:i: ~). valor propio y 'li1a (x)
ser la funcin propia ortogonal a ~ 1 (x) que fe corresponde. En general, si se conocen ya los k primeros valores propios postivos
"1_~).1~ ~Ak
y el sistema ortogonal correspondiente de funciones propias
~ (x), 1J>ii (x), . . . 'l>.1t (x),
el valor propio siguiente. ser igual a
b

SyL(y) dx
1-k+t = mfn -...b - - l'ED

'i ry2 dx
a

con la particularidad de que se consideran aquellas funciones admisibles y (x) que, a parte de (13), satisfacen las siguientes condiciones
complementariu
b

Jr (x)lj>v (x)y(x)dx=O

(v=l, 2, .. .,k).

a
Si en la ecuacin (11) se tiene que la funcin r (x) >O en [a, b)
con frecuencia se emplea para estimar por arriba el menor valor
propio positivo /..1 la $iguiente desigualdad (principio de ~a yle igh)t
b

S yL (y) dx

-< _R..,b_ __

S ry2 dx
a
EJEMPLO 5. Valindose del principio de Rayleigh, estimar A.i
en el siguiente problema de contorno
-yw = ')..y,
y' (0) = O,
Y (1) = O.
SOLuciN. En nuestro caso tenemos L (y) = -y, o sea, p (x) e
s 1 > O, q (x) s O y r (x) e 1 > O en [O, 1]. E.s obvio que <Xi = O,
~ 1 = 1, a,= 1 y ~s =O de modo que af
~r = l >O y cx.t +
~i = 1 >O. Tomemos como funcin a(imisible y (x) = 1 - x11 ;
seg(tn el principio de ~aylelgh, tendremos

~ yL (y) dx

Xt

S yr2 dx
o

i 2 (l -

x2) d.~

=-.:....1- - - - - =-8- = 2.5.

S (l-x2) dx
o
:n2

15

Recordemos que el valor exacto es A. 1 =--- ~ 2,4674.


12-0t 387

CAP. 111 .

178

Marooos

DIRECTOS

Estimar el menor valor propio en los problemas que


siguen:
229. -y" =A. (10 - x2 ) y; y (-1) =y (1) =O.
230. -y"= 1.y; y (O) = y ( I) = O.
En el problema de la determinacin de los valores y de las fun
dones propi9s tambin se puede emplear el mtodo de Kantorv ich
(mtodo de reduccin a ecuaciones diferenciales ordinarias). Supongamos, por ejemplo, que en un recinto D se tiene la ecuacin
6.z

y que

+ AZ = 0

Zlr=O,
donde r es la frontera del recinto D.
Busquemos la solucin en la forma
m

Zm (x,, Xz, .... Xn)

~ c:r.k (x) tpA (x, y) -1-q>o (x, y)


k-1

escogiendo las funciones coordenadas <flli (x, y) y las funciones a.11 (x),
por ahora incgnitas, de modo que Zin (x, y) se anule en todos los
puntos de
Las funciones Gt1 (x), a, (X), .... CXm (X) deben satisfa
cer el sistema de ecuaciones

r.

J [Mm+A.zmJ<J111(x,y)dy=O

(k=l,2, .. . ,m,

(14)

D:t

y deben anularse en los valores extremos del argumento. Aqu D."


es la in lerseccin del recinto D y de la recta x = const.

Aquellos valores de A. para los cuales el sistema (14) tendr solucin no trivial darn una aproximacin de los valores propios y las
soluclnes correspondientes darn una aproximacin de las funciones
propias.
EJEMPLO 6. Hallar aproximadamente el primer valor propio
y la primera funcin propia en el problema

6.z

+ A.z =

O,

z 1r

=O

donde el recinto D es el rectngulo: - a ~ x ~ a, -b .:<;;'.y ~ b.


SOLUCIN, Buscamos la solucin del problema en la forma

z1

(x, } = (y2

b) c:t 1 (x) .

La eucaci6n (14) toma en este caso la forma


b

J
-b

[2a 1

+ 1y2- b2) a. + , (y2 -

b2) cx 1} y2 -b2) dy = o.

15.

LOS

VALORES

179

1.AS PUNCIONES PROPIOS

o sea,

..!.!!..bsa+ {~ bS).._.! b) a 1 ~~o


15

15

( 15)

'

a1 (-a)=at (a) = O.
La soluc:ln general de (15) es

V,..-_-2. , .~2-x + C2cos yA. - !

a 1 (x)=C1 sen

x.

Teniendo en cuenta la simetra del problema y tomando una solucin


particular, obtenemos

C 1 "-0

C 2 cos

A.- 2: 2

a=O;

queda claro de aqu que tendremos una solucin no trivial slo si

/
V

hk =
En particular, para

, _ 2b2 a=t2k- 1>2:

k~

(2k -1)2 n2
(2a)2

5
2b2

l encontramos
n.2
to
At ,_- (2a2 + 21>2

siendo el va lor exacto


n2

n.2

At = {2a)Z + 2b2 '

El error es menor que 1,3%.


Para la primera fu ncin prop ia obtenemos la aproximacin

z1 (x .

v> = (JJ2 -

nx

b2) cos 2Q.

Hallar una aproximacin del primer valor propio en los


problemas que siguen.
231. y"
i. 2y = O, y (O) = y (1) = O.
232. y"
i. (2
cos x) y = O, y (O) = y (n) = O.
233. Hallar aproximadamente el primer valor propio en ~ I
problema
!!J.z
AZ = O z Ir = O,

+
+

donde D es el crculo de ra<lio uno con ctnlro m el


de coordenadas.

origen

RESPUESTAS E INDICACIONES
J. a) frnsn = O en el punto (0, O); b) fmx = l en el punto (O, O);
e) no hay extremo. 2. No hay extremo. 3. fmfn = -8 en los puntos
(V2. -V2) y ( -'V2, V2); en el punto (O. O) no hay extremo.
4. fmtn = O en el punto (0, O); en los puntos de la circunferencia
x2+ y2 = 1 hay mximo no estricto. 5. f mx =
en el punto (1, -1).

V3

6. f mln = 4 en el punto (-} , 1, 1 ) . 7. f mfn = -1 en el punto {I, O).

3V3

R. lmio---

en el punto

( T2n T2rc) ; lmflx = -sv3


8- en el punto
n2+n+2

; ) 9. mflx:= (

n 2 +~+ 2 )

nz+ n+ 2 . l t. No. 13. Los nmt!'ros

ct1

para

Xt = X2-== ... =X11 =

~h

deben ser los coefi-

cientes de Fourier de la funcin f (x). 14. f mfn =-+en los puntos

( Vl2 ,
(

v12 .

18
( 13,

1)
- V2'

1)
lf2

(-

12 )
13
.

( - V1 , 1) ; lmb. =21
2 112

en los puntos

36 en el punto
y'il . - v12) . t5. tmsn = 13"

16. f mb.=4 en los puntos (2, 2, 1), (1, 2, 2} y (2, I, 2);

fmh.=42 en los puntos

("334 3
7). (73 34 34)

42

4 3
7 34)
(3

(: , ! );f

17. lmx

=t

mx = 11 en t punto ( -

18. fmtn = I

, -

! ).

en

el

punto

19. f mln = -9

en el punto (- 1 2, -2); f mx =9 en el punto (1 , -- 2, 2). 20.

RESPUESTAS E INDICACIONES

(~. ~' ~).

fmx = i-enelpunto

181

21. Jndicacfin. Hallar el

mnimo de la funcin z=+(x" +!I") con la cond icin x+y = S.

4V5
-.

22. e . 23. -

'

a= R

19112

24.

25.

El cuadrado de dimensin

v2. 26. El radio de Ja base del cilindro

y la altura del cilindro h

=R

V 2-Vs.

= ~ -,/ 2+

Vs

27. Pr imero. 28. Proximi-

dad de cualquier orden. 29. Proxim idad de cualquier orden. 30. p = e-1.
31. P=I. 32. p=e- 1. 33. P1=e-I. 34. 1>2=

2n+3
. 36. P1001-=e.
6

36. Continua. 37. Continua. 38. Discontinua ( considerar la sucesin


!In (x) = sennnx } . 39. a) Disconl nua; b) continua. 40. a) Discontinua

1-e2
2

b) continua. 41. Continua. 45. AJ = --

48.

a;

-0,I
0,01

49. M

.J

6.J

1,2

1
-0,I
0,01

-0,098
0,01002

3 (e2- l) a
4

+6 (3 -

0,1
0,01

a;Z

=a:+ 5; oJ =a:.

- 3 (eZ-1) ex.
e) a i+~., J
u 5

(J.J

4,7919

6,6821

0,4792
0,0479

0,4963
0,048 1

50. 1) Diferenciable; 2) diferenciable: 3) diferenclable; 4) no diferenciables. 51. 6J2 [g (x)] =21 (g (x)J aJ.

63. 6.J = 3k

e
+-e- . k2;
A

3J =3k.
M

4,582

0,1
0,01

0,3158
0,03016

3
0,3
0,03

RESPUESTAS E I NDICACIONF.S

182

5
8
5
k2 61=3k
5-4. llJ--k+- 3
7
'
6J

t:.J

"

2,810

0,1
0,01

0,181
0,0168

1,667
0,167
0,0167

55. llJ=Tk?.; 6J =O.


lt

6J

AJ

o
o

f,3333

-1
0,3
0,03
b

0,1200
0,0012

J~ydx.

57. 61 =

58. 6J = 2

J(yy-y'y' )dx.

a
1

69. 6J = 2y tO) 6y {O)

+ ) (x ~11 + 2y' 6y') dx.


o

80. 6J =

J(J/ cos

y {Jy +sen 6y') cU.

o
I>

61. 61 =

rl (-0ar-

Yt

at

yt + -- 6112 + . . .
0112

' .)

+r
yn
!111

dx.

62. fiZJ (y, y)= 2J (y, y).


63. 6teF<11>=eF(11) {(6F)2
I>

SS.

(JZJ

m
~

+62FJ.

= J L.J a 111iiJ2fay<i>
a .Ir, t - 0 11

68. f>ZJ=

67. 6"'J=

6y<1t>fiycll dx.

f (F~ {&)2+ F ,;r6z~+ ... + F..,, (6z)2jdxdy.


JGJ
Uic
Zr,%71
I>

i, 111

J[ ~

F; 111k6y18t111+

~
i, .11-1

F; .:6y16!1i +
111

TI

RESPUESTAS E INDICACIONES

183

68. Considerar 11\ funcional


J [cp+a:r1J .:mil) (a)
y emplear la segunda definicin de la variacin.
6J =0, llegamos a la ecuacin integral

Exigiendo que

JK

(s, t) cp (s) ds+cp (t)-f (t) =0.

69. Proced iendo de la misma forma que en el problema anterior.


encontramos que la !'cuacin funcion al de Euler , que expresa la anulacin dt> la primera vnriacin, viene dada por
(p~')' -

ci (x+2)- q> (x-2) +<J> (x)+ f (x) = O.

Esta ltima es una ecuacin mixta con derivadas y diferencias.


70. -(pq:')'+q<p :=::: /(x). 71. y= -x3, 72. y=

sh~~~x)

.73.Dos

extremales
1+<3 +

y=

2-V2><2x -1>2
<lf2 1>

74. Dos extremales y=f"<x+l )Z e u=l"'l3x-1)2 75.

+ x) sen x,
x

donde

[e-x +O + e)

C es

xe-~-

una

77.

lJ.

tt=<C+

constante- arbitraria. 76. y =

y= 67 x- 61

x3

78.

2
13
y=T x-

- tf x3+ 2. 79. y= In x. 81. La integral no depende dd cam ino


de Integracin; el problema variacional carece de sentido. 82. y=O
si a=O; siendo a =fr. O, no existe extremu l suave. 83. y=cosx.
84. y=cosx+Csenx. donde Ces una constante arbitraria. 86. 11=
sh X
=x+I. 86. Y=ShT 87. y=e2tt-X>. 88. No hay extrema!es; la
ecuacin de E u ler no

t iene soluciones. 89.

90. No hay exlrcmales. 93. y=C 1e:-: +c 2e-x -

96. y=

y =C1 -t-C2x1
yxex:.

~z. .

94. y=2chx.

Yt sen x
. 1
. 97. y=2x . 98. La clrcunferencia -=K. 99. y=
seo xi
'

=(1-x)shx.

!OO.

u= 6x3

(x3+6x+I). IOJ. No hay extremo. 102.

184

RESPUESTAS E INDICACIONES

El problema variacional carece de sentido porque bajo el signo de la


integral figura una diferencial exacta. 103. y=sh x.

104.

11=+x2.

J06. {

Y=

a = sen 2x,
105. {

x2

Z= - 2+

-{-cx3+Sx-6l.

32+n2
8n

X.

u=sen .x,

107. {

z = x.

z =sen x.

x2

tos.

Y=-y+I,
{

Z=

ilz
110. ( '"Fi

l.

)2 '(jX2
iJ2z
iJz )2
+ ( ay

01-z

iJy2 =

f (X,

y).

f (X1,

X2, ., Xn)

113. Solucin. El planteamiento del problema es el siguiente.


las superficies z = <p (X, g) que se proyectan en el reciQ.to D del
ptano xOy y que pasan por cierta curva cerrada alabeada cuya proyeceln es la curva frontera r del recinto D.'!ha llar la superficie cuyo
1ea
Ent~e

S ==

j l V 1+ cp! + cp;

dx dy

!)

sea mfnlmo (problema de Plateau). La ecuacin dHerencial de Euler


para este problema es

q>$

-;- YI +cp~+cp;

+..!..
~y

'Pv

=O

lft+qi!+IP:

o en forma desarrollada

cp= (1 + cp;) - 2q>;rvq>$<p + 'Pvv ( 1+ cp!) =O.


Esta es la ecuacin diferencial buscada de las superficies de rea
mnimo. La realizacin Hslca de la superficie de rea mlnima se puede
obtener, por ejemplo, con una pelicula de jabn tendida sobre un lato
de alambre.
114. z (x, 11) = !I E l problema tiene solucin nica aunque las
~dicione& de frontera no se dan en toda la frontera.

RESPUES T AS E l NO l C l\CIONE$

115. r cos qi+Ca = C1 In 1r sen cp +


,z seni
117. xi cos C2- y 2 cos Cz-2xy sen C2 = Ct

185

q-cn

lt8. Campo central. 119. a) Campo propio; b) campo centra l;


c) no forma campo. 120. Campo propio. 121. a) Campo central; b) no
forma campo; c) campo propio. 122 . a) Campo central; b) campo propio; c} no forma campo. 123. No lorma campo porque esta familia
de curvas no cubre lodo el recinto D. 124. y =C1 ch x forman un campo
propio de extrema le~; y = C2 sh x forman un campo centra 1 de ex lremales. 125. y= Ccos.v: forman un campo propio de exlremales; y=
= C sen x forman un campo central de extrema les. 126. La extrema l

IJ = ~ (l -x2) puede ser incluida en el campo central de exlremales

x3

y ..c C1x6 con centro en el punto O (0, O). 127. La extrema! u=ex
se puede inc lu ir en el campo propio de extrernales y=ex+C. 128. Si
a < n, la extrema\ y =-~ O se puede incluir en el campo central de extremales y=Csenx con centro en el punto 0(0. O). Si a >n, las cur
vas de la famil ia y= C sen x no forman campo. 129. La extrema! y=
=x+ l se pede incluir en el campo propio y=x + C. 130. y=
:e 131. y ( ~
= O. 132. yi-1 = O. 133. O (1, O).
134 . No hay punto conjugado. 135. Se cumple. 136. Se cl!TTlple cual
quiera que sea a. 137. La condicin de Jacobi se cumple. La extrema!
y= O se puede inclu lr tanto en un carr.tpo central como propio de extrema les. 138. La condicin de J acobi se cumple. La extrema! y=
!..=l..
x+ l se puede incluir en un campo central de extremales con
(l
centro en el punto A (O, l}. 139. La condicin de Jacobi no se cumple .
142. Se puede. 143. Se puede. 144 . Se puede. 145. Se puede aunque la

x: .

-x)

condicin de Legeri~re se cumple slo para


<l. 146. Se alcanza
mrnlmo fuerte en la funcin y=t%. 147. Se alcanza mnimo fuerte
en la funcin y= 2 In (X + l). 148. Se alcanza mlnmo dbil en la funb
cin 11=~2 . .1.49. Se alcanza mi_nimo dbil.. en la recta 11= - x.
a
.
In (l+x)
. UH. Se al 160. Se alcanza mnimo fu~rte en la curva y
ln
2
canza mximo fuer te en la curva g = cosx+senx. 152. No se alcanza
extremo en curvas continuas. J53. Se alcanza mlnimo dbil en la recta
11- 2x l. No hay extremo fuerte. 154. Se alcanza min imo fuerte en
la extrema! 11=2x- l. 156. Se alcanza mlnimo fuerte en la extremal
~=x. 156. Se alcanza mlnimo dbil en la extrema] y = x- l. 157.
la extrem11l 11=.!.x se alcanza mnimo dbil si 1'11 < ~

En

y mhill)Q

a~t>il

:si

'b' >

y2 .

v2

No hay extremo si

1 1=

V2 .

186

RESPUESTAS E INDI CAC IONES

158. Si p =F q, se alcanza minmo db il en

la extrema\

y=

= {[(q3 12 .--p3 121 x+p3 12 )1; si p=q, la extrema! es la recta !J=P


que ofrece mnimo dbil.

sh
159. a) Para e> O la extrema] !1 =

sh
fuerte de Ja func ional. b) Si

E.<

o y

1e'>

y;X

realiza el mlnimo

nz1 .

la extrema! !I =

V8

sen -=--

V ~ .,,

realiza cl mximo ruerte de la funcional. e) S e = O,

sen V 1 el
no existe solucin di!! pro blema extrema! en la clase de funciones
x-1

continuas. Consideremos la funcin Yr. (x) =e. Ve (e> 0) que es


solucin de la <.'c uacin de Euler ey"-y=O de la funcional conside
rada. La funcin Ye (xl satisface la cond icin de frontera y (1) = 1
y no satisface la segunda condicin de frontera g (O)= O. No obstante,
lim !Ir. (0} =O. Para e-.. O obtenemos de !Je (x) la csoluc in limilet

e-o

!J (X)=

{ 1,

O ~x< I .

x= J.

2 In li J.. X)
.
realiza el mnimo fuerte.
In
2
161. Hay mn imo fuerte en la extrema) y(x)=I. 162. En la extrema]
160. La extrema! y= -

b
. .
de. b.,
b
va2 - y max1mo
, . d.eb.11
x =-ax
se a 1canza mm1mo
1 si. -a<' b
113 . b lf3
1
. .
1 t
SI a > - - ; SI a=- - , no se a canza n1 s1qu1era e eX remo
y( )

dbil. 163. En 111 recia y ..

~x se alcanza mnimo dbil si b<a


a

va.

y mximo dbil si b >a si b ;p. a


hay mximo fuer te mientras
que para b <a
no hay ni mn imo ni mximo fuertes. 164. Se
alcan:ia minimo dbil en la extrema! Y=2x, z=4x. 165. La extrema!

V3

es la parbola { !I = x~
Z

tral de extremales

x~

, que se puede inclu ir en el campo cen-X

{ 1)

187

RESPUESTAS E 1 NOICACJONES

a y ~ son unos parmetros) con centro en el punto (O, O, O). Es


obvio que se cumplen las condiciones reforzadas de Legendre. Demos1 no hay punto x conjugado del
tremos que en el segmento O~ x
punto x = O. Para ello bastar persuadirse de que las extremales de la
familia {I) no cortan la extremal dada si x E [O, IJ. Supongamos que
en un punto x E(O, 1) se cortan dos extremales e la familia (\).
Tendremos entonces

<

a 1x=a2x ,
}
x 2 + ~tx = x 2 + ~zx.
De aqui resu!ta que a 1 = a.2 y que ~. = ~ 2 Luego, no hay dos extrema les distintas que se corten. Por consiguiente, la condicin reforzada
de Jacobi se cumple en e l segmento
1) e incluso en cualquier segmento de lonttud finita. 166. La familia de extremales es g (.x) =

ro.

= C 1 ch .xCt

A.. Las constantes arbitrarias C1 y C2 y e l parme-

tro A. se determinan de las condiciones

Xo-C2

!lo= e t ch

A.,

C1

lCi

) VT+7idx==C1 (sh x1-z.C2

}-I
h xo-C3
C1
- .

X&

167. g (x) = 3x2

+ 2x + J.

168. g (.x) = 2 sen rutx, donde n es un


1
,rmimero entero 169. y(x)=T(2x-xZ), 170. 11 6. 171. r = R. z=C1

lfTI
-r-.

, r-

4
VS
.

+Czcp. 172.

, r-

173. V 20. 174. 2 V 2-1. t76.

'\IO
-o-.

-.r-)

5
179. lyf" 17 +4 ,
Vr
6 ( 2-11
6 . 180. \. 181. Si cos

el extremo se puede alcanzar solamente en la rec ta {

178.

Xt :{;

~=~

0,

En

cambio, si cosx 1 = 0 , o sea x 1 = ; +me, donde n es un nmero entero,

se tiene {

y=C,senx,
Z= -

6 .sen X,

siendo Ci. una constante arbitraria. 182.

26

J (A, B) = 4 cth l. 183. J {A, 8) =T. 184. !J = 2x 213 . 185. Las lneas

quebradas formadas por los segmentos de las rectas y = x e y :::;:. 1 o por


los segmentos de las rectas g=O e u=x-1 realizan el mnimo abso-

lut~.

La recta

Y=+x

realiza el mximo dbil. 186. !/=

-x

para

RE SPUESTAS E IN DICACIONE S

188

O<x<I; y=x-2para I<.1:~4ey=xparaO<.x~3; Y=-x+6


para 3 < x 4. En ambas quebradas la funcional alcanza su mnimo

<

o,

abso1ulo. 187. No existen. 188. y= {

x~o.

189. Las extremales


x, x> O.
son lneas rectas. Si J Yz- Yt / < 1, existen dos soluciones discontX2-x1
nuas que son lneas quebradas paralel as a las bisectrices de los ngulos coordenados. 190. La recta y =x tg <p que pasa por los puntos ijos
realiza mximo dbil si O<tgq><n, realiza mnimo dbil si n<
< tg q < 2Jt, etc. El mnimo fuerte se alcanza en la lnea quebrada
formada por segmentos de rectas tales que la ta ngente de los ngulos
1
de su inclinacin es igual a
n (n es un nmero entero) .

4n;-

191.

y (X)

c ,

.- 16
o ' x~5.

Tx,

r ~V9 -- (x-

16

_ 3(x - JO)

.-

...... 34

5"'-x..,.,,5,

5)2.

34

T<x ~IO.

192. Las exlremales son las elipses

( 11

con centros en el eje Ox. La frontera del recinto admisible se determina por las ecuaciones y=O e y2= 2(x-C3) (la ltima es la
solucin de la ecuacin 1-y2y'2= 0). Los parmetros C1 y C2 se escogen de modo que Ja elipse (1) pase por los puntos fijos A y B. La funcional
alcanza mximo en el arco de la elipse. Si el cam ino del punto A al
punto B se escoge segn los arcos de dos parbolas (y, posiblemente,
segn un segmen to de la reda y= O), se obtiene una solucin con
puntos angulares en la que la funcional akam:a su mnimo (mn J =O).
dy
x2y2
dp
xZy
dy
p
dp
pZ
193

195.

d'.t =

!!Y_
dx

P1

1=

'

'

-Vpz,

'

dx

Vx2... y2- pz

dp
O
~=
clp _

=2
_ J!L
2y2

4p2

dx

' dp =
dx

194

dx

2xg '

dx = 4xy2'

190.

y xz+ y2-p2

dyt =
dx

dp2
2
197. dyt =..!!2._ dyz
([X' = Y2
dx
2y 1 ' -=
dp2
P~
tl!Ji
Pt
d112
198
{[X= -7' Tx =
2y~ ' -;;-- -- 4y~
dy2

P2

d'X'""'=T'

Pr

dp1 '-' 2x,


dx

=2;)

dp2 -0:
dx

cfy1

dx""

Pt

- 2

'

dy2

-- =

~ -

V pz,

189

RESPUESTAS E INDI CACION ES

~I

=2X,

~~

=C1

J V oi C!l>-C:

= 0. 199. g3=C1x + C2 . 200. y3 =1 n2x. 201. )("'

dy

x2-x - l
s~
2

+C2 202. En la extrema! 11 =

alcanza el mn imo fuerte: mn J = -

. 203. p (x, y )

112

2~gxz

Las exlremales son las semicircunferenc ias g = V q-(x-C2)Z con


centros en el eje Ox; g = V 2C1x - .x2 son las extrema les que pasan
por el origen 0(0, O); el campo es el semiplano superior. 204. E l arco
de la circunferencia que pasa por el punto M 1 (x1 , y 1) y que t iene el
centro en el punto O (O, O) real iza el mnimo fuerte. 205. xP (~) =C.
206. Las elipses 3xZ- 8xy + 6y2=C. 207. x3+2gS - 3xgZ-2x2y=C.
2os. /=Yt+72. 209. f = xyVy'. 21 0. f = xy1/z. 211. t=

=v(*+ ;2)

22

(x y' +y2), 212. Una catenaria. 213. Indicacin.

La integral de la accin es J =
tra:;cctotias son las elipses

JV P~

xz

2y2

-e+-2h -

215. La solucin exacta es 11 =

:~ ~

+2h

e -

V p2-l- p'2 dcp. 214. La~


2 cos p

5~h; -

xy = - k -

- x. 216. Las soluciones exactas

son a) y e= O, b) Y = X. 217 . La solucin exacta es !I =


La solucin exacta es g=

sen2 ~

. C{2h-C)

(x2 - x). 218.

x. 219. Indicacin: la solucin apro-

xlmada debe buscarse en Ja forma lln (x)

= (J-x2)

11

~ <X1tx21t, La so11-0

1U

lucin exac:ta es 11 = cos T

coordenada; entonces,

Zt

220. lndlcacln: tomar xy como funcin

b2 -al!

b?+az xy. 22l. lnd1cac16n: tomac cp0 (x, y)=

= xi + yz,

q>t(x, y)=xy ( l - x - g), 1J>2. (x, g)=x2y (l-x-y), ...


. . ., Cl>n (x, !1) =x'lg(l -x - y) como (unciones coordenadas; entonces,

23

(x, y)=xz+11z+xy(l-x-y)[3,0401-0,0562(x+1C2)]. 222. Indica-

cin; hallar la primera aproximacin en la rorma z1 (x, g) = ( y2


2

)a(x);

entonces

.<i(x, y) ... -

~ (1 - ~ ) (y2- ~

).

~ESPUESTAS

190

E I NDICACIONES

3 ( yi- 1 xi ) ( 1 _ 35 x- 2_. 35 x-5-1 ) . 224. "-n=


223. zt(x. y) =-1 311
1 36
3
2
2
= 1 n .n , !In lX) = 2 sen nnx (n = l, 2, .. ).
225.
h.,.=

\!

(To;-- In -x}

lnZ 2+ 4nztrz
)- sen
4 lnZ 2
' !In (x -

_+

!In (x) - _

\! 2 sen\!x(nn In x)

=y! sennx

'

V 1nV2v x
... 1 2

(n -

226 '
25 + 2n2n2
"'n =
4

) 227 ,
)
11.,1 = 1 -n 2 , !In (X =

(n = 1, 2, .. . ).

228.

nn In (l+x)

13 ln2 2+4nznz
' Yn (x) =
4 lnZ 2

J.

sen [
In 2
i r---,--= , / V In V 2 V 1

+x

(n = I, 2, ... ).
229. Tomando y = 1- xi, encontramos A.1
1

-<

!.

3
1

El valor exacto

es A.1 =T. 230. Tomando y= x ( 1-x), encontramos A. 1

-< 10. El valor

exacto es
231. A.( = 10 el valor exacto es A.f = n 2. 232.
= 0,493. 233. A., = 6; Zt (x, JI)= IX (xZ gZ - 1).
;>.. 1 = :rt2

A., =

A NUESTROS LECTORES:

MIR-RUBIOS 1860 edita libros soviticos traducidos al espaol. Entre ellos figuran las mejores obras
de las distintas ramas de la ciencia y la tcnica; manuales para los centros de enseanza superior y escuelas tecnolgicas; literatura sobre ciencias naturales y
mdicas. Tambin se incluyen monografas, libros de
divulgacin cientfica y ciencia ficcin.
Dirijan sus opiniones a MIR-RUBIOS 1860.
C/. Alcal, 98 - 28009 Madrid.

Al priocJp10 de cada capitulo


se resumen los resultadQS prin
cipales. se exponen los conoei
mientos tericos necesarios, ltls
rmulas requeridas y se estudian con gran detalle ejemplos
tpicos i lu.strativos.
Este manual contiene ms de
100 ejemplos analizados y 230
problemas destinados para resol
verse independientemente. Unos
problemas se acompaan con
las respuestas, otros, con las
refereot'ias de cmo deben resol verse.
La obra est destinada para los
estudiantes de centros de enseanza tcnica superior que se
especializan en los clculos ma
tcm lieos.

ISBN 84-604-1605 - 4

Você também pode gostar